График парабола: Построение графика квадратичной функции — урок. Алгебра, 8 класс.

Пошаговое руководство построение графика квадратичной функции

Для того, чтобы начертить график функции в Прямоугольной системе координат, нам необходимы две перпендикулярные прямые xOy (где O это точка пресечения x и y), которые называются «координатными осями», и нужна единица измерения.

У точки в этой системе есть две координаты.
M(x, y): M это название точки, x это абсцисса и она измеряется по Ox, а y это ордината и мерится по Oy.
Две координаты отображают расстояние от точки до двух осей.

Если мы рассмотрим функцию f: A -> B (где A — область определения, B — область значений функции), тогда точку на графике данной функции можно представить в форме P(x, f(x)).

Пример
f:A -> B, f(x) = 3x — 1
If x = 2 => f(2) = 3×2 — 1 = 5 => P(2, 5) ∈ Gf (где Gf это график данной функции).

Квадратичная функция

Стандартная форма: f(x) = ax2 + bx + c

Вершинная форма: $f(x)=(a+\frac{b}{2a})^2-\frac{\Delta}{4a}$
где Δ = b2 — 4ac

Если a > 0, то минимальным значением f(x) будет $-\frac{\Delta}{4a}$ , которое получается, если $x=-\frac{b}{2a}$. Графиком будет выпуклая парабола, вершина которой (точка, в которой она меняет направление) это $V(-\frac{b}{2a};-\frac{\Delta}{4a})$.

Если a < 0, то минимальное значение f(x) будет $-\frac{\Delta}{4a}$ , которое получается, если $x=-\frac{b}{2a}$. Графиком будет вогнутая парабола, вершина которой это$V(-\frac{b}{2a};-\frac{\Delta}{4a})$.

Парабола симметрична относительно прямой, которую она пересекает $x=-\frac{b}{2a}$ и которая называется «осью симметрии».
Именно поэтому, когда мы присваиваем знаячения x, то вибираем их симметричными относительно $-\frac{b}{2a}$.
При построении графика, точки пересечения с осями координат очень важны.

|. Точка, расположенная на оси Ox имеет форму P(x, 0), потому что расстояние от неё до Ox равно 0. Если точка находиться и на Ox и на графике функции,то она также имеет вид P(x, f(x)) ⇒ f(x) = 0.

Таким образом, для того чтобы найти координаты точки пересечения с осью Ox, мы должны решить уравнение f(x)=0. Мы получаем уравнение a2 + bx + c = 0.

Решение уравнения зависит от знака Δ = b2 — 4ac.

Иммем следующие варианты:

1) Δ < 0,
тогда у уравнения нет решений в R (множестве действительных чисел) и график не пересекает Ox. Форма графика будет:

или

2) Δ = 0,
тогда у уравнения два решения $x_1=x_2=-\frac{b}{2a}$
График касается оси Ox в вершине параболы. Форма графика будет:

или

3) Δ > 0,
тогда у уравнения два разных решения.

$x_1=\frac{-b-\sqrt{\Delta}}{2a}$ и $x_2=\frac{-b+\sqrt{\Delta}}{2a}$

График функции будет пересекать ось Ox в точках M(x1 и Ox. Форма графика будет:

или

||. Точка, находящаяся на оси Oy имеет форму R(0, y), потому что расстояние от Oy равно 0. Если точка находиться и на Oy и на графике функции, то она также имеет форму R(x, f(x)) ⇒ x = 0 ⇒ R(0, f(0)).

В случае квадратичной функции,
f(0) = a×02 + b×0 + c ⇒ R(0, c).

Необходимые шаги для построения графика квадратичной функции

f: R → R
f(x) = ax2 + bx + c

1. Составляем таблицу переменных, куда заносим некоторые важные значения x.

2. Вычисляем координаты вершины$V(-\frac{b}{2a};-\frac{\Delta}{4a})$.

3. Также записываем 0 в таблицу и нулевые значения симметричные $-\frac{b}{2a}$.

или

4. Мы определяем точку пересечения с осью Ox,решая уравнение f(x)=0 и записываем корни x1 и x2 в таблице.
Δ > 0 ⇒

Δ < 0 ⇒ точек пересечения нет. В этом случае мы выберем два удобных значения, которые симметричны $-\frac{b}{2a}$

Δ = 0 ⇒ график касается Ox прямо в вершине параболы. Мы снова выберем два удобных значения, симметричных $-\frac{b}{2a}$. Для лучшего определения формы графика мы может выбрать другие пары значений для x, но они должны быть симметричны $-\frac{b}{2a}$.

5. Мы наносим эти значения на систему координат и строим график, соединяя эти точки.

Пример 1
f: R → R
f(x) = x2 — 2x — 3
a = 1, b = -2, c = -3
Δ = b2 — 4×a×c = (-2)2 — 4×1×(-3) = 16
$-\frac{b}{2a}=\frac{2}{2}=1$ ⇒ V(1; -4)

1. $-\frac{\Delta}{4a}=-\frac{16}{4}=-4$

2. f(0) = -3
Симметричное 0 значение относительно 1 равно 2.
f(2) = -3

3. f(x) = 0 ⇒ x2 — 2x — 3 = 0
Δ = 16 > 0
$x_1=\frac{-b-\sqrt{\Delta}}{2a}=\frac{2-4}{2}=-1$

$x_1=\frac{2+4}{2}=3$

Мы нашли точки:
A(-1; 0)
B(0; -3)
V(1; -4)
C(2; -3)
D(3; 0)

График будет иметь вид:

Пример 2
f: R → R
f(x) = -x2 — 2x + 8
a = -1, b = -2, c = 8
Δ = b2 — 4×a×c = (-2)2 — 4×(-1)×8 = 36
$-\frac{b}{2a}=\frac{2}{-2}=-1$ ⇒ V(-1; 9)

1. $-\frac{\Delta}{4a}=-\frac{-36}{-4}=9$

2. f(0) = 8
f(-2) = 8 (симметричное 0 значение относительно -1 равно -2)

3. f(x) = 0 ⇒ -x2 — 2x + 8 = 0
Δ = 36
x1 = 2 и x2 = -4

A(-4; 0)
B(-2; 8)
V(-1; 9)
C(0; 8)
D(2; 0)

Пример 3
f: R → R
f(x) = x2 — 4x + 4
a = 1, b = -4, c = 4
Δ = b2 — 4×a×c = (-4)2 — 4×1×4 = 0
$-\frac{b}{2a}=\frac{4}{2}=2$ ⇒ V(2; 0)

1. $-\frac{\Delta}{4a}=0$

2. f(0) = 4
f(4) = 4 (симметричное 0 значение относительно 2 равно 4)

3. f(x) = 0 ⇒ x2 — 4x + 4 = 0
Δ = 0
x1 = x2 = $-\frac{b}{2a}$ = 2

A(-2; 9)
B(0; 4)
V(2; 0)
C(4; 4)
D(5; 9)

Пример 4
f: R → R
f(x) = -x2 + 4x — 5
a = -1, b = 4, c = -5
Δ = b2 — 4×a×c = 42 — 4×(-1)×(-5) = 16 — 20 = -4
$-\frac{b}{2a}=\frac{-4}{-2}=2$ ⇒ V(2; -1)

1. $-\frac{\Delta}{4a}=-\frac{-4}{-4}=-1$

2. f(0) = -5
f(4) = -5 (симметричное 0 значение относительно 2 равно 4)

3. f(x) = 0 ⇒ -x2 + 4x — 5 = 0, Δ < 0
У этого уравнения нет решений. Мы выбрали симметричные значения вокруг 2

A(-1; -10)
B(0; 5)
V(2; -1)
C(4; -5)
D(5; -10)

Если область определения не R (множество действительных чисел), а какой-то интервал, то мы стираем часть графика, которая соответствует тем значениям x, которые не находятся в данном интервале. Необходимо записать конечные точки интервала в таблице.

Пример 5
f: [0; +∞) → R
f(x) = x2 — 2x — 3
a = 1, b = -2, c = -3
Δ = b2 — 4×a×c = (-2)2 — 4×1×(-3) = 16
$-\frac{b}{2a}=1$ ⇒ V(1; -4)

1. $-\frac{\Delta}{4a}=-4$

2. f(0) = -3
f(2) = -3 симметричное 0 значение относительно 1 равно 2)

3. f(x) = 0 ⇒ x2 — 2x — 3 = 0, Δ = 16
x1 = -1 ∉ [0; ∞)
x2 = 3

A(0; -3)
V(1; -4)
B(2; -3)
C(3; 0)

Квадратичная функция и её график

Вершина этой параболы находится в точке (0; 0). И не забудь про то, что ветви параболы бесконечно поднимаются ввысь и не ограничены точками с координатами (3; 9) и (3; -9).

Еще одна стандартная парабола задается функцией y = —x(в этом случае а = -1). Для этого графика я тоже напишу табличку:

x -3 -2 -1 0 1 2 3
y -9 -4 -1 0 -1 -4 -9

 

Начало координат тоже является вершиной этой параболы, как и в предыдущем случае, но ветви уже будут направлены вниз:

Сразу напрашивается вывод: если перед х2 стоит положительное число, то ветви параболы направлены вверх, если отрицательное — то вниз.

Если у тебя черный пояс по рисованию стандартных парабол, то следующий раздел пройдет у тебя «на ура».

Параболы со смещенной вершиной.

Зачем я начала статью со стандартной параболы? Ответ прост. Графиком любой квадратичной функции y = ±x2 + bx + c (обязательно коэффициент перед х2 должен равняться ±1) является стандартной параболой, только вот вершины этих парабол не будут находится в начале координат.

Чтобы начертить подобные параболы нужно сначала узнать, где находится вершина.

Пусть вершиной параболы будет точка О с координатами (x1; y1). Тогда найти эти координаты можно по формулам:

Кстати, можно найти координаты вершины и другим способом.

Координату хО находим по той же формуле, а координату уО можно найти подстановкой координаты хО в функцию.

Без примера не обойтись)

Пример 1.

Дана функция y = x2 — 4x + 4. Найдите вершину параболы и постройте график.

Найдем сначала вершину параболы двумя способами, чтобы убедится, что оба способа рабочие.

1 способ: по формулам.

2 способ: подстановкой.

Одну координаты мы уже нашли по формуле. Подставляем ее в исходную функцию.

Итак, получили, что О(2; 0) — вершина параболы. Отмечаем ее на координатной плоскости.

Перед х2 стоит положительное число, значит ветви параболы направлены вверх. Наша задача: нарисовать стандартную параболу, представив, что точка О — начало координат. Если тебе это сложно сделать, то необходимо начертить таблицу значений и уже по ней рисовать параболу.

Параболы-стройняшки и параболы-пухляшки.

Удивительно, но числовой коэффициент перед х2 оказывается влияет на стройность и полноту парабол.

Если числовой коэффициент лежит в промежутке (-1; 0) ∪ (0; 1), то парабола будет более обширно смотреться на координатной плоскости.

А если числовой коэффициент лежит в промежутке (-∞; -1) ∪ (1; +∞), то парабола будет прижиматься к оси Оу и занимать меньше места на плоскости.

Не веришь? Давай проверим! Для примера возьмем две функции:

К сожалению, здесь схитрить не получится: обе параболы нестандартные и для обеих необходимо создать таблицы значений. Но перед эти определимся с их вершинами.

Пусть вершиной первой параболы будет точка А(хА; уА), а вершиной второй параболы — точка B(хB; уB). Вершины буду находить по второму способу (см. выше).

Переходим к таблицам значений.

Голубая парабола.

x 0 2 4 6 8
y 3 6 7 6 3

 

 Зеленая парабола.

x -1,5 -1 -0,25 0 1
y -3 1 4,5 3 -3

 

Чертим обе параболы по получившимся координатам.

Вот о чем я и говорила) Перед тобой парабола-стройняшка и парабола-пухляшка во всей красе.

А ты заметил, что свободный член в уравнении функции — это точка пересечения графика с осью Оу? В обеих функциях свободный член равен 3 и графики пересекают ось Оу в точке с координатами (0; 3).

Практикум по параболам.

Теорию о параболах можно еще писать и дальше, но тебя, скорее всего, интересует практика по графикам.

Поскольку речь идет о параболах, то с параболами мы и будем сейчас возиться.

 

Задание 1. На рисунке изображены графики функций вида y = ax​+ bx + c. Установите соответствие между графиками функций и знаками коэффициентов a и c.

Решение. Коэффициент а, стоящий перед х2, отвечает за направление ветвей параболы, а свободный член с — за пересечение графика с осью Оу.

А) Если коэффициент а положителен, то ветви направлены вверх; если коэффициент с отрицателен, то график пересекает ось Оу ниже нуля. Подходит график 1.

Б) Если коэффициент а отрицателен, то ветви направлены вниз; если коэффициент с положителен, то график пересекает ось Оу выше нуля. Подходит график 3.

В) Если коэффициент а положителен, то ветви направлены вверх; если коэффициент с положителен, то график пересекает ось Оу выше нуля. Подходит график 2.

 

Задание 2 (наоборот). На рисунке изображены графики функций вида y = ax​+ bx + c. Установите соответствие между графиками функций и знаками коэффициентов a и c.

А) Ветви направлены вверх, значит а > 0; график пересекает ось Оу выше нуля, значит и с > 0.  Подходит вариант под номером 3.

Б) Ветви направлены вверх, значит а > 0; график пересекает ось Оу ниже нуля, значит и с < 0.  Подходит вариант под номером 1.

В) Ветви направлены вниз, значит а < 0; график пересекает ось Оу выше нуля, значит и с > 0.  Подходит вариант под номером 2.

 

Задание 3. Установите соответствие между графиками и их функциями.

График В отличается от остальных тем, что его ветви направлены вниз. За направление ветвей отвечает коэффициент перед х2 — он отрицательный. Отрицательный коэффициент только в функции под номером 3. Значит В-3.

Дальше рекомендую отработанную годами технику. Она минимизирует твои ошибки, если ты, конечно, умеешь считать)

Итак, рассматриваем график А и выбираем на нем точку с красивыми координатами (красивые значит не дробные). Мне нравится тут вершина. Ее координаты (4; -3). Даже не спрашивайте почему не прорисованы оси; эти задания взяты с сайта ФИПИ)

Теперь эти координаты подставляем в оставшиеся функции: вместо у подставляем -3, а вместо х подставляем 4. 

Подставляем в первую функцию: -3 = 2 · 42 — 16 · 4 + 29; -3 = -3 — верно. Значит, А-1.

И остается Б-2.

 

Задание 4 (наоборот, но принципе тот же). Установите соответствие между функциями и их графиками.

Очевидно, что В-2.

На графике 1 выбираем точку. Вершина снова четкая, но для разнообразия давайте возьмем другую точку, например, точку с координатами (-4; 1). Будь внимателен и смотри, чтобы точно такой же точки не было на третьем графике!

Подставляем в функцию А: 1 = (-4)2 + 4 · (-4) + 1; 1 = 1 — верно. Значит, А-1.

Соответственно, Б-3.

 

Если ты считаешь, что чего-то не хватает или у тебя есть ещё задания из первой части, связанные с параболами, — напиши мне в VK)

OpenAlgebra.com: Графические параболы

На этом этапе нашего исследования мы должны быть в состоянии найти x и y пересечений и решить любое квадратное уравнение. Теперь мы изучим простой метод, используемый для их построения.

Плейлист графических парабол на YouTube

График квадратного уравнения называется параболой.

Одна из наших основных функций

можно изобразить, нанеся точки. Мы делаем это, выбирая около пяти x -значения и нахождение соответствующих им y -значений.

График :

 

Чем больше точек мы наносим, ​​тем легче увидеть, что график имеет U-образную форму. Вершина в данном случае является точкой изменения графика с убывающей на возрастающую, или точкой с наименьшим значением y . Здесь вершина (0, 0), которая также является точкой пересечения x и y . Линия х = 0, ось y является линией симметрии . Это линия, по которой мы могли бы согнуть нашу бумагу, чтобы увидеть, что две стороны графика совпадают.

По заданному графику найдите точки пересечения x и y , вершину, 5-ю точку на графике и линию симметрии.

Линия симметрии: x = 1

Точки пересечения по оси x: (-2,0) и (4,0)

Точки пересечения с y: (0, -8)

Вершина: (1, — 9)

5-я точка: (2, -8)

Напомним, что две точки определяют прямую — для парабол это не так. Параболы требуют минимум 3 точки, но обычно мы хотим найти по крайней мере пять точек, чтобы построить хороший график. Найдите вершину, x — и y — точки пересечения, а также линию симметрии.

График:

Шаг 1 : Найдите точку пересечения и (0, c ).

Шаг 2 : Найдите x — перехватывает, устанавливая y = 0 и вычисляя x .

Шаг 3 : Найдите вершину. Вы можете найти значение x вершины, используя вершину x = -b/(2a).

Шаг 4 : Нанесите точки и определите ось симметрии.

Область и диапазон вышеуказанной функции можно определить по графику. В предыдущей задаче домен состоит из всех действительных чисел, а диапазон состоит из всех действительных чисел, больших или равных -1. Также полезно отметить, что у нас минимум y -значение -1, это будет важным фактом при работе над текстовыми задачами.

Совет : осью симметрии любой квадратичной функции будет вертикальная линия

При попытке найти точки пересечения x , где результирующее квадратное уравнение не учитывается, просто используйте квадратную формулу для его решения.

График:

Эта парабола выглядит немного по-другому, обратите внимание, что она открывается вниз, а также обратите внимание, что предыдущая парабола открылась. Есть простой тест, чтобы узнать, как он открывается, еще до того, как мы начнем.

Поэтому, когда вас попросят нарисовать параболу, вы можете получить две важные части информации, не выполняя никакой работы. При осмотре вы можете сказать, открывается ли он вверх или вниз, и вы можете определить и -перехват.

Нанесите на график и пометьте все важные точки :

  

 

 

Нанесите на график и пометьте все важные точки :

0 70 0 0 0 7

0  

 

Областью предыдущей задачи являются все действительные числа, а диапазон состоит из всех действительных чисел, больших или равных -5. Также обратите внимание, что минимум г — значение равно -5. Оказывается, не все параболы имеют два пересечения x , как можно было бы ожидать. Иногда у них есть только один перехват x , а иногда и нет.

Пожалуйста, имейте в виду, что все квадратичные функции имеют вершину и точку пересечения и . Кроме того, мы все равно сможем найти другую точку, используя симметрию. Поэтому в некоторых случаях допустимо наносить и маркировать только три точки.

Отметьте и обозначьте все важные моменты: 

 

 

 

Нанесите на график и обозначьте все важные точки:

 

Снаряд брошен в начальный объект 91002 Снаряд Проблема: 9002 скорость 44 метра в секунду. Сколько времени потребуется, чтобы достичь максимальной высоты? Какая максимальная высота?


Примеры видео на YouTube :

Как построить параболу за 3 простых шага — Mashup Math

Полное руководство по построению параболы на координатной плоскости

Предварительный просмотр руководства: Как построить параболу.

В алгебре важно понимать поведение квадратичных функций и их графики. В этом кратком руководстве рассматриваются следующие ключевые понятия (включая примеры):

  • Как построить параболу? 92+bx+c (обратите внимание, что в этом руководстве мы будем использовать f(x)= и y= взаимозаменяемо). Построение графика параболы на координатной плоскости с использованием ее формулы является важным навыком алгебры, и это руководство научит вас простому 3-этапному процессу, который вы можете использовать для построения графика параболы практически в любом сценарии.

    Прежде чем вы научитесь строить параболы в стандартной форме, давайте рассмотрим некоторые ключевые понятия и словарный запас, связанные с квадратичными функциями и их графиками, прежде чем перейти к нескольким примерам построения графиков парабол. 92 -4x + 5, a=1, b=-4 и c=5

    График функции представляет собой параболу — U-образную плоскую кривую, симметричную относительно линии симметрии, проходящей через вершинная точка. Парабола симметрична и образует зеркальное отражение самой себя по разные стороны от линии симметрии, проходящей через точку вершины.

    На рисунке 01 ниже показаны примеры квадратичных функций в стандартной форме вместе с соответствующим графиком. Обратите внимание на U-образную параболу для каждого графика вместе с точкой вершины. Также обратите внимание, что парабола может открываться вверх (U-образная форма) или открываться вниз (перевернутая U-образная форма).

    Рисунок 01: Как построить параболу: примеры построения параболы на координатной плоскости

    Теперь, когда вы знаете, как выглядит график квадратичной функции в стандартной форме, давайте еще раз посмотрим, что такое вершина параболы.

    Что такое вершина параболы?

    Вершина параболы — это координатная точка (x,y), в которой пересекаются парабола и ее ось симметрии.

    В Рисунок 01 выше вершина параболы помечена, а осью симметрии является горизонтальная пунктирная линия, которая проходит через нее.

    Проще говоря, что является вершиной параболы? Вершина — это точка в самом низу кривой для парабол, открывающихся вверх, или в самом верху кривой для парабол, открывающихся вниз.

    Если вы можете найти вершину параболы, то вы можете легко научиться рисовать параболу на координатной плоскости, поэтому важно понять, что такое вершина параболы и как ее найти. К счастью, найти вершину параболы несложно, если выполнить два простых шага:2-4х+5 равно (2,1).

    График этой функции, включая точку вершины, показан на координатной плоскости Рисунок 03 .

    Теперь, когда вы знаете, как найти точку вершины параболы, вы готовы выполнить несколько пошаговых примеров того, как изобразить параболу на координатной плоскости, используя 3 простых шага (первый из которых определить, что является вершиной параболы).

    Как построить параболу: Пример №1

    92 + 6x + 5

    Теперь вы готовы начать рисовать параболу на координатной плоскости, используя простой трехэтапный метод:

    • Шаг 1: Найдите координаты точки вершины

    • Шаг 2: Создайте таблицу функций и «постройте» две точки на каждой стороне вершины

    • Шаг 3: Нанесите точки и постройте кривую (параболу)

    Теперь давайте продолжим и применим эти три шага к этому первому примеру: 92 + 6x + 5, мы будем использовать тот же процесс, что и в Рисунок 02 выше, следующим образом:

    Поскольку b=6 и a=1, x=-b/2a → x= -6 / 2(1) → x = -6/2 → x = -3

    • Шаг B: Введите значение координаты x из шага A в функцию, чтобы найти значение координаты y.

      Теперь, когда вы знаете, что значение координаты x для вершины равно x=-3, вы можете ввести x=-3 в функцию, чтобы найти значение координаты y следующим образом: 92 -6(-3) + 5 → f(2) = 4 — 8 + 5 = -4 y=-4

    Итак, координаты вершины парабола (-3,-4)

    Как построить параболу: используйте таблицу функций.

    Шаг 2: Создайте таблицу функций и «постройте» две точки на каждой стороне вершины

    Как только вы определите координаты точки вершины, вы можете построить таблицу функций, как показано на рис. 9.0191 Рисунок 04 . Ваша таблица функций должна включать столбец с координатами x и столбец с координатами y, а также строки не менее чем для пяти точек на графике.

    Поскольку вершина представляет собой «точку поворота» U-образной параболы, координаты вершины должны быть помещены в середину таблицы функций.

    Затем пришло время «построить» точки по обе стороны от точки вершины, введя значения x, которые меньше -3 и больше 3, в исходную функцию и выводят значения.

    Опять же, поскольку координата x точки вершины равна -3, мы выберем два значения x меньше -3 (-4 и -5 в этом примере) и два значения x больше — 3 (-2 и -1 в этом примере).

    Затем возьмите каждое значение x и введите его в функцию, чтобы найти соответствующее значение y. Например, чтобы найти значение координаты y при x=-5, просто оцените значение f(-5), как показано на Рисунок 04 ниже.

    92+6(-5)+5 = 0, вы можете заключить, что когда x равно -5, y равно 0 (и что (-5,0) является точкой на параболе. Теперь повторите этот процесс чтобы найти f(-4), f(-2) и f(-1), чтобы заполнить таблицу функций, как показано на Рисунок 05 ниже.

    Рисунок 05: Заполненная таблица функций

    Шаг 3: Нанесите точки и постройте кривую (параболу)

    Третий и последний шаг — нанесение всех пяти точек таблицы функций на координатную плоскость. Вы должны заметить, что точки следуют симметричной U-образной форме с точкой поворота вокруг точки вершины, которая в этом примере (-3,-4).

    Обратите внимание, что точки в заполненной таблице функций: (-5,0), (-4,-3), (-3,-4), (-2,-3) и (-1,0) . После нанесения этих пяти точек на координатную плоскость у вас должен получиться график, похожий на Рисунок 06 ниже.

    Рисунок 06: Как построить параболу: начертите все точки из таблицы функций.

    После того, как ваши точки были нанесены, вы готовы построить свою параболу, нарисовав U-образную кривую, которая проходит через все пять точек, как показано на рисунке 9.2 -2(1) -6 → f(1) = 1- 2 -6 = -7 y=-7

Итак, координаты вершины параболы равны (1,-7)

Шаг 2: Создайте таблицу функций и «постройте» две точки с каждой стороны вершины

Теперь, когда вы знаете координаты точки вершины (1,-7), вы можно начать построение точек по обе стороны от вершины, вводя значения x, которые меньше 1 (0 и -1 в этом примере) и больше 1 (2 и 3 в этом примере) в исходную функцию и выводят значения координаты y. 92-2(0)-6 = 0-0-6 = -6

Итак, вы можете заключить, что, когда x равно 0, y равно -6 (и что (-6,0) является точкой на параболе На рисунке 08 ниже показано, как вычислить f(0), а также все другие точки в таблице данных.

Рисунок 08: Заполненная таблица данных

Шаг 3: Нанесите точки и постройте кривую (параболу)

После заполнения таблицы данных вы готовы нанести на координатную плоскость все пять следующих точек: (-1,-3), (0,-6), (1,-7), (2,-6) и (3,-3). 92 +8(4) -12 → f(4) = -16 + 32 — 12 → y=4

Итак, координаты вершины параболы равны (4,4)

Шаг 2: Создайте таблицу функций и «постройте» две точки на каждой стороне вершины

Затем используйте точку вершины (4,4) для построения точек на каждой стороне вершины путем ввода значений x, которые оба меньше 4 (3 и 2 в этом примере) и больше 4 (5 и 6 в этом примере) в исходную функцию и выводят значения координаты y. 92+8(5)-12 = -25 + 40 — 12 = 3

Итак, вы можете заключить, что когда x равно 5, y равно 3 (и что (5,3) является точкой на параболе. На рисунке 11 ниже показано, как вычислить f(5), а также все другие точки в таблице данных.

Рисунок 11: Завершенная функциональная схема

Шаг 3: Нанесите точки и постройте кривую (параболу)

Теперь вы готовы нанести точки (2,0), (3,3), (4,4), (5,3), и (6,0) на координатной плоскости, как показано на Рисунок 12 внизу:

Рисунок 12

Обратите внимание, что парабола в примере №3 раскрывается вниз (в то время как параболы в примерах 1 и 2 раскрываются вверх). Это совершенно нормально, так как параболы могут раскрываться вверх или вниз.

Последним шагом будет начертить кривую, проходящую через точки, чтобы завершить построение параболы, как показано на .

Таблица производственных возможностей: ТАБЛИЦА ПРОИЗВОДСТВЕННЫХ ВОЗМОЖНОСТЕЙ | InvestFuture

2.2. Таблица производственных возможностей

Сущность проблемы экономии можно проиллюстри­ровать даже еще более четко, используя для этого таблицу производственных возможностей. Этот ме­тод выявляет самую суть проблемы экономии: по­скольку ресурсы редки, экономика полной занятости, полного объема производства не может обеспечить неограниченный выпуск товаров и услуг. Более того, необходимо принимать решения о том, какие товары и услуги следует производить, а от каких отказаться.

Допущения. Чтобы наилучшим образом проил­люстрировать проблему экономии, мы принимаем несколько определенных допущений.

1.ЭФФЕКТИВНОСТЬ. Экономика функциони­рует в условиях полной занятости и достигает пол­ного объема производства.

2.ПОСТОЯННОЕ КОЛИЧЕСТВО РЕСУРСОВ. Имеющиеся факторы производства постоянны как по количеству, так и по качеству. Но, разумеется, в определенных пределах может изменяться соот­ношение их использования на различные цели, то есть их можно перераспределять; например, относи­тельно неквалифицированный работник может ра­ботать на ферме, в ресторане быстрого обслужива­ния или на автозаправочной станции.

3.НЕИЗМЕННАЯ ТЕХНОЛОГИЯ. Технология производства принимается постоянной, то есть в хо­де нашего анализа она не изменяется. Второе и тре­тье допущение подразумевают, что мы рассматри­ваем нашу экономику по ее состоянию на опреде­ленный момент времени или на протяжении очень короткого периода. Для относительно долгого пе­риода было бы нереалистично исключать техничес­кий прогресс и возможность изменения состава име­ющихся ресурсов.

4.ДВА ПРОДУКТА. Для еще большего упроще­ния сделаем допущение, что наша экономика произ­водит не бесчисленное множество товаров и услуг, как это имеет место в действительности, а лишь два продукта —промышленные роботы и пиццу. Пицца символизирует потребительские товары, то есть те товары, которые непосредственно удовле­творяют наши потребности. Промышленные робо­ты символизируют средства производст­ва, товары производственного на­значения, то есть те товары, которые удовлетво­ряют наши потребностикосвенно,обеспечивая более эффективное производство потребительских това­ров.

Необходимость выбора. А теперь, не становится ли очевидным из принятых нами допущений, что наша экономика сталкивается с необходимостью выбора между альтернативами? Общий объем име­ющихся ресурсов ограничен. Следовательно, огра­ниченна и способность нашей экономики произ­водить промышленные роботы и пиццу.Ограничен­ность ресурсов означает ограниченность выпуска. Поскольку ресурсы ограниченны и применяются целиком, всякое увеличение производства промыш­ленных роботов потребует переключения части ре­сурсов с производства пиццы. Правилен также и противоположный вывод: если мы предпочтем увеличить производство пиццы, необходимые для этого ресурсы должны быть получены лишь за счет сокращения производства роботов.Общество не может преследовать две взаимоисключающие цели. Образно говоря, «бесплатных ланчей не бывает». В этом суть проблемы экономии.

Давайте порассуждаем на примере приведенных в таблице 1альтернативных комбинаций коли­честв промышленных роботов и пиццы, из которых наше общество может выбирать. Несмотря на то, что данные в этой и последующих таблицах гипотетические, иллюстрируемые ими положения имеют огромное практическое значение. Согласно альтернативе А, наша экономика направила бы все свои ресурсы на производство роботов, то есть товаров производственного назначения. А при альтернативе Е все наличные ресурсы были бы употреблены на производство пиццы, то есть предметов потребления. Обе эти альтернативы представляют собой явно нереалистичные крайно­сти; всякая экономика обычно находит баланс в распределении общего объема своего производст­ва между товарами производственного назначения и потребительскими товарами. По мере продвиже­ния от альтернативы А до Е мы увеличиваем производство предметов потребления (пиццы). Как? Переключением ресурсов с производства средств производства. Поскольку мы знаем, что потреби­тельские товары непосредственно удовлетворяют наши потребности, любое продвижение в направле­нии альтернативы Е представляется нам заман­чивым. Двигаясь в этом направлении, общество увеличивает удовлетворение своих текущих потреб­ностей. Однако такая политика дорого обходится. Подобное переключение ресурсов со временем нанесет удар самому обществу, поскольку запас его средств производства сокращается или, по крайней мере, перестает увеличиваться обычным темпом, а в результате потенциал будущего производства снижается. Короче говоря, продвигаясь от альтер­нативы А к Е, общество фактически делает выбор в пользу политики «больше сейчас» за счет полити­ки «намного больше потом». Напротив, продвига­ясь от альтернативы Е к А, общество выбирает политику воздержания от текущего потребления. Такое жертвование текущим потреблением высво­бождает ресурсы, которые могут быть исполь­зованы для увеличения производства средств произ­водства. Наращивая таким путем запас своего капитала, общество может рассчитывать на боль­ший объем производства, а поэтому и на большее потребление в будущем.

Таблица 1

Возможности производства пиццы в промышленных роботов при полной занятости ресурсов, 1990(гипотетические данные)

Вид

Производственные альтернативы

А

В

С

D

Е

Пицца (сотни тыс. )

0

1

2

3

4

Роботы (тыс.)

10

9

7

4

0

Главная идея здесь сводится к следующему: в любой момент времени экономика полной занято­сти и полного объема производства должна жертво­вать частью продукта X, чтобы получить больше продукта Y.Тот решающий факт, что экономичес­кие ресурсы редки, не позволяет такой экономике увеличивать одновременно и Х и Y.

КРИВАЯ ПРОИЗВОДСТВЕННЫХ ВОЗМОЖНОСТЕЙ • Большая российская энциклопедия

Авторы: Р. М. Нуреев

Рис. 1. Кривая производственных возможностей.

Рис. 2. Сдвиг кривой производственных возможностей при одностороннем расширении одного из видов производства.

КРИВА́Я ПРОИЗВО́ДСТВЕННЫХ ВОЗМО́ЖНОСТЕЙ, по­ка­зы­ва­ет аль­тер­на­тив­ные ва­ри­ан­ты при пол­ном ис­поль­зо­ва­нии ре­сур­сов. Про­из­водств. воз­мож­но­сти – воз­мож­но­сти об­ще­ст­ва по про­из-ву эко­но­мич. благ при пол­ном и эф­фек­тив­ном ис­поль­зо­ва­нии всех имею­щих­ся ре­сур­сов при дан­ном уров­не раз­ви­тия тех­но­ло­гии. Имен­но по­тен­ци­аль­ный (воз­мож­ный) вы­пуск про­дук­ции ха­рак­те­ри­зу­ет К. п. в. Напр., до­пус­тим, что в об­ще­ст­ве про­из­во­дят­ся лишь два бла­га: зер­но и ра­ке­ты. Ес­ли об­ще­ст­во ис­поль­зу­ет свои ре­сур­сы для про­из-ва толь­ко зер­на, то оно про­из­во­дит его 5 млн. т; ес­ли же толь­ко для про­из-ва ра­кет, то их про­из­во­дит­ся 6 шт. При од­но­вре­мен­ном про­из-ве обо­их благ воз­мож­ны сле­дую­щие со­че­та­ния (рис. 1). Из таб­ли­цы вид­но, что вся­кое уве­ли­че­ние про­из-ва ра­кет (с 0 до 6 шт.) сни­жа­ет про­из-во зер­на (с 5 млн. т до 0 т), и на­обо­рот. Ли­ния $ABCDEFG$ яв­ля­ет­ся К. п. в. Все точ­ки, рас­по­ло­жен­ные внут­ри фи­гу­ры $OAG$, оз­на­ча­ют не­пол­ное ис­поль­зо­ва­ние ре­сур­сов, напр. точ­ка $K$ (од­но­вре­мен­ное про­из-во 2,5 млн. т зер­на и трёх ра­кет). И на­обо­рот, лю­бая про­из­водств. про­грам­ма, ха­рак­те­ри­зуе­мая точ­ка­ми за пре­де­ла­ми фи­гу­ры $OAG$, не бу­дет обес­пе­че­на на­лич­ны­ми ре­сур­са­ми (напр., точ­ка $H$). К. п. в. обыч­но име­ет вы­пук­лую фор­му (во­гну­та к на­ча­лу ко­ор­ди­нат). Это оз­на­ча­ет, что, из­ме­няя струк­ту­ру про­из-ва, напр., в поль­зу ра­кет, в боль­шей ме­ре бу­дут ис­поль­зо­вать­ся в про­из-ве ра­кет срав­ни­тель­но ма­ло­эф­фек­тив­ные для это­го ре­сур­сы. По­это­му ка­ж­дая до­пол­нит. ра­ке­та тре­бу­ет всё боль­ше­го со­кра­ще­ния про­из-ва зер­на (и на­обо­рот). Про­из-во пер­вой ра­ке­ты вы­зва­ло со­кра­ще­ние про­из-ва зер­на на 0,2 млн. т, вто­рой – на 0,3 млн., треть­ей – на 0,6 млн. т и т. д. Этот пример на­гляд­но ил­лю­ст­ри­ру­ет убы­ваю­щей пре­дель­ной про­из­во­ди­тель­но­сти за­кон. К. п. в. ис­то­рич­на, она от­ра­жа­ет дос­тигну­тый уро­вень раз­ви­тия тех­но­ло­гии и сте­пень ис­поль­зо­ва­ния имею­щих­ся ре­сур­сов. Ес­ли уве­ли­чи­ва­ют­ся ре­сур­сы или улуч­ша­ет­ся тех­но­ло­гия, пло­щадь фи­гуры $OAG$ рас­тёт, кри­вая $ABCDEFG$ сдви­га­ет­ся вверх и впра­во.

Таблица производственных возможностей
ВозможностиЗерно, млн. тРакеты, шт.
A5,00
B4,81
C4,52
D3,93
E3,04
F1,85
G06

Ес­ли про­цесс про­ис­хо­дит рав­но­мер­но, то кри­вая $AG$ сим­мет­рич­но сме­ща­ет­ся до по­ло­же­ния $А’G’$ (рис. 1). Ес­ли про­ис­хо­дит од­но­сто­рон­нее уве­ли­че­ние эф­фек­тив­но­сти тех­но­ло­гии про­из-ва од­но­го из благ, то сдвиг но­сит асим­мет­рич­ный ха­рак­тер (рис. 2). При од­но­сто­рон­нем рас­ши­ре­нии про­из-ва зер­на кри­вая $AG$ сме­ща­ет­ся до по­ло­же­ния $A_1G$, при уве­ли­че­нии про­из-ва ра­кет – до по­ло­же­ния $AG_1$. К. п. в. мо­жет быть ис­поль­зо­ва­на для ха­рак­те­ри­сти­ки струк­тур­ных сдви­гов ме­ж­ду пром-стью и с. х-вом, об­ществ. и ча­ст­ны­ми бла­га­ми, те­ку­щим и бу­ду­щим по­треб­ле­ни­ем (по­тре­би­тель­ски­ми и ин­ве­сти­ци­он­ны­ми то­ва­ра­ми и ус­лу­га­ми) и т. д.

В ус­ло­ви­ях ог­ра­ни­чен­но­сти ре­сур­сов про­бле­ма эко­но­мич. вы­бо­ра не­уст­ра­ни­ма, од­на­ко в разл. эко­но­мич. сис­те­мах она ре­ша­ет­ся по-раз­но­му. В тра­диц. об­ще­ст­ве вы­бор за­ви­сит от тра­ди­ций и обы­ча­ев, в ко­манд­ной эко­но­ми­ке – от во­ли пра­вя­щей эли­ты, в ры­ноч­ном хо­зяй­ст­ве – от ры­ноч­ной конъ­юнк­ту­ры.

EconPort — Таблица производственных возможностей

Справочник >> Производственные возможности >>

В таблице производственных возможностей перечислены альтернативные издержки выбора путем суммирования того, какие альтернативные результаты вы можете получить с помощью ваших ресурсов. Ниже перечислены две таблицы. В первой таблице вход — это просто процент ресурсов, которые страна выделяет на производство, а выход — «рис» или «железная руда».

% ресурсов, предназначенных для производства риса тонн риса % ресурсов, предназначенных для производства железной руды тонн железной руды
 100% 90 005  100  0%  0
80% 80 20% 5
 60%  60  40%  10
 40%  40  60%  15
 20%  20  80%  20
0% 0 100% 25

В нашей вымышленной стране есть множество вариантов. Он мог решить производить только рис и накопить 100 тонн. Или он может решить посвятить только 20% своего производства рису и производить 20 тонн, выделяя 80% своего производства на железную руду и производя 20 тонн. Какую бы комбинацию ни выбрала страна по одному вопросу, она постоянна: если она хочет производить больше риса, она должна производить меньше железной руды, а если она хочет производить больше железной руды, она должна производить меньше риса. Это отражает альтернативные издержки, связанные с производством. Или, более конкретно, если страна хочет получить еще 20 тонн риса, она должна отказаться от 5 тонн железной руды. Эти 5 тонн железной руды представляют собой альтернативные издержки производства 20 тонн риса. Эта альтернативная стоимость постоянна во всей таблице.

Во второй таблице, приведенной ниже, входными данными снова является просто процент ресурсов, которые страна выделяет на производство, а выходными данными являются либо «оружие», либо «масло».
                                                                                                                                                                                                                                              

% ресурсов, выделенных на производство оружия Количество ружей % ресурсов, выделенных на производство масла Фунты масла
0 0 100 19
20 6 80 18
40 10 60 16
60 13 40 12
80 16 20 7
100 17 0 0

Обратите внимание, что в нашей вымышленной стране снова есть множество вариантов, касающихся оружия и масла. Он может решить ценить мир превыше всего и полностью сосредоточиться на производстве масла, производя 19 фунтов масла за свои усилия. Или он может занять противоположную позицию и направить 80% своих ресурсов на наращивание орудий, дав ему 16 орудий, но оставив только 20% своих ресурсов для производства масла и производя только 7 фунтов масла. В качестве альтернативы он может принять более умеренное решение и направить 40% своих ресурсов на производство ружей и 60% своих ресурсов на производство масла, в результате чего получится 10 ружей и 16 фунтов масла. Какую бы комбинацию ни выбрала страна, неизменно одно: если страна решает производить больше ружей, она должна производить меньше масла, а если страна хочет производить больше масла, она должна производить меньше ружей. Это отражает альтернативные издержки, связанные с производством.

Тем не менее, есть кое-что в отношениях между «Ружьями и маслом», которые отличаются от отношений между «Железной рудой и рисом». В то время как отношение между «железной рудой и рисом» имело постоянные альтернативные издержки, связанные с ним (нашей стране всегда приходилось отказываться от 20 тонн риса, чтобы произвести 5 дополнительных тонн железной руды), альтернативные издержки, связанные с «оружием и маслом», меняются. в зависимости от того, сколько каждого наша страна решит производить.

Присмотритесь. Если наша страна выделяет 100% своих ресурсов на производство масла и решает, что хочет произвести 6 ружей, ей нужно отказаться только от 1 фунта масла (альтернативные издержки производства 6 дополнительных ружей). Предположим, наша страна решает, что ей нужно больше оружия. Чтобы произвести 10 ружей (на 4 больше, чем производилось изначально), он должен отказаться от 2 фунтов масла (производя 16 фунтов масла, а не 18 фунтов). Таким образом, альтернативная стоимость производства 4 дополнительных ружей составляет 2 фунта масла. Теперь давайте предположим, что наша страна выделяет 80% своих ресурсов на производство ружей и производит 16 ружей, но решает, что ей нужно больше. Чтобы произвести 1 дополнительное ружье, оно должно отказаться от 7 фунтов масла.

Как видите, по мере того, как наша страна увеличивает производство оружия, она получает все меньше и меньше оружия на каждый отданный фунт масла. Это означает, что альтернативная стоимость выбора оружия вместо масла увеличивается по мере того, как мы увеличиваем производство оружия. Эта концепция называется принципом возрастания предельной альтернативной стоимости. Это происходит потому, что некоторые ресурсы лучше подходят для производства масла, чем для производства ружей, и мы используем в первую очередь более эффективные ресурсы.

Вернуться к возможностям производства Модель

Copyright 2006 Experimental Economics Центр. Все права защищены. Отправить нам обратная связь

Таблица производственных возможностей – UNISA

После того, как вы проработаете этот раздел модуля обучения, вы сможете:

  • интерпретировать данные в таблице производственных возможностей

Мы составим таблицу производственных возможностей для гипотетической страны Занаду. Два продукта, которые производит эта страна, полностью и эффективно используя свои ресурсы, — это ноутбуки и мобильные телефоны.

Различные комбинации ноутбуков и мобильных телефонов при полном и эффективном использовании ресурсов, которые могут быть произведены, указаны в следующей таблице производственных возможностей:

В этом разделе мы создадим таблицу производственных возможностей для гипотетической страны под названием Занаду. Два продукта, которые производит эта страна, полностью и эффективно используя свои ресурсы, — это ноутбуки и мобильные телефоны.

Различные комбинации ноутбуков и мобильных телефонов с полным и эффективным использованием ресурсов, которые могут быть произведены, указаны в следующей таблице производственных возможностей:

Таблица 1 Возможности производства Zanadu

Комбинация Ноутбуки Мобильные телефоны
А 5 000         0
Б 4 000 10 000
С 3 000 18 000
Д 2 000 24 000
Е 1 000 28 000
Ф       0 30 000

Давайте обратим внимание на таблицу, чтобы увидеть, что именно говорят нам данные.

Изучите приведенную выше таблицу и ответьте на следующие вопросы:

Сколько ноутбуков будет произведено, если страна использует все свои ресурсы для производства ноутбуков?

0 5000 4000 3000 2000 г.

Это 5 000, как показано комбинацией A.

Подумайте еще раз.

Обратите внимание, что если будет произведено 10 000 мобильных телефонов, Zanadu сможет произвести только 4 000 ноутбуков, а не 5 000 ноутбуков.

Комбинация C указывает на то, что Zanadu может производить 3 000 ноутбуков и 18 000 мобильных телефонов, а комбинация D указывает на возможность производства 2 000 ноутбуков и 24 000 мобильных телефонов. Комбинация E указывает на возможность производства 1 000 ноутбуков и 28 000 мобильных телефонов.

Сколько мобильных телефонов будет произведено, если страна направит все свои ресурсы на производство ноутбуков?

0 10 000 24 000 30 000

Это ноль. Если страна использует все свои ресурсы для производства ноутбуков, то для производства мобильных телефонов не будет ресурсов, а производство мобильных телефонов действительно должно быть нулевым. На это указывает комбинация A.

Подумайте еще раз.

С точки зрения комбинации F в приведенной выше таблице, сколько мобильных телефонов можно было бы произвести, если бы все ресурсы использовались для производства мобильных телефонов, и сколько ноутбуков?

Ноутбуки

0 1 000 2 000 3 000 4 000 5 000

Мобильные телефоны

0 10 000 18 000 24 000 28 000 30 000

Комбинация F говорит нам, что если все ресурсы использовать для производства мобильных телефонов, то будет произведено 30 000 мобильных телефонов и ноль ноутбуков.

Подумайте еще раз.

С точки зрения комбинации B в приведенной выше таблице, сколько ноутбуков может быть произведено и сколько мобильных телефонов?

Ноутбуки

0 1 000 2 000 3 000 4 000 5 000

Мобильные телефоны

0 10 000 18 000 24 000 28 000 30 000

Если вы посмотрите на такую ​​комбинацию, как B, она говорит вам, что если Zanadu направит часть своих ресурсов на производство ноутбуков, а часть — на производство мобильных телефонов и эффективно использует эти ресурсы, то она сможет производить 4 000 ноутбуков и 10 000 мобильных телефонов.

Подумайте еще раз.

Обратите внимание, что если будет произведено 10 000 мобильных телефонов, Zanadu сможет произвести только 4 000 ноутбуков, а не 5 000 ноутбуков.

Комбинация C указывает на то, что Zanadu может производить 3 000 ноутбуков и 18 000 мобильных телефонов, а комбинация D указывает на возможность производства 2 000 ноутбуков и 24 000 мобильных телефонов. Комбинация E указывает на возможность производства 1 000 ноутбуков и 28 000 мобильных телефонов.

Посмотрите следующий видеоклип, чтобы ознакомиться с таблицей возможностей.

Деятельность

Выполните следующее задание, чтобы убедиться, что вы понимаете таблицу производственных возможностей, потому что мы будем использовать данные в таблице производственных возможностей для построения кривой или границы производственных возможностей:

  • Если Paradiso эффективно использует все свои ресурсы для производства продуктов питания, то сможет произвести 1000 тонн продуктов питания.
  • Если Paradiso эффективно использует все свои ресурсы, она сможет производить 900 тонн еды и миллион орудий.
  • Если Paradiso эффективно использует все свои ресурсы, то сможет производить 750 тонн еды и два миллиона орудий.
  • Если Paradiso эффективно использует все свои ресурсы, то сможет производить 550 тонн еды и три миллиона единиц оружия.
  • Если Paradiso эффективно использует все свои ресурсы, она сможет производить 300 тонн еды и четыре миллиона единиц оружия.
  • Если Paradiso эффективно использует все свои ресурсы для производства оружия, то сможет произвести пять миллионов единиц оружия.

Комбинация Продукты питания (тонны) Ружья (млн.)
А 0
Б
С
Д
Е
Ф 0

Нажмите, чтобы посмотреть ответ

Комбинация Продукты питания (тонны) Ружья (млн.

А логистика задачи: Логистика: что это такие, основные принципы логистики

Логистика: что это такие, основные принципы логистики

Логистика — это наука, которая изучает процессы снабжения, поставок и перевозок. Она помогает прогнозировать, контролировать и оптимизировать передачу товаров, услуг или информации между производителями, поставщиками и потребителями. Данная сфера охватывает закупку, перевозку, продажу и хранение грузов, а также управление сопутствующими потоками финансов и информации.

Логистика находит оптимальные решения в процессе доставки товаров от поставщика к потребителю

Задачи и функции логистики

Основная задача логистики — оптимизация издержек в процессе хранения, транспортировки и сбыта. Наука учит оптимальному распределять нагрузку и ресурсы. Хорошая логистика помогает увеличить общую прибыль предприятия за счет сокращения расходов на транспортировку и склад. 

Некоторые функции логистики: 

  • выбор транспортного средства для перевозки грузов; 
  • определение оптимального маршрута; 
  • распределение товаров для безопасной транспортировки; 
  • таможенное оформление получаемых из-за границы товаров; 
  • нанесение необходимых маркировок и оформление документов на продукцию; 
  • перевозка и доставка грузов компании; 
  • грамотное размещение товаров на складских площадях; 
  • управление товарными запасами.

Принципы логистики

Выделяют шесть главных принципов логистики : 

  1. Груз — отгрузить нужный товар.
  2. Качество — необходимого качества. 
  3. Количество — в нужном количестве.
  4. Время — вовремя, точно в срок.
  5. Место — в нужное место.
  6. Затраты — с минимальными затратами.

Грамотная организация логистических процессов улучшает экономику предприятия, помогает оптимизировать бизнес-процессы и снизить затраты на хранение и доставку продукции. Важно следовать принципам на каждом этапе производства: от закупки материалов до доставки грузов. 

Логистические концепции

Чтобы организовать логистические процессы на производстве, компании расставляют приоритеты. Они определяют, какие факторы будут учитывать в первую очередь, исходя из чего будут планировать работу предприятия. 

Принципы планирования складываются в логистические концепции. Компании определяют, на основе какой концепции они будут работать, и следуют ей.

Выделяют несколько базовых концепций логистики: 

Just In Time (точно в срок). Самая популярная концепция. Она появилась в 50-х годах, когда японская компания «Тойота» начала внедрять систему управления проектами Kanban. 

В основе концепции лежит идея планирования всех поставок к определенному времени. Если вы знаете точные сроки отгрузки, то организуете процесс производства и сборки изделий согласно расписанию. У вас нет большого запаса готовой продукции, вы не ждете материалов от поставщиков. Все уже готово к нужной дате. 

Requirements / resource planning (планирование производства). На основе данной концепции работают следующие логистические системы в производстве и снабжении:

  • MRP I / MRP II — Materials / manufacturing requirements resource planning. Система планирования потребностей в материалах и ресурсах на производстве.
  • DRP I / DRP II — Distribution requirements resource planning. Система планирования распределения продукции / ресурсов в дистрибьюции.

Самое главное в этих системах — ответить на вопрос, сколько и в какие сроки необходимо произвести продукции. Далее определяют количество нужных ресурсов. При этом MRP ориентируется на потребности производства, а вот DRP — на потребительский спрос. 

Lean production (тощее производство). Базируется на идее сокращения запасов и снижении объемов партий товаров. Высокое качество продукции обеспечивается за счет квалифицированного персонала и оборудования. Производственные потери и риски сводятся к минимуму.

Quick response (быстрый ответ). Предполагает быструю координацию между ритейлерами и оптовиками, что позволяет мгновенно реагировать на изменение спроса. Продавцы мониторят продажи и передают данные об объемах оптовикам и производителям. Запасы готового товара практически отсутствуют, компания удовлетворяет текущий спрос. 

Виды логистики

Поскольку логистику применяют в самых разных сферах деятельности, то выделяют несколько её видов. И хотя общие подходы и принципы схожи, каждое направление обладает своими особенностями. 

Транспортная логистика

Главная задача транспортной логистики — доставка грузов из одной точки в другую с минимальными затратами и без вреда для товаров. Конечная цель — найти самый оптимальный маршрут из всех возможных.  

Охватывает следующие функции: 

  • изучение груза и определение необходимых условий для его доставки; 
  • подбор транспорта или нескольких видов транспорта; 
  • построение маршрута с учётом кратчайшей доставки; 
  • организация и обеспечение качественной погрузки и разгрузки; 
  • контроль всех этапов транспортировки до передачи груза получателю. 

Закупочная логистика

Отвечает за снабжение бизнеса или производства необходимыми материалами в заданные сроки и с наименьшими затратами. 

В задачи входит:  

  • определение требуемых объёмов необходимых материалов; 
  • поиск поставщиков с наиболее выгодными условиями сотрудничества; 
  • планирование графика поставок;
  • организация закупки материалов; 
  • контроль приёма и проверки качества поставляемого товара.  

Информационная логистика

В задачи информационной логистики входит развитие систем передачи информации между субъектами. Для этого необходимо организовать своевременный и бесперебойный обмен данными между всеми отделами: снабжения, производства, хранения, транспортировки, распределения. Чаще всего для этого используют дополнительные программы, например, «1С». 

Функции: 

  • сбор данных об особенностях поставки товаров и услуг; 
  • тщательный анализ информации, которой обмениваются участники процесса;
  • распределение данных по ответственным отделам; 
  • обработка, хранение и архивация документов. 

Целью выступает объединение всех информационных потоков в единое целое. 

Производственная логистика

Оптимизирует перемещения материалов и готовых товаров внутри производственного цикла. Необходимо следить за своевременной доставкой сырья, соответствием материалов установленным критериям, отгрузкой и транспортировкой в запланированные сроки.  

К реализуемым функциям относят следующие: 

  • организация производственных операций с учётом рыночного спроса и текущих заказов;
  • планирование всех этапов производства продукции; 
  • отслеживание качественного завершения каждого отдельного этапа; 
  • постановка сроков и контроль над их соблюдением. 

Логистическая цель достигнута, если упорядочена работа всего производства и каждого отдельного цеха. 

Распределительная логистика

Основная цель распределительной, или маркетинговой логистики — распространить товары между поставщиками и продать их конечным потребителям. Важно правильно выбрать объемы поставок и сократить время транспортировки, особенно для скоропортящейся продукции. 

Функции: 

  • определение каналов распределения; 
  • выбор подходящих посредников; 
  • рационализация доставки товара до потребителя.  

Складская логистика

Складская логистика отвечает за хранение готовой продукции, грамотное использование площадей, создание резерва товаров в нужном объеме.

Охватывает следующие функции: 

  • учёт хранимых позиций товаров; 
  • организация резервного запаса; 
  • комплектация заказов на доставку;
  • выстраивание процессов приёмки и отгрузки. 

Таможенная логистика

Главная цель — соблюдение правовых норм при импорте и экспорте продукции, доставка и перевозка грузов за границу с минимальными рисками. 

В задачи таможенной логистики входит:

  • подготовка необходимых деклараций; 
  • получение сертификатов соответствия; 
  • страхование грузов; 
  • организация ответственного хранения; 
  • заключение договоров купли-продажи, в том числе по валютным сделкам; 
  • послетаможенное сопровождение.  

Цифровизация логистики

Цифровизация — это глобальный тренд логистики. Она упрощает и ускоряет процессы, что положительно отражается на качестве выполнения поставленных задач. 

С помощью различных программ и сервисов можно автоматизировать многие логистические процессы. Например: 

  • создание и изменение заявок; 
  • документооборот между участниками сделок; 
  • подбор оптимальных маршрутов; 
  • расчёт рентабельности грузоперевозок; 
  • аналитику операционной деятельности компании;  
  • управление взаиморасчётами с контрагентами; 
  • прогнозирование техобслуживания транспорта с учётом данных об эксплуатации; 
  • отслеживание передвижений транспорта. 

Яркий пример автоматизации логистической деятельности — маркетплейсы. Большинство процессов, связанных с приемом товаров и отгрузкой заказов, автоматизированы. Это помогает контролировать поставки в режиме реального времени и максимально быстро доставлять товары покупателю.

Доставка грузов на склад Яндекс.Маркета осуществляется с помощью системы

Активно применяют цифровизацию в складской логистике. Программы (их еще называют WMS-системами) помогают определить наиболее грамотное размещение товаров с учётом особенностей хранения, вести учёт в реальном времени, управлять удалёнными складами, оформлять нужные документы.

Программное обеспечение помогает автоматизировать работу склада

В транспортной логистике цифровые технологии (TMS-системы) помогают быстро и максимально точно составлять маршруты, планировать цепочки поставок, контролировать грузы при транспортировке и учитывать расходы ГСМ.

Пример онлайн-сервиса по автоматизации транспортных перевозок

Бизнес, который внедряет цифровые технологии в логистике, имеет больше шансов стать лидером рынка. Если для компании это слишком сложно и дорого, то можно частично передать логистику на аутсорс. Например, делегировать транспортировку, обработку или хранение грузов. Такой подход помогает решать логистические задачи максимально эффективно и профессионально.  

Главные мысли

Логистика: что это такие, основные принципы логистики

Логистика — это наука, которая изучает процессы снабжения, поставок и перевозок. Она помогает прогнозировать, контролировать и оптимизировать передачу товаров, услуг или информации между производителями, поставщиками и потребителями. Данная сфера охватывает закупку, перевозку, продажу и хранение грузов, а также управление сопутствующими потоками финансов и информации.

Логистика находит оптимальные решения в процессе доставки товаров от поставщика к потребителю

Задачи и функции логистики

Основная задача логистики — оптимизация издержек в процессе хранения, транспортировки и сбыта. Наука учит оптимальному распределять нагрузку и ресурсы. Хорошая логистика помогает увеличить общую прибыль предприятия за счет сокращения расходов на транспортировку и склад.  

Некоторые функции логистики: 

  • выбор транспортного средства для перевозки грузов; 
  • определение оптимального маршрута; 
  • распределение товаров для безопасной транспортировки; 
  • таможенное оформление получаемых из-за границы товаров; 
  • нанесение необходимых маркировок и оформление документов на продукцию; 
  • перевозка и доставка грузов компании; 
  • грамотное размещение товаров на складских площадях; 
  • управление товарными запасами.

Принципы логистики

Выделяют шесть главных принципов логистики : 

  1. Груз — отгрузить нужный товар.
  2. Качество — необходимого качества. 
  3. Количество — в нужном количестве.
  4. Время — вовремя, точно в срок.
  5. Место — в нужное место.
  6. Затраты — с минимальными затратами.

Грамотная организация логистических процессов улучшает экономику предприятия, помогает оптимизировать бизнес-процессы и снизить затраты на хранение и доставку продукции. Важно следовать принципам на каждом этапе производства: от закупки материалов до доставки грузов. 

Логистические концепции

Чтобы организовать логистические процессы на производстве, компании расставляют приоритеты. Они определяют, какие факторы будут учитывать в первую очередь, исходя из чего будут планировать работу предприятия. 

Принципы планирования складываются в логистические концепции. Компании определяют, на основе какой концепции они будут работать, и следуют ей.

Выделяют несколько базовых концепций логистики: 

Just In Time (точно в срок). Самая популярная концепция. Она появилась в 50-х годах, когда японская компания «Тойота» начала внедрять систему управления проектами Kanban. 

В основе концепции лежит идея планирования всех поставок к определенному времени. Если вы знаете точные сроки отгрузки, то организуете процесс производства и сборки изделий согласно расписанию. У вас нет большого запаса готовой продукции, вы не ждете материалов от поставщиков. Все уже готово к нужной дате. 

Requirements / resource planning (планирование производства). На основе данной концепции работают следующие логистические системы в производстве и снабжении:

  • MRP I / MRP II — Materials / manufacturing requirements resource planning. Система планирования потребностей в материалах и ресурсах на производстве.
  • DRP I / DRP II — Distribution requirements resource planning. Система планирования распределения продукции / ресурсов в дистрибьюции.

Самое главное в этих системах — ответить на вопрос, сколько и в какие сроки необходимо произвести продукции. Далее определяют количество нужных ресурсов. При этом MRP ориентируется на потребности производства, а вот DRP — на потребительский спрос. 

Lean production (тощее производство). Базируется на идее сокращения запасов и снижении объемов партий товаров. Высокое качество продукции обеспечивается за счет квалифицированного персонала и оборудования. Производственные потери и риски сводятся к минимуму.

Quick response (быстрый ответ). Предполагает быструю координацию между ритейлерами и оптовиками, что позволяет мгновенно реагировать на изменение спроса. Продавцы мониторят продажи и передают данные об объемах оптовикам и производителям. Запасы готового товара практически отсутствуют, компания удовлетворяет текущий спрос. 

Виды логистики

Поскольку логистику применяют в самых разных сферах деятельности, то выделяют несколько её видов. И хотя общие подходы и принципы схожи, каждое направление обладает своими особенностями. 

Транспортная логистика

Главная задача транспортной логистики — доставка грузов из одной точки в другую с минимальными затратами и без вреда для товаров. Конечная цель — найти самый оптимальный маршрут из всех возможных.   

Охватывает следующие функции: 

  • изучение груза и определение необходимых условий для его доставки; 
  • подбор транспорта или нескольких видов транспорта; 
  • построение маршрута с учётом кратчайшей доставки; 
  • организация и обеспечение качественной погрузки и разгрузки; 
  • контроль всех этапов транспортировки до передачи груза получателю. 

Закупочная логистика

Отвечает за снабжение бизнеса или производства необходимыми материалами в заданные сроки и с наименьшими затратами. 

В задачи входит:  

  • определение требуемых объёмов необходимых материалов; 
  • поиск поставщиков с наиболее выгодными условиями сотрудничества; 
  • планирование графика поставок;
  • организация закупки материалов; 
  • контроль приёма и проверки качества поставляемого товара.  

Информационная логистика

В задачи информационной логистики входит развитие систем передачи информации между субъектами. Для этого необходимо организовать своевременный и бесперебойный обмен данными между всеми отделами: снабжения, производства, хранения, транспортировки, распределения. Чаще всего для этого используют дополнительные программы, например, «1С». 

Функции: 

  • сбор данных об особенностях поставки товаров и услуг; 
  • тщательный анализ информации, которой обмениваются участники процесса;
  • распределение данных по ответственным отделам; 
  • обработка, хранение и архивация документов. 

Целью выступает объединение всех информационных потоков в единое целое. 

Производственная логистика

Оптимизирует перемещения материалов и готовых товаров внутри производственного цикла. Необходимо следить за своевременной доставкой сырья, соответствием материалов установленным критериям, отгрузкой и транспортировкой в запланированные сроки.  

К реализуемым функциям относят следующие: 

  • организация производственных операций с учётом рыночного спроса и текущих заказов;
  • планирование всех этапов производства продукции; 
  • отслеживание качественного завершения каждого отдельного этапа; 
  • постановка сроков и контроль над их соблюдением. 

Логистическая цель достигнута, если упорядочена работа всего производства и каждого отдельного цеха. 

Распределительная логистика

Основная цель распределительной, или маркетинговой логистики — распространить товары между поставщиками и продать их конечным потребителям. Важно правильно выбрать объемы поставок и сократить время транспортировки, особенно для скоропортящейся продукции. 

Функции: 

  • определение каналов распределения; 
  • выбор подходящих посредников; 
  • рационализация доставки товара до потребителя.  

Складская логистика

Складская логистика отвечает за хранение готовой продукции, грамотное использование площадей, создание резерва товаров в нужном объеме.

Охватывает следующие функции: 

  • учёт хранимых позиций товаров; 
  • организация резервного запаса; 
  • комплектация заказов на доставку;
  • выстраивание процессов приёмки и отгрузки. 

Таможенная логистика

Главная цель — соблюдение правовых норм при импорте и экспорте продукции, доставка и перевозка грузов за границу с минимальными рисками. 

В задачи таможенной логистики входит:

  • подготовка необходимых деклараций; 
  • получение сертификатов соответствия; 
  • страхование грузов; 
  • организация ответственного хранения; 
  • заключение договоров купли-продажи, в том числе по валютным сделкам; 
  • послетаможенное сопровождение.  

Цифровизация логистики

Цифровизация — это глобальный тренд логистики. Она упрощает и ускоряет процессы, что положительно отражается на качестве выполнения поставленных задач. 

С помощью различных программ и сервисов можно автоматизировать многие логистические процессы. Например: 

  • создание и изменение заявок; 
  • документооборот между участниками сделок; 
  • подбор оптимальных маршрутов; 
  • расчёт рентабельности грузоперевозок; 
  • аналитику операционной деятельности компании;  
  • управление взаиморасчётами с контрагентами; 
  • прогнозирование техобслуживания транспорта с учётом данных об эксплуатации; 
  • отслеживание передвижений транспорта. 

Яркий пример автоматизации логистической деятельности — маркетплейсы. Большинство процессов, связанных с приемом товаров и отгрузкой заказов, автоматизированы. Это помогает контролировать поставки в режиме реального времени и максимально быстро доставлять товары покупателю.

Доставка грузов на склад Яндекс.Маркета осуществляется с помощью системы

Активно применяют цифровизацию в складской логистике. Программы (их еще называют WMS-системами) помогают определить наиболее грамотное размещение товаров с учётом особенностей хранения, вести учёт в реальном времени, управлять удалёнными складами, оформлять нужные документы.

Программное обеспечение помогает автоматизировать работу склада

В транспортной логистике цифровые технологии (TMS-системы) помогают быстро и максимально точно составлять маршруты, планировать цепочки поставок, контролировать грузы при транспортировке и учитывать расходы ГСМ.

Пример онлайн-сервиса по автоматизации транспортных перевозок

Бизнес, который внедряет цифровые технологии в логистике, имеет больше шансов стать лидером рынка. Если для компании это слишком сложно и дорого, то можно частично передать логистику на аутсорс. Например, делегировать транспортировку, обработку или хранение грузов. Такой подход помогает решать логистические задачи максимально эффективно и профессионально.  

Главные мысли

Шаблон описания вакансии менеджера по логистике

 

Этот шаблон описания вакансии менеджера по логистике оптимизирован для публикации описания вакансии в сфере логистики на онлайн-досках вакансий или на страницах вакансий, и его легко настроить для вашей компании.

В обязанности менеджера по логистике входит:

  • Планирование и управление логистикой, складом, транспортировкой и обслуживанием клиентов
  • Руководство, оптимизация и координация полного цикла заказа
  • Взаимодействие и ведение переговоров с поставщиками, производителями, розничными торговцами и потребителями

Нанимаете менеджера по логистике? Подпишитесь на 15-дневную бесплатную пробную версию Workable, чтобы опубликовать эту вакансию и нанимать лучше и быстрее.

Краткое описание вакансии

Мы ищем надежного менеджера по логистике, который будет отвечать за общее управление цепочкой поставок. В обязанности менеджера по логистике входит организация и контроль хранения и распределения товаров.

Цель состоит в том, чтобы управлять всем циклом заказа, чтобы улучшить развитие бизнеса и обеспечить устойчивость и удовлетворенность клиентов.

Обязанности

  • Стратегическое планирование и управление логистикой, складом, транспортом и обслуживанием клиентов
  • Управление, оптимизация и координация полного цикла заказа
  • Поддерживать связь и вести переговоры с поставщиками, производителями, розничными торговцами и потребителями
  • Следите за качеством, количеством, запасами, сроками доставки, транспортными расходами и эффективностью
  • Организация склада, каталогизация товаров, планирование маршрутов и обработка отгрузок
  • Решить любые возникающие проблемы или жалобы
  • Надзор, обучение и обучение персонала склада
  • Достижение целей по затратам, производительности, точности и своевременности
  • Поддерживать метрики и анализировать данные для оценки производительности и внедрения улучшений
  • Соответствовать законам, правилам и требованиям ISO

Требования и навыки

  • Подтвержденный опыт работы менеджером по логистике
  • Запись об успешном управлении дистрибуцией и логистикой
  • Демонстрируемая способность руководить и управлять персоналом
  • Владение стандартным программным обеспечением для логистики
  • Отличные аналитические способности, навыки решения проблем и организаторские способности
  • Способность работать независимо и вести несколько проектов
  • BS в области делового администрирования, логистики или цепочки поставок

Часто задаваемые вопросы

Чем занимается менеджер по логистике?

Менеджеры по логистике занимаются перемещением и хранением запасов в компании. Они планируют маршруты для снижения затрат при обработке грузов для эффективного достижения бюджетных целей.

Каковы обязанности и ответственность менеджера по логистике?

Менеджер по логистике отвечает за распределение, перемещение и хранение запасов компании. В их основные обязанности входит рассмотрение бюджетов, а также обработка отгрузок.

Что делает хорошего менеджера по логистике?

Хороший менеджер по логистике должен обладать сильными математическими способностями, так как ему необходимо отслеживать множество предметов в своей организации. Когда что-то изменится на рынке или другие компании объявят о новых продуктовых линейках, они не удивятся. Гибкость также важна, потому что продукты и процессы могут быстро меняться и развиваться.

С кем работает менеджер по логистике?

Менеджер по логистике обычно руководит командой координаторов по логистике, которые наблюдают за общими процессами цепочки поставок, происходящими в компании, и получают обновления и информацию через менеджера.

определение логистической задачи | Английский словарь для учащихся

   , логистический          Логистический или логистический    означает, относящийся к организации чего-то сложного. adj   ADJ n  
Причиной задержки могут быть проблемы с логистикой… Она описала распределение продуктов питания и медикаментов как логистический кошмар.
  логистически      adv   ADV adj, ADV с v, ADV с cl  
Организованный юниорский футбол был либо ограничен, либо технически невозможен…, Пришло время, чтобы ООН рассмотрела возможность развертывания дополнительных военных ресурсов с точки зрения материально-технического обеспечения…, С точки зрения материально-технического обеспечения это очень трудно оценить юнит-связанную политику.

Перевод английского словаря Cobuild Collins &nbsp

Смотрите также:

логистика, логистика, логический, логистика

Collaborative Dictionary     English Cobuild

н.

Задача

صفحتي الشخصية Перевод платежа Remitetasks Ремитезадания выполнены Зарабатывайте деньги, выполняя небольшие задания, отвечая на опросы, вход, إعمل من أجل Я работаю над удаленными задачами, чтобы помочь другим, работа будет выполнена

эксп.

непростая задача

н.

1. [Великобритания] рабочий день; задание на день 2. [Aus.] фиксированный или определенный объем работы; рабочая квота.

[Hist.], образующийся в результате обморока от дневной работы. С арендаторов взимается дарг (или дневная работа) за каждый акр.

в.

SuperUser Do: используется в Unix/Linux «обычным» (не root) пользователем для выполнения или предварительного формирования команды или задачи «Администратора» (суперпользователь, пользователь root).

идентификатор.

пытаться или браться за задачу, которая слишком велика и выходит за рамки ваших возможностей

Интересно, сможет ли этот ремесленник выполнить три взятых на себя обязательства одновременно; по-моему он откусывает больше, чем может прожевать!

н.

Основная задача Саму по общественному здравоохранению. Оценка приема, сортировка неотложной медицинской помощи через горячую линию медицинской помощи.

См. также Врач медицинского регулятора.

н.

свободное время, потраченное на решение рабочих задач

формируется на основе «работа» и «досуг»

н.

ориентированный на детали документ, в котором четко излагаются цели проекта, такие как технические, условия и финансовые аспекты проекта программного обеспечения. Эти предложения программного обеспечения помогают бизнес-профессионалам автоматизировать рутинные задачи.

[Комп.]

в.

расставлять приоритеты в использовании своих природных способностей и определенных навыков, особенно путем выполнения задач или целей, соответствующих таким навыкам

Вы хотите отклонить эту запись: дайте нам свои комментарии (неправильный перевод/определение, повторяющиеся записи.

Конвертер из ppt в pdf онлайн бесплатно: Конвертировать PowerPoint в PDF — быстрый, онлайн, бесплатный

Объединяйте видео онлайн — объединяйте видео и клипы бесплатно

Объединяйте видео онлайн — объединяйте видео и клипы бесплатно

Убрать рекламу

Объедините два видеоклипа вместе в кратчайшие сроки!

Эта мощная бесплатная платформа для редактирования видео позволяет объединять видео онлайн, редактировать их по своему усмотрению и с легкостью создавать профессионально выглядящее видео. Вам не нужно загружать какое-либо программное обеспечение, кодеки или расширения для браузера. Здесь нет кривой обучения, а в пользовательском интерфейсе есть все необходимые инструменты. Это видео слияние работает в браузере на настольных компьютерах и мобильных устройствах, таких как смартфоны или планшеты. Платформа поддерживает различные форматы видео, кодеки и форматы, такие как MP4, MOV, AVI, WEBM, WMV, M4V, HEVC и другие. Система полностью онлайн, и это абсолютно бесплатно. Начните создавать идеальное видео из отдельных файлов прямо сейчас.

Как объединить видео онлайн?

Вы можете комбинировать два и более видеофайлов с разными входными форматами, соотношениями сторон и размерами кадров. Следуйте этим инструкциям, чтобы создать собственное видео.

  1. Загрузить видео

    Откройте «Объединение видео» в браузере на компьютере или мобильном устройстве. Нажмите «Открыть файл» или перетащите первое видео. После загрузки нажмите «Добавить» в нижней части окна редактирования и выберите другой файл.

  2. Объединить видео

    Чтобы соединить видео, вы можете обрезать их по длине прямо на временной шкале. Кроме того, вы можете расположить указатель воспроизведения и использовать инструмент «Ножницы» на выбранном видеоклипе, чтобы разделить его. Видео может иметь разное соотношение сторон, поэтому используйте Transform, чтобы избавиться от черных полос.

  3. Выберите формат для кодирования

    Нажмите на значок шестеренки рядом с Сохранить, чтобы увидеть настройки кодирования. MP4 будет работать для Интернета, MKV для автономного использования и MOV для устройств Apple. Когда вы закончите, нажмите «Сохранить», и кодирование видео займет несколько минут.

  4. Сохраните и продолжите работу

    Теперь вы можете загрузить результат видео на свое устройство или сразу же поделиться им в социальных сетях. Вы можете продолжить работу над другим видео и использовать другие инструменты, такие как «Добавить музыку», «Зациклить видео» или «Добавить текст».

Объединяйте два или более видео вместе и создавайте отличный контент

Используйте этот мощный инструмент для производства видео, чтобы объединять отдельные видеоклипы, которые вы сняли или скачали, в фильм, который передает вашу идею и эмоции миллионам пользователей в Интернете. Например, соберите витрину ваших вещей и разместите ее в Instagram. Создайте видео о своей последней поездке или проекте и станьте видеоблогером без вложений в программное обеспечение и без найма команды по производству видео. Добавьте заголовки и сделайте видеопрезентацию из нескольких клипов, которые помогут вам на вашем основном мероприятии на работе.

Наши преимущества

Безопасность

Мы обеспечиваем все необходимые меры безопасности, чтобы ваши данные, которыми вы делитесь с нами, никогда не попадали куда-либо, кроме как обратно к вам. Все наши услуги соответствуют требованиям США и ЕС по защите данных.

Хороший опыт

Монтаж видео не должен быть сложным. Мы узнали, как это делается в самых популярных инструментах и как люди ожидают, что это будет работать. Эта платформа предлагает все необходимые вам функции, и их использование очевидно.

Мощный

Наши серверы несравнимы с ПК по вычислительной мощности, поэтому все ваши видео будут преобразованы и отредактированы в одно мгновение. Кроме того, настольные инструменты создают видео высокого разрешения во время редактирования, что требует много места на диске.

Онлайн-объединитель видео

Трудно поверить, что в наши дни редактирование видео возможно через Интернет, но этот сервис делает это возможным. Больше никаких загрузок инструментов, пробных периодов, водяных знаков и других сложностей только для редактирования видео.

Интуитивно понятный интерфейс

Пользовательский интерфейс построен на временной шкале и средстве просмотра видео с помощью только необходимых инструментов в виде кнопок. Объединяйте видео, обрезайте и не ищите в интернете, как сделать то и это.

Присоединяйтесь к любому формату видео

Наш инструмент включает в себя коллекцию из более чем 30 популярных кодеков и форматов видеоконтейнеров, которые были созданы и активно используются в течение последних нескольких десятилетий, включая MPEG, MOV, WMV и многие другие.

Инструменты для конкретных форматов

Объединить GIF-файлы Соединить AVI Объединить MKV Соединить WMV Соединитель MOV Объединение MPEG Объединить FLV-файлы Объединить WEBM Соединить MP4

Часто задаваемые вопросы

Какие типы видеофайлов я могу объединить?

С помощью этой платформы вы можете загружать и объединять видеофайлы любого типа. Это связано с форматом файла, соотношением сторон, количеством кадров в секунду, алгоритмами сжатия и многими другими параметрами видео.

Могу ли я объединить видео разных форматов?

Поддерживается более 30 кодеков и типов видеофайлов, поэтому справедливо сказать, что он будет обрабатывать любое видео. Кроме того, вы можете объединить несколько форматов вместе и вывести его как один фильм, который можно воспроизводить в любом месте в Интернете.

Влияет ли качество на качество, когда я объединяю видео онлайн?

Этот онлайн-конвертер включает в себя последние версии наиболее эффективных алгоритмов сжатия мультимедиа, которые уменьшают размер выходного файла, сохраняя при этом качество видео и звука, неотличимое от исходных загруженных вами файлов.

Как соединить видео MP4 вместе?

Загружайте файлы MP4 один за другим, следуя инструкциям на этой странице. Обрежьте видео, если вам нужно, отрегулируйте размер, чтобы удалить черные полосы видео с другим соотношением сторон, и объедините их в одно выходное видео, нажав «Сохранить».

English Português Italiano Español Deutsch Français Русский Polski Türkçe 日本語 한국어 简体中文 繁體中文 tiếng Việt ภาษาไทย Bahasa Indonesia

Войти через Google

Войти через Facebook

или

Забыли пароль?

Нет аккаунта? Зарегистрируйтесь

Разрешите этому приложению работать с вашим Google Диском

PPT to PDF — онлайн-конвертер

Выберите файл для конвертации

Перетащите файлы сюда. Максимальный размер файла 100 МБ или Зарегистрироваться

У вас есть возможность преобразовать файл ppt в pdf, а также во множество других форматов, используя наш бесплатный онлайн-конвертер.

С помощью этих шагов можно легко преобразовать файл PDF в формат ppt.

Шаг 1

Загрузить ppt-файл

Вы можете выбрать ppt-файл, который хотите конвертировать, с компьютера, Google Диска, Dropbox или просто перетащить его на страницу.

Шаг 2

Выберите «в pdf»

Выберите pdf или любой другой из 200+ поддерживаемых форматов, в который вы хотите конвертировать.

Шаг 3

Загрузите файл в формате PDF

Дождитесь завершения преобразования, затем нажмите кнопку загрузки, чтобы получить преобразованный файл в формате PDF.

Преобразование ppt в pdf

Быстро и просто

Просто перетащите файлы ppt на веб-страницу, и вы сможете конвертировать их в pdf или более чем в 250 различных форматов файлов без необходимости регистрации, предоставьте адрес электронной почты или добавить водяной знак.

Вам не нужно беспокоиться о безопасности.

Как только вы загрузите файлы ppt, мы сразу же их удалим. Затем преобразованные файлы удаляются через 24 часа. Кроме того, мы гарантируем, что все передачи файлов безопасны благодаря расширенному шифрованию SSL.

Все хранится в облаке.

Вам не нужно утруждать себя установкой какого-либо программного обеспечения. Мы обрабатываем все преобразования ppt в pdf в облаке, что означает, что никакие ресурсы вашего компьютера не будут использоваться в процессе.

Microsoft PowerPoint

Portable Document Format

ppt converter

ppt to bmpppt to gifppt to jpegppt to pngppt to ppmppt to psdppt to svgppt to tiffppt to htmlppt to pdfppt to docppt to docxppt to pptxppt to odtppt to wpsppt to rtfppt to txtppt в odpppt в odgppt в otpppt в sxippt в stippt в ppsppt в potppt в sdappt в sddppt в epsppt в ppsxppt в jpgppt в epubppt в mobippt в rarppt в mp3ppt в mp4ppt в keyppt в videoppt в psp0013

arw в pdfbmp в pdfcgm в pdfcr2 в pdfdcm в pdfdjvu в pdfdng в pdfemf в pdffax в pdfgif в pdfhdr в pdfhrz в pdfico в pdfjpeg в pdfmng в pdfmtv в pdfnef в pdfpam в pdfpcx в pdfpes в pdfpgm в pdfpict в pdfnpng в pdfnpng pdfppm в pdfpsd в pdfpwp в pdfrla в pdfsfw в pdfsun в pdfsvg в pdftga в pdftiff в pdfwpg в pdfxcf в pdfxwd в pdfepdf в pdfhtml в pdfdoc в pdfdocx в pdfxls в pdfxlsx в pdfpptx в pdfodt в pdfott в pdfsxw в pdfmldocm в pdfmldocm pdfwps в pdfdot в pdfhwp в pdfrtf в pdftxt в pdfwpd в pdfdotm в pdfpages в pdflrf в pdffb2 в pdfsdw в pdfods в pdfots в pdfsxc в pdfstc в pdfxlt в pdfcsv в pdfwks в pdfdbf в pdfwodp в pdfodg в pdf topdf pdfpps в pdfdxf в pdfdxf pdfpptm в pdfppsx в pdfppt в pdfdotx в pdfpdb в pdfjpg в pdfepub в pdfmobi в pdfazw3 в pdflit в pdfsnb в pdftcr в pdfai в pdfmp3 в pdfmp4 в pdfxps в pdfoxps в pdfcbr в pdfcbz в pdftif в pdfhtm в pdfdst в pdfwaznumbers в pdf pdfdwg в ​​pdfraw в pdfprc в pdfwebp в pdfpub в pdfcdr в pdfheic в pdfplt в pdfheif в pdfavif в pdfmidi в pdfcad в pdfiso в pdfpsp в pdffig в pdfpat в pdfone в pdfjnt в pdfexp в pdfcfm в pdfosm в pdfasd в pdfcam в pdfcontact в pdfvss в pdfcal в pdfdps в pdfmac в pdftax в pdfcfa в pdfabc в pdfrtl в pdfolm в pdfmax в pdfind в pdfepf в pdffit в pdfsla в pdfmuse в pdfpez в pdfipynb в pdfaspx в pdfdat в pdfjpe в pdfjp2 в pdfjps в pdfexr в pdfmap в pdfppsm в pdfjif в pdfjbg в pdfabw в pdfpotx в pdfdbk в pdfwmz в pdf

PPT в PDF — конвертировать PPT в PDF онлайн

Перетащите документ PowerPoint (ppt или pptx), чтобы преобразовать его в документ PDF.

максимально допустимый размер: 5 МБ

Выберите файл PPT/PPTX

Преобразование PPT в PDF онлайн

Воспользуйтесь нашим онлайн-инструментом для преобразования презентаций Powerpoint PPT или PPTX в файл PDF.

Лучший конвертер PPT в PDF!

С помощью функции одним щелчком можно перейти от презентации Powerpoint к формату PDF одним нажатием кнопки. нажмите.

Быстрое преобразование PowerPoint в PDF

Просто перетащите или загрузите файлы PPT и получите новый формат PDF в секунды.

Выполните следующие действия, чтобы быстро преобразовать файл PowerPoint PPT в PDF:

  • 1. Нажмите «Выбрать файлы PPT» или перетащите файл в зону перетаскивания.
  • 2. Выберите файл PPT или PPTX, который вы хотите преобразовать.
  • 3. Наш инструмент будет автоматически конвертировать.
  • 4. Подождите, пока конвертер завершит преобразование файлов.
  • 5. Ваша презентация PPT была преобразована в файл PDF.

Решите систему уравнений методом крамера онлайн: Онлайн калькулятор. Решение систем линейных уравнений. Метод Крамера

Примеры решения системы линейных алгебраических уравнений 3-его порядка методом Крамера, пример № 1

СЛАУ 3-его порядка: 1 — 2 — 3 — 4 — 5 — 6 — 7 — 8 — 9 — 10 — 11 — 12


Условие

 2x 1 + x 2 + 2x 3   =   1
 3x 1 — x 2 + 2x 3   =   1
 4x 1 — x 2 + 5x 3   =   -3

Решение системы линейных алгебраических уравнений методом Крамера

Для проверки ответов можете воспользоваться нашим онлайн сервисом — Решение системы линейных уравнений методом Крамера. Если после изучения примеров решения задач у Вас останутся вопросы, то Вы всегда можете задать их на форуме, и не забывайте про наши онлайн калькуляторы для решения задач по математике и другим предметам!

Систему уравнений можно представить в матричной форме: Ax = B, где А — основная матрица (квадратная матрица), В — матрица свободных членов.

Теперь необходимо найти 4 определителя: определитель основной матрицы (определитель системы) и 3 определителя дополнительных матриц. Перед нахождением определителей советуем ознакомиться с теорией определителей матриц, а для нахождения определителей советуем использовать нашу программу — нахождение определителя матрицы.

Перепишем систему линейных алгебраических уравнений в матричную форму. Слева от разделительной линии стоят коэффициенты при переменных, а справа стоят свободные члены.


Найдем определитель основной матрицы:


Δ =  =  — 2 · 1 · 5 + 1 · 2 · 4 — 2 · 3 · 1 + 2 · 1 · 4 + 2 · 1 · 2 — 5 · 1 · 3 = -11

Определитель основной матрицы не равен нуля, значит система невырожденная.

Найдем определители 3 дополнительных матриц:

Дополнительная матрица получается из основной путем замены элементов одного из трех столбцов основной матрицы элементами матрицы свободных членов.


Δ 1 =  =  — 1 · 1 · 5 — 1 · 2 · 3 — 2 · 1 · 1 — 2 · 1 · 3 + 2 · 1 · 1 — 5 · 1 · 1 = -22

Δ 2 =  =  2 · 1 · 5 + 1 · 2 · 4 — 2 · 3 · 3 — 2 · 1 · 4 + 2 · 3 · 2 — 5 · 1 · 3 = -11

Δ 3 =  =  2 · 1 · 3 + 1 · 1 · 4 — 1 · 3 · 1 + 1 · 1 · 4 + 1 · 1 · 2 + 3 · 1 · 3 = 22

Найдем решения системы алгебраических уравнений:

х1 = Δ1/Δ = 2
х2 = Δ2/Δ = 1
х3 = Δ3/Δ = -2


Вы поняли, как решать? Нет?

Другие примеры

Метод Крамера. Примеры решения систем линейных алгебраических уравнений методом Крамера.

Высшая математика » Системы линейных алгебраических уравнений » Метод Крамера

Метод Крамера предназначен для решения тех систем линейных алгебраических уравнений (СЛАУ), у которых определитель матрицы системы отличен от нуля. Естественно, при этом подразумевается, что матрица системы квадратна (понятие определителя существует только для квадратных матриц). Решение системы уравнений методом Крамера проходит за три шага простого алгоритма:

  1. Составить определитель матрицы системы (его называют также определителем системы), и убедиться, что он не равен нулю, т.е. $\Delta\neq 0$.
  2. Для каждой переменной $x_i$($i=\overline{1,n}$) необходимо составить определитель $\Delta_{x_i}$, полученный из определителя $\Delta$ заменой i-го столбца столбцом свободных членов заданной СЛАУ.
  3. Найти значения неизвестных по формуле $x_i=\frac{\Delta_{x_{i}}}{\Delta}$ ($i=\overline{1,n}$).

Перед переходом к чтению примеров рекомендую ознакомиться с правилами вычисления определителей второго и третьего порядка, изложенными здесь.

Пример №1

Решить СЛАУ $\left\{\begin{aligned} & 3x_1+2x_2=-11;\\ & -x_1+5x_2=15. \end{aligned}\right. $ методом Крамера.

Решение

Матрица системы такова: $ A=\left( \begin{array} {cc} 3 & 2\\ -1 & 5 \end{array} \right)$. Определитель этой матрицы:

$$\Delta=\left| \begin{array} {cc} 3 & 2\\ -1 & 5 \end{array}\right|=3\cdot 5-2\cdot(-1)=17.$$

Как вычисляется определитель второго порядка можете глянуть здесь.

Так как определитель системы не равен нулю, то продолжаем решение методом Крамера. Вычислим значения двух определителей: $\Delta_{x_1}$ и $\Delta_{x_2}$. Определитель $\Delta_{x_1}$ получаем из определителя $\Delta=\left| \begin{array} {cc} 3 & 2\\ -1 & 5 \end{array}\right|$ заменой первого столбца (именно этот столбец содержит коэффициенты при $x_1$) столбцом свободных членов $\left(\begin{array} {c} -11\\ 15\end{array}\right)$:

$$ \Delta_{x_1}=\left|\begin{array}{cc}-11&2\\15&5\end{array}\right|=-55-30=-85. $$

Аналогично, заменяя второй столбец в $\Delta=\left|\begin{array}{cc}3&2\\-1&5\end{array}\right|$ столбцом свободных членов, получим:

$$ \Delta_{x_2}=\left|\begin{array} {cc} 3 & -11\\ -1 & 15\end{array}\right|=45-11=34. $$

Теперь можно найти значения неизвестных $x_1$ и $x_2$.

$$x_1=\frac{\Delta_{x_1}}{\Delta}=\frac{-85}{17}=-5;\;x_2=\frac{\Delta_{x_2}}{\Delta}=\frac{34}{17}=2.$$

В принципе, можно ещё проверить, правильно ли решена система методом Крамера. Подставим в заданную СЛАУ $x_1=-5$, $x_2=2$:

$$\left\{\begin{aligned} & 3x_1+2x_2=3\cdot(-5)+2\cdot{2}=-11;\\ & -x_1+5x_2=-(-5)+5\cdot{2}=15. \end{aligned}\right.$$

Проверка пройдена, решение системы уравнений методом Крамера найдено верно. Осталось лишь записать ответ.

Ответ: $x_1=-5$, $x_2=2$.

Пример №2

Решить СЛАУ $ \left\{\begin{aligned} & 2x_1+x_2-x_3=3;\\ & 3x_1+2x_2+2x_3=-7;\\ & x_1+x_3=-2. \end{aligned} \right.$, используя метод Крамера.

Решение

Определитель системы:

$$\Delta=\left| \begin{array} {ccc} 2 & 1 & -1\\ 3 & 2 & 2 \\ 1 & 0 & 1 \end{array}\right|=4+2+2-3=5.$$

Как вычисляется определитель третьего порядка можете глянуть здесь.

Заменяя первый столбец в $\Delta$ столбцом свободных членов, получим $\Delta_{x_1}$:

$$ \Delta_{x_1}=\left| \begin{array} {ccc} 3 & 1 & -1\\ -7 & 2 & 2 \\ -2 & 0 & 1 \end{array}\right|=6-4-4+7=5. $$

Заменяя второй столбец в $\Delta$ столбцом свободных членов, получим $\Delta_{x_2}$:

$$ \Delta_{x_2}=\left| \begin{array} {ccc} 2 & 3 & -1\\ 3 & -7 & 2 \\ 1 & -2 & 1 \end{array}\right|=-14+6+6-7-9+8=-10. $$

Заменяя третий столбец в $\Delta$ столбцом свободных членов, получим $\Delta_{x_3}$:

$$ \Delta_{x_3}=\left| \begin{array} {ccc} 2 & 1 & 3\\ 3 & 2 & -7 \\ 1 & 0 & -2 \end{array}\right|=-8-7-6+6=-15. $$

Учитывая все вышеизложенное, имеем:

$$ x_1=\frac{\Delta_{x_1}}{\Delta}=\frac{5}{5}=1;\; x_2=\frac{\Delta_{x_2}}{\Delta}=\frac{-10}{5}=-2; \; x_3=\frac{\Delta_{x_3}}{\Delta}=\frac{-15}{5}=-3. $$

Метод Крамера завершён. Можно проверить, верно ли решена система уравнений методом Крамера, подставив значения $x_1=1$, $x_2=-2$ и $x_3=-3$ в заданную СЛАУ:

$$\left\{\begin{aligned} & 2x_1+x_2-x_3=2\cdot{1}+(-2)-(-3)=3;\\ & 3x_1+2x_2+2x_3=3\cdot{1}+2\cdot(-2)+2\cdot(-3)=-7;\\ & x_1+x_3=1+(-3)=-2. \end{aligned} \right.$$

Проверка пройдена, решение системы уравнений методом Крамера найдено верно.

Ответ: $x_1=1$, $x_2=-2$, $x_3=-3$.

Пример №3

Решить СЛАУ $\left\{\begin{aligned} & 2x_1+3x_2-x_3=15;\\ & -9x_1-2x_2+5x_3=-7. \end{aligned}\right.$ используя метод Крамера.

Решение

Матрица системы $ \left( \begin{array} {ccc} 2 & 3 & -1\\ -9 & -2 & 5 \end{array} \right) $ не является квадратной. Однако это вовсе не означает, что решение системы уравнений методом Крамера невозможно. Преобразуем заданную СЛАУ, перенеся переменную $x_3$ в правые части уравнений:

$$ \left \{ \begin{aligned} & 2x_1+3x_2=x_3+15;\\ & -9x_1-2x_2=-5x_3-7. \end{aligned} \right. $$

Теперь матрица системы $ \left( \begin{array} {cc} 2 & 3 \\ -9 & -2 \end{array} \right) $ стала квадратной, и определитель её $\Delta=\left| \begin{array} {cc} 2 & 3\\ -9 & -2 \end{array}\right|=-4+27=23$ не равен нулю. Применим метод Крамера аналогично предыдущим примерам:

$$ \begin{aligned} & \Delta_{x_1} =\left| \begin{array} {cc} x_3+15 & 3\\ -5x_3-7 & -2 \end{array}\right| =-2x_3-30-\left(-15x_3-21\right) =13x_3-9;\\ \\ & \Delta_{x_2} =\left| \begin{array} {cc} 2 & x_3+15\\ -9 & -5x_3-7 \end{array}\right| =-10x_3-14-\left(-9x_3-135\right) =-x_3+121. \end{aligned} $$ $$ x_1=\frac{\Delta_{x_1}}{\Delta}=\frac{13x_3-9}{23};\; x_2=\frac{\Delta_{x_2}}{\Delta}=\frac{-x_3+121}{23}. $$

Ответ можно записать в таком виде: $\left\{\begin{aligned} & x_1=\frac{13x_3-9}{23};\\ & x_2=\frac{-x_3+121}{23};\\ & x_3\in R. \end{aligned}\right.$ Переменные $x_1$, $x_2$ – базисные (в иной терминологии – основные), а переменная $x_3$ – свободная (в иной терминологии – неосновная). Проверка, при необходимости, проводится так же, как и в предыдущих примерах.

Примечание

В подобных примерах возможна ситуация, когда после переноса переменной (или переменных) в правые части уравнений, определитель системы равняется нулю. В этом случае можно перенести в правую часть иную переменную (или переменные). Например, рассмотрим СЛАУ $\left\{\begin{aligned} & 2x_1-5x_2+10x_3=14;\\ & -4x_1+10x_2-7x_3=5. \end{aligned}\right.$. Если перенести в правые части уравнений $x_3$, получим: $ \left\{\begin{aligned} &2x_1-5x_2=-10x_3+14;\\ &-4x_1+10x_2=7x_3+5. \end{aligned}\right.$. Определитель данной системы $\Delta=\left| \begin{array} {cc} 2 & -5\\ -4 & 10 \end{array}\right|=20-20=0$. Однако если перенести в правые части уравнений переменную $x_2$, то получим систему $ \left\{\begin{aligned} &2x_1+10x_3=5x_2+14;\\ &-4x_1-7x_3=-10x_2+5. \end{aligned}\right.$, определитель которой $\Delta=\left| \begin{array} {cc} 2 & 10\\ -4 & -7 \end{array}\right|=-14+40=26$ не равен нулю. Дальнейшее решение аналогично рассмотренному в примере №3.

Пример №4

Решить СЛАУ

$$\left\{\begin{aligned} &x_1-5x_2-x_3-2x_4+3x_5=0;\\ &2x_1-6x_2+x_3-4x_4-2x_5=0; \\ &-x_1+4x_2+5x_3-3x_4=0. \end{aligned}\right.$$

методом Крамера.

Решение

Матрица системы $\left(\begin{array} {ccccc} 1 & -5 & -1 & -2 & 3 \\ 2 & -6 & 1 & -4 & -2 \\ -1 & 4 & 5 & -3 & 0 \end{array}\right)$ не является квадратной. Преобразуем заданную СЛАУ, перенеся переменные $x_4$, $x_5$ в правые части уравнений, и применим метод Крамера:

$$ \left\{\begin{aligned} & x_1-5x_2-x_3=2x_4-3x_5;\\ & 2x_1-6x_2+x_3=4x_4+2x_5; \\ & -x_1+4x_2+5x_3=3x_4. \end{aligned}\right.$$ $$ \begin{aligned} & \Delta =\left| \begin{array} {ccc} 1 & -5 & -1\\ 2 & -6 & 1\\-1 & 4 & 5 \end{array}\right| =19;\\ \\ & \Delta_{x_1} =\left| \begin{array} {ccc} 2x_4-3x_5 & -5 & -1\\ 4x_4+2x_5 & -6 & 1\\3x_4 & 4 & 5 \end{array}\right| =-17x_4+144x_5;\\ \\ & \Delta_{x_2} =\left| \begin{array} {ccc} 1 & 2x_4-3x_5 & -1\\ 2 & 4x_4+2x_5 & 1\\-1 & 3x_4 & 5 \end{array}\right| =-15x_4+41x_5;\\ \\ & \Delta_{x_3} =\left| \begin{array} {ccc} 1 & -5 & 2x_4-3x_5\\ 2 & -6 & 4x_4+2x_5\\-1 & 4 & 3x_4 \end{array}\right| =20x_4-4x_5. \end{aligned} $$

Ответ таков: $\left\{\begin{aligned} & x_1=\frac{-17x_4+144x_5}{19};\\ & x_2=\frac{-15x_4+41x_5}{19};\\ & x_3=\frac{20x_4-4x_5}{19}; \\ & x_4\in R; \; x_5\in R. \end{aligned}\right.$ Переменные $x_1$, $x_2$, $x_3$ – базисные, переменные $x_4$, $x_5$ – свободные.

Естественно, что применение метода Крамера в случаях вроде того, что рассмотрен в примере №4, не всегда оправдано с точки зрения временных затрат. Мы ведь не можем гарантировать, что после переноса каких-либо переменных в правые части уравнений, определитель системы не будет равен нулю. А перебирать различные варианты – слишком долгий процесс. Гораздо удобнее в таком случае применить метод Гаусса. Я привёл пример №4 лишь с одной целью – показать, что метод Крамера применим вне зависимости от содержимого правых частей уравнений заданной СЛАУ (числа, переменные, функции – не имеет значения). Главное, чтобы определитель матрицы системы был отличен от нуля.

Вернуться к списку тем

Задать вопрос на форуме

Записаться на занятия

Онлайн-занятия по высшей математике

9.8: Решение систем с правилом Крамера

  1. Последнее обновление
  2. Сохранить как PDF
  • Идентификатор страницы
    1390
    • OpenStax
    • OpenStax
    Цели обучения
    • Оценить определители 2 × 2.
    • Используйте правило Крамера, чтобы решить систему уравнений с двумя переменными.
    • Оценить 3 × 3 определителя.
    • Используйте правило Крамера, чтобы решить систему из трех уравнений с тремя переменными.
    • Знать свойства определителей.

    Мы научились решать системы уравнений с двумя переменными и тремя переменными, а также несколькими методами: подстановкой, сложением, методом исключения Гаусса, использованием обратной матрицы и построением графика. Некоторые из этих методов легче применять, чем другие, и они более подходят в определенных ситуациях. В этом разделе мы изучим еще две стратегии решения систем уравнений.

    Вычисление определителя матрицы 2 × 2

    Определитель — это действительное число, которое может быть очень полезным в математике, поскольку оно имеет множество применений, например, для вычисления площади, объема и других величин. Здесь мы будем использовать определители, чтобы выяснить, является ли матрица обратимой, используя элементы квадратной матрицы, чтобы определить, существует ли решение системы уравнений. Однако, возможно, одним из наиболее интересных приложений является их использование в криптографии. Защищенные сигналы или сообщения иногда отправляются закодированными в матрице. Данные могут быть расшифрованы только с помощью обратимой матрицы и определителя. Для наших целей мы сосредоточимся на определителе как признаке обратимости матрицы. Вычисление определителя матрицы включает в себя следование определенным шаблонам, описанным в этом разделе.

    НАЙТИ ОПРЕДЕЛИТЕЛЬ МАТРИЦЫ 2 × 2

    Определитель матрицы 2 × 2,

    \(A=\begin{bmatrix}a&b\\c&d\end{bmatrix}\)

    равен определяется как

    Обратите внимание на изменение обозначения. Есть несколько способов указать определитель, в том числе \(\det(A)\) и замена скобок в матрице прямыми, \(| A |\).

    Пример \(\PageIndex{1}\): нахождение определителя матрицы \(2 × 2\)

    Найдите определитель данной матрицы.

    \(A=\begin{bmatrix}5&2\\−6&3\end{bmatrix}\)

    Решение

    \[\begin{align*} \det(A)&= \begin{vmatrix} 5&2\\-6&3\end{vmatrix}\\ &= 5(3)-(-6)(2)\\ &= 27 \end{align*}\]

    Использование правила Крамера для решения системы двойки Уравнения с двумя переменными

    Теперь мы представим последний метод решения систем уравнений, использующий определители. Известен как Правило Крамера , этот метод восходит к середине 18 века и назван в честь его новатора, швейцарского математика Габриэля Крамера (1704-1752), который представил его в 1750 году в Introduction à l’Analyse des lignes Courbes algébriques . Правило Крамера — жизнеспособный и эффективный метод поиска решений систем с произвольным числом неизвестных при условии, что у нас есть такое же количество уравнений, как и неизвестных.

    Правило Крамера даст нам единственное решение системы уравнений, если она существует. Однако, если система не имеет решения или имеет бесконечное число решений, на это будет указывать нулевой определитель. Чтобы выяснить, является ли система противоречивой или зависимой, придется использовать другой метод, такой как исключение.

    Чтобы понять правило Крамера, давайте внимательно посмотрим, как мы решаем системы линейных уравнений, используя основные операции со строками. Рассмотрим систему двух уравнений с двумя переменными.

    \[\begin{align} a_1x+b_1y&= c_1 (1) \label{eq1}\\ a_2x+b_2y&= c_2 (2) \label{eq2}\\ \end{align}\]

    Исключаем одну переменную, используя операции со строками, и решить для другой. Скажем, что мы хотим найти \(x\). Если уравнение \ref{eq2} умножается на коэффициент, противоположный коэффициенту \(y\) в уравнении \ref{eq1}, уравнение \ref{eq1} умножается на коэффициент \(y\) в уравнении \ref {eq2}, и мы добавим два уравнения, переменная \(y\) будет исключена.

    \[\begin{align*} &b_2a_1x+b_2b_1y = b_2c_1 & \text{Умножить }R_1 \text{ на }b_2 \\ -&\underline{b_1a_2x-b_1b_2y=-b_1c_2} & \text{Умножить }R_2 \ text{ by }−b_1 \\ & b_2a_1x−b_1a_2x=b_2c_1−b_1c_2 \end{align*}\]

    Теперь найдите \(x\).

    \[\begin{align*} b_2a_1x−b_1a_2x &= b_2c_1−b_1c_2 \\ x(b_2a_1−b_1a_2) &= b_2c_1−b_1c_2 \\ x &= \dfrac{b_2c_1−b_1c_2}{b_2a_1−b_1a_2}=\ dfrac{\begin{bmatrix}c_1&b_1\\c_2&b_2\end{bmatrix}}{\begin{bmatrix}a_1&b_1\\a_2&b_2\end{bmatrix}} \end{align*}\]

    Аналогично, чтобы найти \(y\), мы исключим \(x\).

    \[\begin{align*} & a_2a_1x+a_2b_1y = a_2c_1 & \text{Multiply }R_1 \text{ by }a_2 \\ -& \underline{a_1a_2x-a_1b_2y=-a_1c_2} & \text{Multiply }R_2 \text{ by }-a_1 \\ & a_2b_1y-a_1b_2y =a_2c_1-a_1c_2 \end{align*}\]

    Решение для \(y\) дает

    \[ \begin{align*} a_2b_1y-a_1b_2y &= a_2c_1−a_1c_2 \\ y(a_2b_1−a_1b_2) &= a_2c_1−a_1c_2 \\ y &= \dfrac{a_2c_1−a_1c_2}{a_2b_1−a_1b_2}=\dfrac{a_1c_2−a_2c_1}{a_1b_2−a_2b_1}=\dfrac{ \begin{bmatrix}a_1&c_1\\a_2&c_2\end{bmatrix}}{\begin{bmatrix}a_1&b_1\\a_2&b_2\end{bmatrix}} \end{align*}\]

    Обратите внимание, что знаменатель для \(x\) и \(y\) является определителем матрицы коэффициентов.

    Мы можем использовать эти формулы для нахождения \(x\) и \(y\), но правило Крамера также вводит новые обозначения: детерминанты. Тогда мы можем выразить \(x\) и \(y\) как частное двух определителей.

    ПРАВИЛО КРАМЕРА ДЛЯ СИСТЕМ \(2×2\)

    Правило Крамера — это метод, использующий определители для решения систем уравнений, в которых число уравнений равно числу переменных.

    Рассмотрим систему двух линейных уравнений с двумя переменными.

    \[\begin{align*} a_1x+b_1y&= c_1\\ a_2x+b_2y&= c_2 \end{align*}\]

    Решение с использованием правила Крамера дается как

    \[\begin{align} x& = \dfrac{D_x}{D} = \dfrac{\begin{bmatrix}c_1&b_1\\c_2&b_2\end{bmatrix}}{\begin{bmatrix}a_1&b_1\\a_2&b_2\end{bmatrix}}\; , D\neq 0\\ y&= \dfrac{D_y}{D} = \dfrac{\begin{bmatrix}a_1&c_1\\a_2&c_2\end{bmatrix}}{\begin{bmatrix}a_1&b_1\\a_2&b_2\end{bmatrix }}\; , D\neq 0 \end{align}\]

    Если мы находим \(x\), столбец \(x\) заменяется столбцом констант. Если мы ищем \(y\), столбец \(y\) заменяется постоянным столбцом.

    Пример \(\PageIndex{2}\): использование правила Крамера для решения системы \(2 × 2\)

    Решите следующую систему \(2 × 2\), используя правило Крамера.

    \[\begin{align*} 12x+3y&= 15\\ 2x-3y&= 13 \end{align*}\]

    Решение

    Найдите \(x\).

    \[\begin{align*} x&= \dfrac{D_x}{D}\\ &= \dfrac{\begin{bmatrix}15&3\\13&-3\end{bmatrix}}{\begin{bmatrix} 12&3\\2&-3\end{bmatrix}}\\ &= \dfrac{-45-39}{-36-6}\\ &= \dfrac{-84}{-42}\\ &= 2 \end{align*}\]

    Найдите \(y\).

    \[\begin{align*} y&= \dfrac{D_y}{D}\\ &= \dfrac{\begin{bmatrix}12&15\\2&13\end{bmatrix}}{\begin{bmatrix}12&3\ \2&-3\end{bmatrix}}\\ &= \dfrac{156-30}{-36-6}\\ &= -\dfrac{126}{42}\\ &= -3 \end{align *}\]

    Решение: \((2,−3)\).

    Упражнение \(\PageIndex{1}\)

    Используйте правило Крамера для решения системы \(2 × 2\) уравнений.

    \[\begin{align*} x+2y&= -11\\ -2x+y&= -13 \end{align*}\]

    Ответ

    \((3,−7)\)

    Вычисление определителя матрицы 3 × 3

    Найти определитель матрицы 2 × 2 несложно, но определить определитель матрицы 3 × 3 сложнее. Один из методов состоит в том, чтобы дополнить матрицу 3×3 повторением первых двух столбцов, получив матрицу 3×5. Затем вычисляем сумму произведений записей вниз по по каждой из трех диагоналей (слева вверху справа внизу) и вычесть произведения записей вверх по по каждой из трех диагоналей (слева внизу справа вверху). Это легче понять с визуальным и пример.

    Найдите определитель матрицы 3×3.

    \(A=\begin{bmatrix}a_1&b_1&c_1\\a_2&b_2&c_2\\a_3&b_3&c_3\end{bmatrix}\)

    1. Дополнить \(A\) первыми двумя столбцами.

      \(\det(A)=\left| \begin{array}{ccc|cc} a_1&b_1&c_1&a_1&b_1\\a_2&b_2&c_2&a_2&b_2\\a_3&b_3&c_3&a_3&b_3\end{array} \right|\)

    2. От верхнего левого угла к нижнему правому: умножьте числа по первой диагонали. Прибавьте результат к произведению записей по второй диагонали. Добавьте этот результат к произведению записей вниз по третьей диагонали.
    3. Из нижнего левого угла в верхний правый: вычтите произведение записей вверх по первой диагонали. Из этого результата вычтите произведение вхождений вверх по второй диагонали. Из этого результата вычтите произведение вхождений вверх по третьей диагонали.

    Алгебра выглядит следующим образом:

    \(| A |=a_1b_2c_3+b_1c_2a_3+c_1a_2b_3−a_3b_2c_1−b_3c_2a_1−c_3a_2b_1\)

    Пример определения {3} × 3 Matrix

    Найдите определитель матрицы \(3 × 3\) по данным

    \(A=\begin{bmatrix}0&2&1\\3&−1&1\\4&0&1\end{bmatrix}\)

    Решение

    Дополните матрицу первыми двумя столбцами и следуйте формуле. Таким образом,

    \[\begin{align*} | А | &= \влево| \begin{массив}{ccc|cc}0&2&1&0&2\\3&-1&1&3&-1\\4&0&1&4&0\end{массив}\right| \\ &= 0(−1)(1)+2(1)(4)+1(3)(0)−4(−1)(1)−0(1)(0)−1(3) (2) \\ &=0+8+0+4−0−6 \\ &= 6 \end{align*}\]

    Упражнение \(\PageIndex{2}\)

    Найдите определитель Матрица 3 × 3.

    \(\det(A)=\begin{vmatrix}1&−3&7\\1&1&1\\1&−2&3\end{vmatrix}\)

    Ответ

    \(−10\)

    Вопросы и ответы: Можно ли использовать тот же метод для нахождения определителя матрицы большего размера?

    Нет, этот метод работает только для матриц 2 × 2 и 3 × 3. Для больших матриц лучше всего использовать графическую утилиту или компьютерное программное обеспечение.

    Использование правила Крамера для решения системы трех уравнений с тремя переменными

    Теперь, когда мы можем найти определитель матрицы \(3 × 3\), мы можем применить правило Крамера для решения системы трех уравнений с тремя переменные. Правило Крамера является простым и следует шаблону, согласующемуся с правилом Крамера для матриц \(2 × 2\). Однако по мере увеличения порядка матрицы до \(3 × 3\) требуется гораздо больше вычислений.

    Когда мы вычисляем определитель равным нулю, правило Крамера не указывает, имеет ли система решение или бесконечное число решений. Чтобы выяснить это, мы должны выполнить исключение в системе.

    Рассмотрим систему уравнений \(3 × 3\).

    \[\begin{align} a_1x+b_1y+c_1z &= \color{blue}d_1 \\ a_2x+b_2y+c_2z &= \color{blue}d_2 \\ a_3x+b_3y+c_3z &= \color{blue }d_3 \\ \end{align}\]

    \(x=\dfrac{D_x}{D}\), \(y=\dfrac{D_y}{D}\), \(z=\dfrac{ D_z}{D}\), \(D≠0\)

    где

    \[D = \begin{vmatrix} a_1 & b_1 & c_1\\ a_2 & b_2 & c_2\\ a_3 & b_3 & c_3 \end{vmatrix}\; ,\; D_x = \begin{vmatrix} \color{blue}d_1 & b_1 & c_1\\ \color{blue}d_2 & b_2 & c_2\\ \color{blue}d_3 & b_3 & c_3 \end{vmatrix}\; ,\; D_y = \begin{vmatrix} a_1 & \color{blue}d_1 & c_1\\ a_2 & \color{blue}d_2 & c_2\\ a_3 & \color{blue}d_3 & c_3 \end{vmatrix}\; ,\; D_z = \begin{vmatrix} a_1 & b_1 & \color{blue}d_1\\ a_2 & b_2 & \color{blue}d_2\\ a_3 & b_3 & \color{blue}d_3 \end{vmatrix}\]

    Если мы записываем определитель \(D_x\), мы заменяем столбец \(x\) постоянным столбцом. Если мы записываем определитель \(D_y\), мы заменяем их столбец y на постоянный столбец. Если мы записываем определитель \(D_z\), мы заменяем столбец \(z\) постоянным столбцом. Всегда проверяйте ответ.

    Пример \(\PageIndex{4}\): решение системы \(3 × 3\) с помощью правила Крамера

    Найдите решение данной системы \(3 × 3\) с помощью правила Крамера.

    \[\begin{align*} x+y-z&= 6\\ 3x-2y+z&= -5\\ x+3y-2z&= 14 \end{align*}\]

    Решение

    Используйте правило Крамера.

    \(D=\begin{vmatrix}1&1&−1\\3&−2&1\\1&3&−2\end{vmatrix}\), \(D_x=\begin{vmatrix}6&1&−1\\−5&−2&1 \\14&3&-2\end{vmatrix}\), \(D_y=\begin{vmatrix}1&6&-1\\3&-5&1\\1&14&-2\end{vmatrix}\), \(D_z=\begin{ vmatrix}1&1&6\\3&-2&-5\\1&3&14\end{vmatrix}\)

    Затем

    \[\begin{align*} x&= \dfrac{D_x}{D}&= \dfrac{- 3}{-3}&= 1\\ y&= \dfrac{D_y}{D}&= \dfrac{-9}{-3}&= 3\\ z&= \dfrac{D_z}{D}&= \dfrac{6}{-3}&= -2\\ \end{align*}\]

    Решение: \((1,3,−2)\).

    Упражнение \(\PageIndex{3}\)

    Используйте правило Крамера для решения матрицы \(3 × 3\).

    \[\begin{align*} x-3y+7z&= 13\\ x+y+z&= 1\\ x-2y+3z&= 4 \end{align*}\]

    Ответ

    \(\влево(−2,\dfrac{3}{5},\dfrac{12}{5}\вправо)\)

    Пример \(\PageIndex{5A}\): использование правила Крамера для решения несогласованной системы

    Решите систему уравнений с помощью правила Крамера.

    \[\begin{align} 3x-2y&= 4 \label{eq3}\\ 6x-4y&= 0 \label{eq4}\end{align}\]

    Решение

    Начнем с нахождения определители \(D\), \(D_x\) и \(D_y\).

    \(D=\begin{vmatrix}3&-2\\6&-4\end{vmatrix}=3(-4)−6(-2)=0\)

    Мы знаем, что определитель нуля означает либо система не имеет решений, либо имеет бесконечное число решений. Чтобы увидеть, какой из них, мы используем процесс исключения. Наша цель — исключить одну из переменных.

    1. Умножить уравнение \ref{eq3} на \(−2\).
    2. Добавьте результат к уравнению \ref{eq4}.

    \[\begin{align*} &−6x+4y=−8 \\ &\;\;\;\underline{6x−4y=0} \\ &\;\;\;\;\;\ ;\;\;\;\; 0=−8 \end{align*}\]

    Получаем уравнение \(0=−8\), которое неверно. Следовательно, система не имеет решения. График системы показывает две параллельные линии. См. рисунок \(\PageIndex{1}\).

    Рисунок \(\PageIndex{1}\)
    Пример \(\PageIndex{5B}\): использование правила Крамера для решения зависимой системы

    Решите систему с бесконечным числом решений.

    \[\begin{align} x-2y+3z&= 0 \label{eq5}\\ 3x+y-2z&= 0 \label{eq6}\\ 2x-4y+6z&= 0 \label{eq7} \ end{align}\]

    Решение

    Сначала найдем определитель. Настройте матрицу, дополненную первыми двумя столбцами.

    \(\left| \begin{array}{ccc|cc}1&−2&3&1&-2\\3&1&−2&3&1\\2&−4&6&2&-4\end{array}\right|\)

    Затем

    \(1(1)(6)+(−2)(−2)(2)+3(3)(−4)−2(1)(3)−(−4)(−2)( 1)−6(3)(−2)=0\)

    Поскольку определитель равен нулю, решения либо нет, либо существует бесконечное число решений. Мы должны выполнить исключение, чтобы узнать.

    1. Умножьте уравнение \ref{eq5} на \(−2\) и добавьте результат к уравнению \ref{eq7}:

    \[\begin{align*} &−2x+4y−6x=0 \ \ &\;\;\underline{2x−4y+6z=0} \\ &\;\;\;\;\;\;\;\;\;\;\;\;\;\;\; \;0=0 \end{align*}\]

    2. Получение ответа \(0=0\), утверждение, которое всегда истинно, означает, что система имеет бесконечное число решений. Изобразив систему, мы видим, что две плоскости одинаковы и обе пересекают третью плоскость по прямой. См. рисунок \(\PageIndex{2}\).

    Рисунок \(\PageIndex{2}\)

    Понимание свойств определителей

    У определителей много свойств. Здесь перечислены некоторые свойства, которые могут быть полезны при вычислении определителя матрицы.

    СВОЙСТВА ОПРЕДЕЛИТЕЛЕЙ
    1. Если матрица имеет верхнетреугольную форму, определитель равен произведению элементов по главной диагонали.
    2. При перестановке двух строк определитель меняет знак.
    3. Если две строки или два столбца идентичны, определитель равен нулю. 9{−1}\) — величина, обратная определителю матрицы \(A\).
    4. Если какая-либо строка или столбец умножается на константу, определитель умножается на тот же коэффициент.
    Пример \(\PageIndex{6}\): Иллюстрация свойств определителей

    Проиллюстрируйте каждое из свойств определителей.

    Решение

    Свойство 1 гласит, что если матрица имеет форму верхнего треугольника, определитель равен произведению элементов, расположенных вниз по главной диагонали.

    \(A=\begin{bmatrix}1&2&3\\0&2&1\\0&0&-1\end{bmatrix}\)

    Дополнить \(A\) первыми двумя столбцами.

    \(A=\left[ \begin{array}{ccc|cc}1&2&3&1&2\\0&2&1&0&2\\0&0&−1&0&0\end{array}\right]\)

    Затем

    \[\begin{align* } \det(A)&= 1(2)(-1)+2(1)(0)+3(0)(0)-0(2)(3)-0(1)(1)+1 (0)(2)\\ &= -2 \end{align*}\]

    Свойство 2 утверждает, что перестановка строк меняет знак. Учитывая

    \[\begin{align*} A&=\begin{bmatrix}-1&5\\4&-3\end{bmatrix}\\ \det(A)&= (-1)(-3)-(4) (5)\\ &= 3-20\\ &= -17 \end{align*}\]

    \[\begin{align*} B&= \begin{bmatrix}4&-3\\-1&5\end {bmatrix}\\ \det(B)&= (4)(5)-(-1)(-3)\\ &= 20-3\\ &= 17 \end{align*}\]

    Свойство 3 утверждает, что если две строки или два столбца идентичны, определитель равен нулю.

    \[\begin{align*} A&=\left[ \begin{array}{ccc|cc}1&2&2&1&2\\2&2&2&2&2\\-1&2&2&-1&2\end{массив}\right]\\ \det(A) &=1(2)(2)+2(2)(-1)+2(2)(2)+1(2)(2)-2(2)(1)-2(2)(2) \\ &=4-4+8+4-4-8\\ &=0 \end{align*}\] 9{-1})&=-2\left(-\dfrac{1}{2}\right)-\dfrac{3}{2}(1)\\ &=-\dfrac{1}{2} \ end{align*}\]

    Свойство 6 гласит, что если любая строка или столбец матрицы умножается на константу, определитель умножается на тот же коэффициент. Таким образом,

    \[\begin{align*} A&=\begin{bmatrix}1&2\\3&4\end{bmatrix}\\ \det(A)&=1(4)-2(3)\\ &= -2 \end{align*}\]

    \[\begin{align*} B&=\begin{bmatrix}2(1)&2(2)\\3&4\end{bmatrix}\\ \det(B) &=2(4)-3(4)\\ &=-4 \end{align*}\]

    Пример \(\PageIndex{7}\): использование правила Крамера и свойств определителя для решения системы

    Найдите решение заданной системы \(3 × 3\).

    \[\begin{align} 2x+4y+4z&=2 \label{eq8}\\ 3x+7y+7z&=-5 \label{eq9}\\ x+2y+2z&=4 \label{eq10} \end{align}\]

    Решение

    Используя правило Крамера, мы имеем

    \(D=\begin{bmatrix}2&4&4\\3&7&7\\1&2&2\end{bmatrix}\)

    Обратите внимание, что второй и третий столбцы идентичны. Согласно свойству 3 определитель будет равен нулю, поэтому решения либо нет, либо решений бесконечное множество. Мы должны выполнить исключение, чтобы узнать.

    1. Умножьте уравнение \ref{eq10} на \(–2\) и добавьте результат к уравнению \ref{eq8}.

    \[\begin{align*} -2x-4y-4x&=-8\\ 2x+4y+4z&=2\\ 0&=-6 \end{align*}\]

    Получение оператора, который является Противоречие означает, что система не имеет решений.

    Медиа

    Доступ к этим онлайн-ресурсам для получения дополнительных инструкций и практических занятий по правилу Крамера.

    • Решение системы двух уравнений с помощью правила Крамера
    • Решите систему из трех уравнений, используя правило Крамера

    Основные понятия

    • Определитель для \(\begin{bmatrix}a&b\\c&d\end{bmatrix}\) равен \(ad-bc\). См. пример \(\PageIndex{1}\).
    • Правило Крамера заменяет столбец переменной столбцом константы. Решения: \(x=\dfrac{D_x}{D}\), \(y=\dfrac{D_y}{D}\). См. пример \(\PageIndex{2}\).
    • Чтобы найти определитель матрицы \(3×3\), увеличьте первые два столбца. Сложите три диагональных элемента (слева вверху справа внизу) и вычтите три элемента по диагонали (слева внизу справа вверху). См. пример \(\PageIndex{3}\).
    • Чтобы решить систему из трех уравнений с тремя переменными с помощью правила Крамера, замените столбец переменных столбцом констант для каждого требуемого решения: \(x=\dfrac{D_x}{D}\), \(y=\dfrac{ D_y}{D}\), \(z=\dfrac{D_z}{D}\). См. пример \(\PageIndex{4}\).
    • Правило Крамера также полезно для нахождения решения системы уравнений без решения или с бесконечным числом решений. См. Пример \(\PageIndex{5}\) и Пример \(\PageIndex{6}\).
    • Некоторые свойства определителей полезны при решении задач. Например: 9{−1}\) — величина, обратная определителю матрицы \(A\).
    • Если какая-либо строка или столбец умножается на константу, определитель умножается на тот же коэффициент. См. Пример \(\PageIndex{7}\) и Пример \(\PageIndex{8}\).

    Эта страница под названием 9.8: Решающие системы с правилом Крамера распространяется под лицензией CC BY 4.0 и была создана, изменена и/или курирована OpenStax с помощью исходного контента, который был отредактирован в соответствии со стилем и стандартами платформы LibreTexts; подробная история редактирования доступна по запросу.

    1. Наверх
      • Была ли эта статья полезной?
      1. Тип изделия
        Раздел или Страница
        Автор
        ОпенСтакс
        Лицензия
        СС BY
        Версия лицензии
        4,0
        Программа OER или Publisher
        ОпенСтакс
        Показать страницу TOC
        нет
        Включено
        да
      2. Теги
        1. Правило Крамера
        2. Детерминанты
        3. источник@https://openstax. org/details/books/precalculus

      Калькулятор правила Крамера — Система уравнений 2 и 3

      Сегодня мы поделимся еще одним простым, но мощным методом анализа цепей, известным как « Правило Крамера ».

      • Анализ цепи SUPERMESH | Шаг за шагом с решенным примером

      Обновление: Мы добавили онлайн-калькулятор правила Крамера, где вы можете решить систему двух уравнений, а также систему трех уравнений. Проверьте оба калькулятора правил Крамера в обоих разделах поста. Спасибо

       

      Ниже приведено пошаговое руководство с примерами решения, в котором подробно рассказывается, как решить сложную электрическую цепь и сеть по правилу Крамера.

      Калькулятор правила Крамера для 2×2 (система двух уравнений)

      Калькулятор правила Крамера 2 x 2 (система 2 уравнений):

      х + у =

      х + у =

      х =

      у =

      Нахождение двух переменных по правилу Крамера:

      Пример 1:

      (В нашем случае неизвестными значениями являются два тока: i 1 и i 2 ) по правилу Крамера. Теперь давайте начнем.

       

      Как показано ниже, это простая электрическая цепь, и мы собираемся решить ее по правилу Крамера.

      Правило Крамера для анализа линейных цепей | 2 переменные (2×2) Пример решения.

      Решение:

      Во-первых, переставьте схему с соответствующими метками (поскольку два резистора по 5 Ом включены последовательно, поэтому мы заменим его на 10 Ом).

      Примените анализ сетки и упростите по правилу Крамера, чтобы найти неизвестные значения токов я 1 и я 2 .

      Теперь напишем уравнения КВЛ неизвестных для данной схемы

      Применить KVL на сетке (1).

      6 = 14 i 1 + 10( i 1 i 2 )

                     6 = 24 i 1 – 10 i 2 ….. → Уравнение (1)

      Также примените KVL к сетке (2).

      -5 = 10 i 2 + 10( и 2 и 1 )

                        -5 = – 10 i 1 + 20 i 2 …. . → уравнение (2)

      Здесь мы получили два уравнения, т.е.

           24 i 1 – 10 i 2   =   6

        – 10 и 1 + 20 и 2 = -5

       

      Теперь решим эти два уравнения по правилу Крамера, чтобы найти неизвестные значения (токов), которые равны i 1 и i 2 .

       

      Решение по правилу Крамера:

      Шаг 1:

      Прежде всего, запишите приведенные выше уравнения в матричной форме. то есть

      Шаг 2:

      Теперь напишите матрицу коэффициентов приведенных выше уравнений и назовите ее ∆. Убедитесь, что он квадратный, т.е. количество строк x количество столбцов. В приведенном выше случае он имеет 2 строки и 2 столбца.

       

      Шаг 3:

      Теперь найдите определитель |∆| матрицы коэффициентов ∆ следующим методом. (Приведенный ниже рисунок поможет вам в этом.)

      Нажмите на картинку, чтобы увеличить

      Нахождение матрицы коэффициентов ∆ по правилу Крамера. Простое объяснение.

      В соответствии с приведенным выше рис. последний шаг будет таким.

      Шаг 4:

      Теперь найдите определитель коэффициента Δ 1 тем же способом, что указан выше, но замените первый столбец Δ на «Столбец ответов» (Если вы не уловили смысл столбца ответов, см. рис. на шаге 2 выше или проверьте инфографику на конец примера просто обратитесь ко второму примеру ниже, где мы сделали то же самое, чтобы найти Δ 1 ), то есть

      Шаг 5:

      Теперь найдите определитель коэффициента Δ 2 , просто замените второй столбец на «Столбец ответов», то есть

      .

      Шаг 6:

      As Cramer’s rule tells that i 1 = Δ 1 / Δ and i 2 = Δ 2 / Δ .

      Сейчас найди i 1 и i 2 по правилу Крамера.

      i 1 = 0,184,2 А или 184,2 мА

       

      А,

      i 2 = 0,157,9 А или 157,9 мА

       

      Ниже представлена ​​сводка инфографики по правилу Крамера для определения двух переменных или неизвестных значений.

      Правило Крамера: простые шаги, инфографика, диаграмма

      Хорошо, это было легко… Теперь, как насчет 3 переменных…. Попробуем решить линейные уравнения с тремя переменными с помощью правила Крамера.

      Нахождение трех переменных по правилу Крамера:

      (In our case, these unknown values ​​are three currents which are i 1 , i 2 and i 3 ) by Правило Крамера. Теперь давайте начнем.

      Калькулятор правила Крамера для 3×3 (система трех уравнений)

      Калькулятор правила Крамера 3 x 3 (3 системы уравнений):

      х + у + z =

      х + у + z =

      х + у + z =

      х =

      у =

      з =

       

      Пример 2:

      Используйте анализ сетки для определения трех токов сетки в схеме ниже. Используйте правило Крамера для упрощения.

      Найдите три неизвестных значения токов по правилу Крамера.

      Прежде всего, примените KVL к каждому мешу один за другим и напишите его уравнения.

      -7+1 ( I 1 I 2 )+6+2 ( I 1 I 3 ) = 09567 1.

      .9007.

      7 1.
      .
      (

      7 19567 1.
      (

      7 1.

      7 1.9007 (

      7 1.
      (

      1

      7 ( 1

      7 ( I 3 ). 2 — I 1 ) + 2 I 2 + 3 ( I 2 I 3 ) = 0

      9 2928 2 (9005 2 (9005 2928 2 (9005 2928 2 (9005 2 (9005 2928 2 (9005 2928 2 (9005 2928 2928 2 (9005 2928 2 (9005 2928 2 (
      2928 2928 2 (
      2928 2928 2 ( 2928 2 ( 2928 2 ( 2 9005 2 ( 2 ) 1 ) – 6+3( и 3 и 2 ) + 1 I 3 = 0

      ,

      3 I 1 I 2 — 2 4 I I 2 — 2 4 I I . 1)

        – i 1 + 6 i 2 – 3 i 3 = 0                    … Eq….. (2)

       -2 i 1 – 3 i 2 + 6 i 3 = 6                           … Уравнение….. (3)

       

      Теперь запишите приведенные выше уравнения в матричной форме.

           3 i 1 i 2 – 2 i 3 = 1

         – i 1 + 6 i 2 – 3 i 3 = 0

      -2 I 1 -3 I 2 + 6 I 3 = 6

      . Как мы это сделаем? Просто проверьте рисунок ниже для лучшего объяснения.

      Щелкните изображение, чтобы увеличить его.

       Итак, полный шаг показан ниже.

      ∆ = +3 (6 x 6) – (- 3 x –3) – (-1 (-1 x 6)-(-2 x –3) + (-2 (-1 x –3) – ( -2 x 6)

      = 81 -12 -30 = 39

       

      Теперь найдите ∆ 1 так же, как описано выше, но просто замените первый столбец матрицы на «Столбец ответов». Подробности см. на рис., показанном ниже.

      Итак, вот полный шаг, чтобы найти ∆ 1 . Здесь мы заменили «Синих парней» в первом столбце на «Черных парней» 🙂

      = +1(36-9) – (–1[0+18]) –2(0-36)

      = 27 + 18 + 72

      1 = 117

       

      Снова найти ∆ 2 тем же методом, который описан ранее. Просто замените второй столбец матрицы на «Столбец ответов», т.е. замените «Красных парней» в центральном столбце на «Черных парней», как показано ниже.

      = +3 (0 +18) -1[(-6)-(+6)] –2(-6-0)

      = 54+12+12 = 78

      2 = 78

       

      Наконец, найдите последний ∆ 3 . Просто замените третий столбец на «Столбец ответов», то есть замените «Зеленых парней в третьем столбце» на «Черных парней», как показано ниже.

      = +3 (6 х 6) – (-3 х 0) – [-1(-1 х 6) – (-2 х 0)] + [1(-1) х (-3) – (- 2) x (6)]

      = 108 + 6 + 15

      3 = 117

      Сейчас, решайте и найдите неизвестные значения, т.е. I 1 , 41557777777777777777777777777777777777777777777777777777777777777777777777777777777777777777777777777777777777777777777777 1 ,

      7777777 1 . 2 и и 3 .

      As, Cramer’s rule says that, variables i.e. i 1 = ∆1/∆ 1 , i 2 = ∆/∆ 2 and i 3 = ∆/∆ 3 .

      Therefore,

       

      i 1 = ∆1/∆ 1

      = 117/39

      i 1 = 3A

       

      And i 2 ,

      i 2 = = ∆/∆ 2

      = 78/39

      i 2 =

       

      И, наконец, i 3 ;

      i 3 = ∆/∆ 3

      = 117/39

      i 3 = 3А.

      Проверить совместимость системы уравнений как: Как исследовать систему уравнений на совместность

      исследование линейных систем уравнений

      Вы искали исследование линейных систем уравнений? На нашем сайте вы можете получить ответ на любой математический вопрос здесь. Подробное решение с описанием и пояснениями поможет вам разобраться даже с самой сложной задачей и исследование матрицы на совместимость, не исключение. Мы поможем вам подготовиться к домашним работам, контрольным, олимпиадам, а так же к поступлению в вуз. И какой бы пример, какой бы запрос по математике вы не ввели — у нас уже есть решение. Например, «исследование линейных систем уравнений».

      Применение различных математических задач, калькуляторов, уравнений и функций широко распространено в нашей жизни. Они используются во многих расчетах, строительстве сооружений и даже спорте. Математику человек использовал еще в древности и с тех пор их применение только возрастает. Однако сейчас наука не стоит на месте и мы можем наслаждаться плодами ее деятельности, такими, например, как онлайн-калькулятор, который может решить задачи, такие, как исследование линейных систем уравнений,исследование матрицы на совместимость,исследование на совместимость матрицы,исследование на совместность системы линейных уравнений,исследование систем линейных уравнений,исследование системы линейных уравнений на совместность,исследование системы на совместность онлайн,исследовать на совместность систему линейных уравнений онлайн,исследовать на совместность систему на совместность онлайн,исследовать на совместность систему уравнений,исследовать систему линейных уравнений,исследовать систему линейных уравнений на совместность,исследовать систему линейных уравнений на совместность онлайн,исследовать систему на совместность и определенность не решая ее,исследовать систему на совместность и решить ее если она совместна,исследовать систему на совместность онлайн,исследовать систему уравнений на совместимость и определенность не решая ее,исследовать систему уравнений на совместность,как исследовать систему линейных уравнений на совместность,как определить совместность системы линейных уравнений,как проверить на совместность систему,как проверить систему на совместность,как проверить совместность системы линейных уравнений,как проверить совместность системы уравнений,матрицы исследование на совместимость,проверить на совместность систему,проверить систему на совместность,проверить совместность системы уравнений,проверка на совместность системы,совместность системы линейных уравнений как проверить. На этой странице вы найдёте калькулятор, который поможет решить любой вопрос, в том числе и исследование линейных систем уравнений. Просто введите задачу в окошко и нажмите «решить» здесь (например, исследование на совместимость матрицы).

      Где можно решить любую задачу по математике, а так же исследование линейных систем уравнений Онлайн?

      Решить задачу исследование линейных систем уравнений вы можете на нашем сайте https://pocketteacher.ru. Бесплатный онлайн решатель позволит решить онлайн задачу любой сложности за считанные секунды. Все, что вам необходимо сделать — это просто ввести свои данные в решателе. Так же вы можете посмотреть видео инструкцию и узнать, как правильно ввести вашу задачу на нашем сайте. А если у вас остались вопросы, то вы можете задать их в чате снизу слева на странице калькулятора.

      Системы линейных алгебраических уравнений | Высшая математика

      Вернуться к списку тем

      Решить систему уравнений \(\left\{ \begin{aligned}& x_1+x_2-2x_3=4;\\& -5x_1-4x_2+x_3=-11;\\& 2x_1-x_2-3x_3=7. \end{aligned} \right.\) тремя способами: по формулам Крамера, методом Гаусса-Жордана, средствами матричного исчисления. Сделать проверку правильности вычисления обратной матрицы.

      Открыть решение

      Доказать совместность системы линейных уравнений \(\left\{ \begin{aligned}& x_1-2x_2+3x_3=6;\\& 2x_1+3x_2-4x_3=20;\\& 3x_1-2x_2-5x_3=6.\end{aligned} \right.\) и решить её: а) средствами матричного исчисления; б) методом Гаусса; в) по формулам Крамера.

      Открыть решение

      Исследуйте квадратную систему линейных уравнений \(\left\{\begin{aligned}& x_1-\lambda x_2=3;\\& 2x_1+4x_2=\lambda.\end{aligned} \right.\) при различных значениях вещественного параметра \(\lambda\). Решите, если это возможно, данную систему методом Крамера при \(\lambda=\frac{1}{2}\).

      Открыть решение

      Проверить систему \(\left\{ \begin{aligned}& 2x_1-x_2+x_3-x_4=2;\\& x_1+x_2+2x_3=1;\\& x_1-x_2+3x_4=1; \\& 3x_1+3x_3-x_4=3. \end{aligned} \right.\) на совместность. В случае, если система совместна, построить решение.

      Открыть решение

      Доказать совместность системы \(\left\{ \begin{aligned}& -x_1-9x_2-4x_3=-8;\\& 2x_1+7x_2+3x_3+x_4=6;\\& 3x_1+5x_2+2x_3+2x_4=4.\end{aligned} \right.\) и решить её тремя способами: по формулам Крамера, методом Гаусса и средствами матричного исчисления.

      Открыть решение

      Дана система \(\left\{ \begin{aligned}& 2x_1+3x_2+x_3+2x_4-x_5=3;\\& -x_1+x_2+2x_3+2x_4-3x_5=3;\\& x_1+4x_2+3x_3+4x_4-4x_5=6.\end{aligned} \right.\). С помощью теоремы Кронекера-Капелли установить совместность системы.

      Открыть решение

      Исследовать СЛАУ \(\left\{ \begin{aligned}& x_1+2x_2-x_3+x_4=1;\\& 3x_1-x_2+2x_3-x_4=-1;\\& 2x_1+3x_2-2x_3+x_4=-3.\end{aligned} \right.\) на совместность и в случае совместности решить её методом Гаусса. Указать число базисных решений и найти одно из них.

      Открыть решение

      Используя метод Жордана-Гаусса, исследовать совместность системы уравнений \(\left\{ \begin{aligned}& x_1+2x_2+x_3+5x_4+7x_5=8;\\& 2x_1+2x_2+2x_3+6x_4+12x_5=12;\\& x_1+3x_2+3x_3+5x_4+12x_5=16. \end{aligned} \right.\) и если она совместна, то найти её решение. Если система неопределённая, то найти два общих и соответствующие им базисные решения.

      Открыть решение

      Исследовать на совместность и несовместность систему \(\left\{ \begin{aligned}& x_1+x_2+x_3=1;\\& x_1+(1+\lambda)x_2+x_3=1;\\& x_1+x_2+(1+\lambda)x_3=1.\end{aligned} \right.\) и найти общее решение в зависимости от значения параметра \(\lambda\).

      Открыть решение

      Используя теорему Кронекера-Капелли, исследовать на совместность систему линейных уравнений: \(\left\{ \begin{aligned}& 2x_1-x_2+3x_3+5x_4=4;\\& x_1+2x_2+6x_3+3x_4=-2;\\& x_1-8x_2-12x_3+x_4=14; \\& 4x_1+3x_2+15x_3+11x_4=0.\end{aligned} \right.\)

      Открыть решение

      Исследовать однородную СЛАУ \(\left\{ \begin{aligned}& x_1+2x_2+x_3+4x_4+x_5=0;\\& 2x_1+x_2+3x_3+x_4-5x_5=0;\\& x_1+3x_2-x_3+6x_4-x_5=0.\end{aligned} \right.\) на существование нетривиального решения. В случае существования найти общее решение и выделить из него фундаментальную систему решений.

      Открыть решение

      Найти фундаментальную систему решений заданной СЛАУ: \(\left\{ \begin{aligned}& 2x_1+2x_3+2x_4+x_5=0;\\& 6x_1+2x_2+4x_3+5x_4+x_5=0;\\& 6x_1+4x_2+2x_3+4x_4-x_5=0\\& 8x_1+8x_2+4x_4-4x_5=0.\end{aligned} \right.\)

      Открыть решение

      Методом Жордана-Гаусса исследовать совместность системы уравнений: \(\left\{ \begin{aligned}& 2x_1+3x_2+7x_3+2x_4+x_5=13;\\& 4x_1+2x_2+10x_3+12x_4+14x_5=14;\\& 5x_1+3x_2+13x_3+14x_4+16x_5=20.\end{aligned} \right.\)

      Открыть решение

      Исследовать на совместность и найти общее решение системы: \(\left\{ \begin{aligned}& 3x_1-5x_2+2x_3+4x_4=2;\\& 7x_1-4x_2+x_3+3x_4=5;\\& 5x_1+7x_2-4x_3-6x_4=3.\end{aligned} \right.\)

      Открыть решение

      Исследовать на совместность и найти общее решение системы: \(\left\{ \begin{aligned}& 9x_1-3x_2+5x_3+6x_4=4;\\& 6x_1-2x_2+3x_3+4x_4=5;\\& 3x_1-x_2+3x_3+14x_4=-8. \end{aligned} \right.\)

      Открыть решение

      Система уравнений в питоне символьно

      Задавать вопрос

      спросил

      Изменено 1 год, 8 месяцев назад

      Просмотрено 87 раз

      Я хочу решить эту систему символически, но это не сработало. где я сделал ошибку? и как я могу это решить?

       импортировать numpy как np
      из символов импорта sympy, матрица
      Y, C, I0, G0, a, b = символы ('Y, C, I_0, G_0, a, b')
      npA = np.массив (([1, -1], [-b, 1]))
      npd = np.массив ((I0 + G0, а))
      х = np.linalg.solve (npA, npd)
      Икс
       

      Я получаю эту ошибку

       ------------------------------------------------------ --------------------------------------------------
      TypeError Traceback (последний последний вызов)
       в <модуле>
            5 npA = np. массив(([1, -1], [-b, 1]))
            6 npd = np.массив((I0 + G0, а))
      ----> 7 x = np.linalg.solve(npA, npd)
            8 х
      <__array_function__internals> в решении (*args, **kwargs)
      ~\anaconda3\lib\site-packages\numpy\linalg\linalg.py в решении (a, b)
          392 подпись = 'DD->D', если isComplexType(t), иначе 'dd->d'
          393 extobj = get_linalg_error_extobj(_raise_linalgerror_singular)
      --> 394 r = gufunc(a, b, подпись=подпись, extobj=extobj)
          395
          396 возвратная обертка (r.astype (result_t, copy = False))
      TypeError: для ufuncsolve1 не найден цикл, соответствующий указанной сигнатуре и кастингу.
       
      • python
      • system
      • символы
      • решение уравнений

      Вы пытаетесь решить такое уравнение: Ax = b. Я не думаю, что вы можете так смешивать команды из разных библиотек, есть некоторая совместимость, но вы должны проверить документацию

      Здесь возможность

       из символов импорта sympy, уравнение, решить
      a_x, a_y, b_x, b_y = символы ('a_x, a_y, b_x, b_y')
      eq_x = Eq(a_x - a_y, b_x)
      eq_y = Eq(-b_x * a_x + a_y, b_y)
      результат = решить ([eq_x, eq_y], (b_x, b_y))
      печать (результат [b_x])
      печать (результат [b_y])
       

      Выход

       а_х - а_у
      -а_х**2 + а_х*а_у + а_у
       

      2

      Если вам нужна более общая настройка (и более похожая на математический подход), то это может быть полезно

       из символов импорта sympy, Matrix,solve_linear_system
      # a: параметр матрицы
      # b: член неоднородности
      a, x_1, x_2, b_1, b_2 = символы ('b, x_1, x_2, b_1, b_2')
      А = Матрица ([[1, -1], [-а, 1]])
      х = Матрица ([[x_1, x_2]]). T
      б = Матрица ([[b_1, b_2]]).T
      A_augmented = A.row_join(b)
      результат = Solve_linear_system (A_augmented, * x)
      печать (результат)
       

      Выход

       {x_1: (-b_1 - b_2)/(b - 1), x_2: (-b*b_1 - b_2)/(b - 1)}
       

      Примечание

      solve_linear_system принять расширенную матрицу в качестве входных данных [A|b], и вы должны либо расширить неизвестные векторы (как указано выше), либо явно передать все ее координаты 0

      Зарегистрируйтесь или войдите в систему

      Зарегистрируйтесь с помощью Google

      Зарегистрироваться через Facebook

      Зарегистрируйтесь, используя электронную почту и пароль

      Опубликовать как гость

      Электронная почта

      Требуется, но не отображается

      Опубликовать как гость

      Электронная почта

      Требуется, но не отображается

      Символьные решения одновременных УЧП первого порядка в одном неизвестном « Журнал Mathematica

      dx. doi.org/doi:10.3888/tmj.20-2

      Мы предлагаем и реализуем алгоритм решения переопределенной системы дифференциальных уравнений в частных производных в одном неизвестном. Наш подход основан на методе Бура-Майера для определения условий совместимости с помощью скобок Якоби-Майера. Мы рекурсивно решаем совместимые системы, имитируя то, что можно было бы сделать с ручкой и бумагой: решить одно уравнение, подставить его решение в остальные уравнения и повторять процесс, пока не будут исчерпаны уравнения системы. Метод, который мы используем для оценки непротиворечивости базовой системы, отличается от традиционного использования дифференциальных базисов Грёбнера, но кажется более эффективным и простым в реализации.

      Введение

      Поиск решений многих задач приводит к переопределенным системам дифференциальных уравнений в частных производных (УЧП). Эти задачи включают вычисление дискретных симметрий дифференциальных уравнений [1], вычисление дифференциальных инвариантов [2] и определение обобщенных операторов Казимира конечномерной алгебры Ли [3]. В этой статье мы сосредоточимся исключительно на интеграции одновременных систем скалярных УЧП первого порядка; то есть наши системы имеют как минимум два уравнения, одну зависимую переменную (неизвестную функцию) и несколько независимых переменных. Нашей конечной целью является автоматизация поиска общих символьных решений этих систем. Принятый нами подход использует метод Бура-Майера [4] для нахождения условий совместимости (т. е. препятствий для интегрируемости) базовой системы УЧП и итеративного добавления этих условий совместимости к системе до тех пор, пока не будет найдена непротиворечивая или несогласованная система. Это отличается от традиционного подхода, который использует дифференциальные базисы Грёбнера [5] для обнаружения условий совместимости. Когда это применимо, он имеет то преимущество, что его легко реализовать и он эффективен. Недавно, используя технику дифференциальной геометрии, Кругликов и Лычагин [6] распространили метод Бура–Майера на системы УЧП от нескольких зависимых и независимых переменных смешанных порядков (т. е. порядки отдельных уравнений в системе могут быть разными). В нашем подходе для ситуации, когда процесс завершения приводит к непротиворечивой системе, мы решаем последнюю, имитируя то, что можно было бы сделать с ручкой и бумагой: решить одно уравнение, подставить его в следующее уравнение и продолжать процесс до тех пор, пока уравнения системы исчерпаны.

      Чтобы зафиксировать идеи, рассмотрим систему УЧП

      (1)

      где to – независимые переменные, – частная производная неизвестной функции по , а ранг матрицы Якоби – . В дальнейшем мы будем говорить, что свойство выполняется локально, если оно истинно на открытом шаре своей области действия. Система уравнений (1) интегрируема (т. е. допускает локально гладкое решение), если получаемые из нее выражения локально удовлетворяют условиям

      (2)

      Чтобы убедиться в этом, рассмотрим решение системы уравнений (1). Затем локально, . Таким образом, последняя дифференциальная форма локально точна. Так, в частности, он локально закрыт. Следовательно, его внешний дифференциал равен нулю; то есть , или, что то же самое, после некоторых вычислений , откуда следует (2). Обратно, если система уравнений (2) локально выполняется, то дифференциальная форма локально замкнута и по лемме Пуанкаре также локально точна. Следовательно, для некоторой локально гладкой функции . Поэтому локально определяется как , где – произвольная постоянная.

      Бур и Майер (см., например, [4]) показали, что (1) при условии на матрицу Якоби относительно интегрируема тогда и только тогда, когда

      (3)

      всякий раз, когда выполняется (1). Отныне сокращайте фразу «всякий раз, когда (1) выполняется» до .

      Для заданной системы уравнений (1), удовлетворяющей указанному условию невырожденности, возникают четыре случая.

      Первый случай, когда и все скобки Якоби–Майера равны нулю, когда выполняется (1). В этом случае мы можем решить (1) для to . Затем решение системы получается путем интегрирования точной дифференциальной формы.

      Второй случай, когда существуют различные индексы и такие, что . Тогда (1) несовместно и решений нет.

      В третьем случае , и все скобки Якоби–Майера равны нулю в (1). Мы должны дополнить (1) дополнительными уравнениями, пока не дойдем до первого или второго случая. Эти уравнения получаются путем решения системы линейных УЧП первого порядка , где и . Например, дополнительное уравнение , где – произвольная константа, получается путем решения системы линейных УЧП первого порядка , где . Решение пополненной системы зависит от произвольных констант. Общее решение исходной системы уравнений (1) получим, выразив одну из произвольных констант через остальные, затем исключив оставшуюся константу между полученными уравнениями и их частными производными первого порядка по произвольным константы.

      В четвертом и последнем случае некоторые скобки равны нулю в (1), а другие скобки имеют вид , где зависят хотя бы от некоторых . В этом случае мы должны добавить уравнения к уравнениям в (1) и действовать, как в третьем случае.

      Только что описанная процедура является сутью подхода Бура–Майера к решению (1). Необходимо решить переопределенные системы линейных скалярных УЧП и убедиться, что уравнения, добавляемые к исходной системе, совместимы с ними, а уравнения результирующих систем линейно независимы. В нашей реализации подхода Бура–Майера мы дополняем исходную систему уравнений (1), добавляя к ней соответствующие ограничения совместимости, заданные скобками Якоби–Майера, до тех пор, пока не получим либо совместимую, либо несовместимую систему. Начиная с ограничений совместимости, мы итеративно решаем совместимую систему, полученную с помощью встроенной функции . Оставшаяся часть этой статьи посвящена реализации и тестированию этого подхода.

      Реализация и тесты

      Здесь мы сосредоточимся на кодировании алгоритма, описанного во введении. В частности, мы начинаем с итеративного решения системы согласованных УЧП первого порядка с одной зависимой переменной. Затем реализуем тест на непротиворечивость системы УЧП первого порядка по одному неизвестному. Наконец, мы связываем последние две программы таким образом, чтобы одна функция использовалась для вычисления общего решения входной системы, когда оно существует, или для указания на его противоречивость.

      Итеративное решение согласованной системы УЧП первого порядка в одном неизвестном

      Наша программа для итеративного решения совместимой системы скалярных УЧП первого порядка состоит из основной функции и трех вспомогательных функций, , и .

      — рекурсивная функция, принимающая на вход решаемую систему, зависимую переменную, список независимых переменных, контейнер для списка последовательных решений, список уравнений, которые не удалось решить, используемую строку в качестве корня для формирования имен промежуточных зависимых переменных и переменной, которая используется для подсчета и именования промежуточных зависимых переменных. Результатом является список правил и список нерешенных уравнений.

      Функция имитирует то, что можно было бы сделать вручную при решении системы УЧП первого порядка с одним неизвестным: решить уравнение, подставить его решение в оставшиеся уравнения и продолжать до тех пор, пока это возможно. На каждом этапе количество независимых переменных уменьшается на одну и необходимо переименовывать переменные перед продолжением. Кроме того, зависимые переменные являются каррированными функциями, которые необходимо отменить, чтобы обеспечить правильное применение цепного правила во время подстановки в оставшиеся PDE. Это, пожалуй, самая сложная часть нашей реализации.

      Функция берет выходные данные и преобразует их в решение решаемой системы. Вспомогательная функция преобразует выражение, зависящее от нескольких переменных, в чистую функцию этих переменных. Наконец, функция составляет и решает совместимую систему скалярных УЧП. Его входные данные такие же, как и у , а его выходные данные отформатированы как у .

      Тест на совместимость и завершение

      В этом подразделе реализуется тест на совместимость, обеспечиваемый методом Бура-Майера, как описано во введении, с использованием . Входными данными являются базовая система УЧП, зависимая переменная и список независимых переменных; выводит пару: первый элемент указывает, совместима ли система, а второй элемент дает завершенную систему.

      Функция вычисляет попарные скобки Якоби–Майера системы УЧП согласно уравнению (3) и заменяет в этих скобках некоторые частные производные первого порядка неизвестной функции, полученные из базовой системы УЧП. Функция проверяет, содержит ли выражение производную от неизвестной функции.

      Собираем все вместе

      Здесь мы используем определенные до сих пор функции для решения переопределенной системы УЧП первого порядка с одним неизвестным. Функция принимает в качестве аргументов решаемую систему , ее зависимые и независимые переменные и . Функция проверяет, дает ли данное правило решение системы УЧП первого порядка с одним неизвестным.

      Тесты

      Этот подраздел в основном касается примеров, взятых из различных указанных источников. Для удобства предупреждения подавляются встроенной функцией. Используются неопределенные глобальные переменные (, , и т. д.), поэтому убедитесь, что нет конфликтов с вашим собственным сеансом.

      Тест 1

      Представленные здесь примеры возникают при поиске дифференциальных инвариантов гиперболических УЧП [2].

    2. Пример 1
    3. За исключением примера 9, дается для всех систем, поэтому здесь он показан только один раз.

    4. Пример 2
    5. Пример 3
    6. 9 Пример 4 5 и 6 происходят из Салтыкова [7].

    7. Пример 5
    8. Пример 6
    9. Тест 4

      Рассматриваемые здесь две системы PDE находятся в Mansion [4].

    10. Пример 7
    11. Пример 8
    12. Пример 9
    13. Вторая запись показывает, что есть два нерешенных УЧП. Несложно разделить эти УЧП относительно и получить новые УЧП, которые легко решаются с помощью встроенной функции . Разделение может быть выполнено автоматически с помощью следующей однострочной строки.

      Пример 5
    14. Пример 10 11
    15. Пример 12
    16. Последний пример принадлежит Булю [ 8].

    17. Пример 13
    18. Заключение

      В данной статье представлен и реализован алгоритм, основанный на методе Бура–Майера, для решения переопределенной системы УЧП с одним неизвестным. Эффективность нашего подхода мы продемонстрировали на рассмотрении 13 примеров.

      Благодарности

      Выражаю благодарность за частичную финансовую поддержку Центра передового опыта в области математических и статистических наук DST-NRF Школы информатики и прикладной математики Университета Витватерсранда, Йоханнесбург, Южная Африка. Я благодарю профессора Ф. М. Магомеда за выделение необходимых средств и его команду за гостеприимство во время моего визита прошлым летом. Эта статья посвящается моей дочери Катлего в день ее шестнадцатилетия.

      Каталожные номера

      [1] PE Hydon, «Как построить дискретные симметрии дифференциальных уравнений в частных производных», European Journal of Applied Mathematics , 11 (5), 2000, стр. 515–527.
      [2] И. К. Джонпиллай, Ф. М. Магомед и К. Вафо Сох, «Основы совместных инвариантов для () линейных гиперболических уравнений», Journal of Nonlinear Mathematical Physics , 9 (Приложение 2), 2002
      , стр. 49–59. doi:10.2991/jnmp.2002.9.s2.5.
      [3] Дж. К. Ндогмо и П. Винтерниц, «Обобщенные операторы Казимира разрешимых алгебр Ли с абелевыми нильрадикалами», Journal of Physics A: Mathematical and General , 27 (8), 1994
      , стр. 2787–2800. iopscience.iop.org/article/10.1088/0305-4470/27/8/016/meta.
      [4] P. Mansion, Théorie des équations aux dérivées partielles du premier ordre , Paris: Gauthier-Villars, 1875.
      [5] Б. Бухбергер и Ф. Винклер, Основы Грёбнера и приложения , Кембридж: Издательство Кембриджского университета, 1998.
      [6] Б. Кругликов и В. Лычагин, «Совместимость, мультискобки и интегрируемость систем УЧП», Acta Applicandæ Mathematicæ , 109 (1), 2010, стр. 151–196.
      doi:10.1007/s10440-009-9446-0.
      [7] Н. Салтыков, «Классические методы интеграции уравнений с производными частями дю премьер-министра в несоответствующей функции», Мемориал математических наук , 50 , 1931
      стр. 1–72. www.numdam.org/item?id=MSM_ 1931__ 50__ 1_ 0.
      [8] Г. Буль, «Об одновременных дифференциальных уравнениях первого порядка, в которых число переменных превышает число уравнений более чем на единицу», Philosophical Transactions of the Royal Society of London , 152 (5) , 1862, стр.

      Решение тригонометрических примеров: Решение тригонометрических уравнений — 39 примеров!

      § 22. Примеры решения более сложных тригонометрических уравнений и их систем.

         Иногда приходится решать тригонометрические уравнения, в которые входят только сумма или разность синуса и косинуса одного и того же аргумента и их произведение. В таком случае целесообразно эту сумму (или разность) обозначить новой переменной.

      Задача 1. Решите уравнение 3 (sin x + cos x) = 2 sin 2х.

      Комментарий

      Если в заданном уравнении привести все тригонометрические функции к одному аргументу х, то получим уравнение (1) (см. решение), в которое входят только сумма синуса и косинуса одного и того же аргумента х и их произведение. Для решения этого уравнения введем новую переменную sin x + cos x = y. Чтобы получить произведение sin x cos x, достаточно возвести в квадрат обе части равенства замены и учесть, что sin2 x + cos2 x = 1. Выполняя обратную замену, удобно также учесть, что

      Решение

         Данное уравнение равносильно уравнению

                                        3 (sin x + cos x) = 4 sin х cos x.                                      (1)

      Если обозначить sin x + cos x = у, то

      Тогда  Подставляя эти значения в уравнение (1), получаем

      Таким образом, sin x + cos x = 2 или sin x+cos x =

      Тогда  или  Получаем  (корней нет, поскольку ) или  Отсюда  Тогда

      Ответ:

         З а м е ч а н и е. При возведении обеих частей уравнения в квадрат можно получить посторонние корни (см. таблицу 7). Но возведение обеих частей равенства замены в квадрат является равносильным преобразованием. Действительно, в этом случае левая и правая части равенства имеют одинаковые знаки, и тогда a = b Если обе части равенства a = b положительны, то для положительных значений t функция y =возрастает и поэтому каждое свое значение принимает только при одном значении аргумента. Таким образом, при a > 0, b > 0 из равенства a = b следует равенство и, наоборот, из равенства следует равенство a = b, что и гарантирует равносильность выполненного преобразования для положительных a и b. Аналогично для  используем то, что для не положительных значений t функция y =убывает и поэтому каждое свое значение принимает только при одном значении аргумента.

         Для решения некоторых тригонометрических уравнений могут применяться свойства функций (соответствующие общие подходы к решению были рассмотрены в § 3, пункт 3.2), в частности, оценка левой и правой частей уравнения.

      Задача 2. Решите уравнение 

               Оценим область значений функции 

               Поскольку  то есть 

               Выясним, существуют ли такие значения х, при которых функция f (x) может принимать наибольшее значение 2. Если cos 6x будет меньше 1, то для того чтобы сумма равнялась 2, необходимо, чтобы значение было больше 1, что невозможно. Аналогично, если допустить, что меньше 1, то для того чтобы сумма равнялась 2, необходимо, чтобы значение cos 6x было больше 1, что невозможно. Таким образом, равенство в данном уравнении возможно тогда и только тогда, когда cos 6x и равны 1. Поэтому данное уравнение равносильно системе

               Приравнивая правые части этих равенств, получаем

               Поскольку k и n — целые числа, то для получения всех решений последнего уравнения в целых числах (см. § 9) достаточно подставить в правую часть последнего равенства вместо п все остатки при делении на 5 и найти, для каких значений п по этой формуле k также будет целым числом. Только при n = 1 получаем целое k = 3. В случае, когда коэффициент 12 при переменной n в числителе дроби и знаменатель 5 — взаимно простые числа, повторение делимости нацело будет только через знаменатель, то есть через 5. Поэтому последнее уравнение имеет решения в целых числах только вида n = 1 + 5m,. Подставляя значение п в одно из решений системы, получаем х = π + 4πm. Эти значения и являются решениями последней системы, а следовательно, и решениями данного уравнения.

      Ответ: х = π + 4πm,.

      Задача 3. Решите уравнение 

      Комментарий

               Преобразуем левую часть по формуле  и оценим область значений функций, стоящих в левой и правой частях уравнения. Решая полученную систему двух уравнений с одним неизвестным, можно несколько упростить выкладки и решить только одно уравнение системы, а для другого проверить, удовлетворяют ли ему полученные решения.

      Решение

               Данное уравнение равносильно уравнению

      (1)

              

       

      Обозначим: . Поскольку 

               Левая часть уравнения (1) меньше или равна 2, а правая часть больше или равна 2. Равенство между ними возможно тогда и только тогда, когда левая и правая части уравнения равны 2, то есть данное уравнение равносильно системе

               Из первого уравнения системы имеем , откуда 

               Проверим, удовлетворяют ли найденные значения второму уравнению системы. Если , тогда sin 8x=0 и поэтому 

      Ответ:

         Иногда для решения тригонометрических уравнений приходится применять тригонометрические формулы, которые приводят к сужению ОДЗ данного уравнения. Такие преобразования могут приводить к потере корней уравнения. Чтобы этого не случилось, можно пользоваться таким о р и е н т и р о м:

      если для решения уравнений (или неравенств) приходится выполнять преобразования, сужающие ОДЗ исходного уравнения (или неравенства), то те значения, на которые сужается ОДЗ, необходимо рассматривать отдельно.

         В таблице 42 указаны тригонометрические формулы, которые могут приводить к сужению ОДЗ, и соответствующие значения переменной, которые приходится проверять при использовании этих формул.

      Чтобы убедиться, что приведенные формулы приводят к сужению ОДЗ, достаточно сравнить области допустимых значений их левых и правых частей.

      Например, рассмотрим формулу 

      ОДЗ левой части: . Для нахождения ОДЗ правой части формулы учитываем, что знаменатель дроби не равен нулю:, таким образом, . То есть ОДЗ правой части задается системой ограничений  Сравнивая ОДЗ левой и правой частей рассмотренной формулы, видим, что ОДЗ правой части содержит дополнительное ограничение. Таким образом, при переходе по этой формуле от ее левой части к правой происходит сужение ОДЗ (отбрасываются именно те значения, которые указаны в таблице: Чтобы не потерять корни данного уравнения, при использовании формулы, значение , необходимо рассмотреть отдельно (конечно, только в том случае, когда оно входит в ОДЗ данного уравнения).

      Приведем пример использования указанного о р и е н т и р а.

      Задача 4. Решите уравнение

      Комментарий

      Если воспользоваться первыми двумя формулами таблицы 42, то мы приведем все тригонометрические выражения в этом уравнении и к одному аргументу, и к одной функции — tg x. Но при использовании указанных формул происходит сужение ОДЗ на значение ,  и вследствие этого можно потерять корни уравнения, если числа такого вида входят в ОДЗ исходного уравнения и являются его корнями. Чтобы этого не случилось, разобьем решение на две части.

      1. Подставляем те значения переменной, на которые сужается ОДЗ, в                уравнение (1). При вычислениях учитываем периодичность функций и формулы приведения.
      2. При (на ОДЗ уравнения (1)) использование формул и приводит к уравнению (2) (см. решение), которое равносильно заданному (на той части ОДЗ, где ), потому что эти формулы сохраняют верное равенство как при переходе от равенства (1) к равенству (2), так и при обратном переходе от равенства (2) к равенству (1). Замена переменной (и обратная замена) также приводит к уравнению, равносильному заданному (на указанной части ОДЗ исходного уравнения).

         Заметим, что ОДЗ уравнения (2) отличается от ОДЗ уравнения (1) только тем, что в нее не входят значения , которые входят в ОДЗ уравнения (1). Поскольку эти «плохие» значения мы учли в процессе решения, то ОДЗ уравнения (1) можно в явном виде не фиксировать (как в приведенном решении). В ответе записываем все корни, которые были получены в первой и второй частях решения.

      Решение

      1. Если , то из данного уравнения получаем:

      – верное равенство.

      Таким образом, – корни уравнения (1).

      1. Если , получаем:

      (2)

       

              Замена tg x = t приводит к уравнению  которое при  и  равносильно уравнению . Тогда 

      Обратная замена даёт: tg x= -1 или , то есть:

         Некоторые тригонометрические уравнения удается решить, исполь­зуя такой ориентир, который условно можно назвать «ищи квадратный трехчлен», то есть:

      попробуйте рассмотреть данное уравнение как квадратное относительно некоторой переменной (или относительно некоторой функции).

      Решение тригонометрических уравнений графически

      Уравнения, с которыми приходится сталкиваться при решении практических задач, как правило, значительно отличаются от тех, которые мы рассматривали. Для таких уравнений иногда вообще нельзя указать никакого способа, который позволял бы найти корни абсолютно точно. В таком случае приходится ограничиваться нахождением лишь приближенных значений корней. Современная математика располагает эффективными методами приближенного решения уравнений. Рассмотрим графический способ решения.

      Пусть, например, нужно решить уравнение

      sin х = 1 — х.

      На одном и том же рисунке начертим два графика: график функции y = sin х и график функции у = 1 — х

      Эти графики пересекаются в одной точке М. Абсцисса этой точки и дает нам единственный корень нашего уравнения:

      х ≈ 0,5.

      Для уточнения полученного результата полезно использовать тригонометрические таблицы или компьютерные программы. При х = 0,5

      sin x ≈ 0,4794,
      1 — х = 0,5;

      следовательно, sin х < 1 — х. Но тогда, как легко понять из рисунка, корень уравнения sin х = 1 — х будет больше, чем 0,5. Проверим значение х = 0,6. Имеем (при х = 0,6):

      sin х ≈ 0,5446,
      1 — х = 0,4;

      следовательно, sin х > 1 — х. Но тогда, как легко понять из того же рисунка, искомый корень x0 должен быть меньше, чем 0,6. Теперь уже мы знаем, что x0 находится в интервале [0,5; 0,6]. Поэтому с точностью до 0,1

      x0 ≈0,5 (с недостатком),
      x0 ≈ 0,6 (с избытком).

      С помощью таблиц можно найти приближенное значение x0 и с точностью до 0,01. Разделим интервал [0,5; 0,6] пополам. В средней точке (x = 0,55) этого интервала

      sin х ≈ 0,5227,
      1 — х = 0,45.

      Опять получаем, что sin х > 1 — х. Следовательно, x0 < 0,55.

      Проверим точку х = 0,52 (она близка к средней точке х = 0,525 интервала [0,50; 0,55], в котором заключен корень x0). При х = 0,52

      sin х ≈ 0,4969,
      1 — х = 0,48.

      Снова sin x > 1 — х; поэтому x0 < 0,52. Итак, 0,50 < x0 < 0,52. Поэтому с точностью до 0,01

      x0 ≈ 0,51.

      Для примера рассмотрим уравнение

      tg x/2 = 2 — x.

      Графики функций у = tg x/2и у = 2 — х пересекаются в бесконечном числе точек. Значит, данное уравнение имеет бесконечное множество корней. Найдем, например, наименьший положительный корень х0. Этот корень является абсциссой точки пересечения графиков. Примерно он равен 1,2.

      Чтобы найти этот корень точнее, воспользуемся таблицами тангенсов В. М. Брадиса (или рассчитаем соответствующие значения в программе «Kалькулятор» или «Excel»). Выпишем значения функций у = tg x/2 и у = 2 — х в окрестности точки х = 1,2.






















      x 1,2 1,3
      y=tg x/2 0,6841 0,7602
      y=2-x 0,8000 0,7000
      tg x/2-(2-x) -0,1159 0,0602

      Как видно из этой таблицы, при переходе от значения х = 1,2 к значению х = 1,3 разность tg x/2 — (2 — х) меняет свой знак на противоположный (с — на +). Значит, в нуль эта разность обращается где-то между значениями 1,2 и 1,3. Следовательно, с точностью до 0,1 х0 ≈ 1,2 (с недостатком) или х0 ≈ 1,3 (с избытком). Используя таблицу тангенсов, можно найти и приближенное значение этого корня
      с точностью до 0,01. Для этого рассмотрим значение х = 1,25, являющееся средним значением чисел 1,2 и 1,3. При х = 1,25

      tg x/2 ≈ 0,7215,

      2 — х = 0,7500.

      Поскольку tg x/2 < 2- х, то х0 >1,25. Итак,

      1,25< х0 < 1,30.

      Теперь испытаем значение х = 1,28, которое близко к среднему значению чисел 1,25 и 1,30. При х = 1,28

      tg x/2 ≈ 0,7445,

      2 — х = 0,7200.

      Теперь уже tg x/2 >2 — х Значит , х0 < 1,28.

      Аналогично, рассматривая значение х = 1,26, мы получили бы tg x/2 < 2 — х и потому х0 > 1,26. Значит,

      1,26 <х0< 1,28.

      Поэтому с точностью до 0,01

      х0 ≈ 1,27

      Если бы нужно было определить, какое это приближенное значение (с недостатком или с избытком), то пришлось бы сравнить значения tg x/2 и 2 — х в точке х = 1,27.

      6.1: Решение тригонометрических уравнений — Математика LibreTexts

      1. Последнее обновление
      2. Сохранить как PDF
    19. Идентификатор страницы
      3342
      • Майкл Коррал
      • Schoolcraft College 9\circ k \quad\text{for \(k=0 \), \(\pm\,1 \), \(\pm\,2 \), \(…\)}
        \nonumber \]

        Это наиболее общее решение уравнения. Часто часть, которая говорит «for \(k=0 \), \(\pm\,1 \), \(\pm\,2 \), \(…\)» опускается, так как обычно понял, что \(k \) варьируется для всех целых чисел.Общее решение в радианах будет:

        \[ А ~=~ 0,6435 \;+\; \pi k \quad\text{for \(k=0 \), \(\pm\,1 \), \(\pm\,2 \), \(…\)}
        \nonumber \]

        Пример 6.

        1 92 \;\theta ~=~ \frac{1}{2} \quad\Rightarrow\quad \cos\;\theta ~=~ \pm\,\frac{1}{\sqrt{2}}
        \quad \Rightarrow\quad \theta ~=~ \frac{\pi}{4}\;,~\frac{3\pi}{4}\;,~\frac{5\pi}{4}\;,~
        \frac{7\pi}{4}~,
        \nonumber \]

        и, поскольку период косинуса равен \(2\pi \), мы должны добавить \(2\pi k \) к каждому из этих углы, чтобы получить общее решение. Но обратите внимание, что приведенные выше углы отличаются кратно \(\frac{\pi}{2} \). Итак, поскольку каждое кратное \(2\pi \) также кратно \(\frac{\pi}{2} \), мы можем объединить эти четыре отдельных ответа в один:

        \[ \boxed{\theta ~=~ \frac{\pi}{4} \;+\; \frac{\pi}{2}\,k}
        \qquad\text{для \(k=0 \), \(\pm\,1 \), \(\pm\,2 \), \( …\)}
        \номер \]

        Пример 6.3

        Решите уравнение \(\;2\,\сек\;\тета ~=~ 1 \).

        Решение:

        Изоляция \(\;\sec\;\theta \) дает нам

        \[ \sec\;\theta ~=~ \frac{1}{2} \quad\Rightarrow\quad \cos\;\theta ~=~ \frac{1}{\sec\;\theta} ~=~ 2~,
        \nonumber \]

        что невозможно. Таким образом, существует \(\fbox{нет решения}\). 92 \;+\; Икс \;-\; 1 ~=~ 0 \quad\Rightarrow\quad x ~=~ \frac{-1 \;\pm\; \sqrt{1 — (4)\,(-1)}}{
        2\,(1)} ~=~ \frac{-1 \;\pm\; \sqrt{5}}{2} ~=~ -1,618\;,~0,618
        \nonumber \]

        по квадратичной формуле элементарной алгебры. Но \(-1,618 < -1 \), поэтому невозможно, чтобы \(\;\sin\theta = x = -1,618 \). Таким образом, мы должны иметь \(\;\sin\;\theta = x = 0,618 \). Следовательно, возможны два решения: \(\theta = 0,666\) рад в QI и его отражение \(\pi - \theta = 2,475\) рад вокруг оси \(y\) в QII. Добавление к ним кратных \(2\pi \) дает нам общее решение:

        \[ \boxed{\theta ~=~ 0,666 \;+\; 2\pi k \quad\text{and}\quad 2,475 \;+\; 2\pi k}
        \qquad\text{for \(k=0 \), \(\pm\,1 \), \(\pm\,2 \), \(…\)}
        \ nonumber \]

        Пример 6.5

        Решите уравнение \(\;\sin\;\theta ~=~ \tan\;\theta \).

        Решение:

        Попробовав тот же метод, что и в предыдущем примере, мы получим

        \[\nonumber \begin{align*}
        \sin\;\theta ~&=~ \tan\;\theta\ \\nonumber
        \sin\;\theta ~&=~ \frac{\sin\;\theta}{\cos\;\theta}\\ \nonumber
        \sin\;\theta~\cos\;\theta ~&=~ \sin\;\theta\\ \nonumber
        \sin\;\theta~\cos\;\theta \;-\; \sin\;\theta ~&=~ 0\\ \nonumber
        \sin\;\theta~(\cos\;\theta \;-\; 1) ~&=~ 0\\ \nonumber
        &\Rightarrow \quad \sin\;\theta ~=~ 0 \quad\text{or}\quad \cos\;\theta ~=~ 1\\ \nonumber
        &\Стрелка вправо\quad \theta ~=~ 0\;, ~\pi \quad\text{or}\quad \theta ~=~ 0\\ \nonumber
        &\Стрелка вправо\quad \theta ~=~ 0\;,~\pi~,
        \end{align*} \ нечисло \]

        плюс кратные \(2\pi \). Итак, поскольку приведенные выше углы кратны \(\pi \), а каждое кратное \(2\pi \) кратно \(\pi \), мы можем объединить два ответа в один для общего решения: 9{-1} \frac{1}{2} = \frac{\pi}{3} \), есть два возможных решения для \(3\theta\): \(3\theta = \frac{\pi {3} \) в QI и его отражение \(-3\theta = -\frac{\pi}{3} \) вокруг оси \(x\) в QIV. Добавление к ним кратных \(2\pi \) дает нам:

        \[ 3\theta ~=~ \pm\,\frac{\pi}{3} \;+\; 2\pi k \qquad\text{for \(k=0 \), \(\pm\,1 \), \(\pm\,2 \), \(…\)}
        \nonumber \ ]

        Итак, разделив все на \(3 \), мы получим общее решение для \(\theta\):

        \[ \boxed{\theta ~=~ \pm\,\frac{\pi}{9} \;+\; \frac{2\pi}{3} k}
        \qquad\text{для \(k=0 \), \(\pm\,1 \), \(\pm\,2 \), \(. ..\)}
        \nonumber \]

        Пример 6.7

        Решите уравнение \(\;\sin\;2\theta ~=~ \sin\;\theta \).

        Решение:

        Здесь мы используем формулу двойного угла для синуса:

        \[\nonumber \begin{align*}
        \sin\;2\theta ~&=~ \sin\;\theta\ \\nonumber
        2\,\sin\theta~\cos\;\theta ~&=~ \sin\;\theta\\ \nonumber
        \sin\;\theta~(2\,\cos\;\theta \;-\;1) ~&=~ 0\\ \номер
        &\Rightarrow\quad \sin\;\theta ~=~ 0 \quad\text{or}\quad \cos\;\theta ~=~ \frac{1}{2}\\ \nonumber
        &\Rightarrow \quad \theta ~=~ 0\;,~\pi \quad\text{or}\quad \theta ~=~ \pm\,\frac{\pi}{3}\\ \nonumber
        &\Rightarrow\ quad \ boxed {\ theta ~ = ~ \ pi k \ quad \ text {and} \ quad \ pm \, \ frac {\ pi} {3} \; + \; 2\pi k}
        \qquad\text{for \(k=0 \), \(\pm\,1 \), \(\pm\,2 \), \(. ..\)}
        \ end{align*}
        \nonumber \]

        Пример 6.8

        Решить уравнение \(\;2\,\sin\;\theta \;-\; 3\,\cos\;\theta ~=~ 1 \).

        Рисунок 6.1.1: Скопируйте и вставьте сюда заголовок. (Авторское право; автор через источник)
        Решение

        Мы будем использовать технику, которую обсуждали в главе 5, для нахождения амплитуды комбинации функций синуса и косинуса. Возьмите коэффициенты \(2\) и \(3\) при \(\;\sin\;\theta\) и \(\;-\cos\;\theta\) соответственно в приведенном выше уравнении и сделайте на них стороны прямоугольного треугольника, как на рис. 6.1.1. Пусть \(\phi \) будет углом, показанным в прямоугольном треугольнике. Участок длины \(3 >0 \) означает, что угол \(\phi \) находится над осью \(x\), а участок длины \(2>0 \) означает, что \(\phi \) находится справа от оси \(y\). Следовательно, \(\phi \) должен быть в QI. Гипотенуза имеет длину \(\sqrt{13} \) по теореме Пифагора, и, следовательно, \(\;\cos\;\phi = \frac{2}{\sqrt{13}} \) и \(\; \sin\;\theta = \frac{3}{\sqrt{13}} \). Мы можем использовать это, чтобы преобразовать уравнение, чтобы решить его следующим образом:

        \[\nonumber \begin{align*}
        2\,\sin\;\theta \;-\; 3\,\cos\;\theta ~&=~ 1\\ \nonumber
        \sqrt{13}\,\left( \tfrac{2}{\sqrt{13}}\,\sin\;\theta \ ;-\; \tfrac{3}{\sqrt{13}}\,\cos\;\theta
        \right) ~&=~ 1\\ \nonumber
        \sqrt{13}\,( \cos\; \phi\;\sin\;\theta \;-\;\sin\;\phi\;\cos\;\theta ) ~&=~ 1\\ \nonumber
        \sqrt{13}\,\sin\ ;(\theta — \phi) ~&=~ 1\quad\text{(по формуле вычитания синусов)}\\ \nonumber
        \sin\;(\theta — \phi) ~&=~ \tfrac{1 }{\sqrt{13}}\\ \номер
        &\Rightarrow\quad \theta — \phi ~=~ 0,281 \quad\text{or}\quad \theta — \phi ~=~ \pi — 0,281 = 2,861\\ \nonumber
        &\Rightarrow\quad \theta ~=~ \фи \;+\; 0,281 \quad\text{or}\quad \theta ~=~ \phi \;+\; 2.861
        \end{align*} \nonumber \]

        Теперь, поскольку \(\;\cos\;\phi = \frac{2}{\sqrt{13}} \) и \(\phi \) равно в QI наиболее общим решением для \(\phi\) является \(\phi = 0,983 + 2\pi k \) для \(k=0 \), \(\pm\,1 \), \(\ ч\,2 \), \(. .. \). Итак, поскольку нам нужно было добавить кратные \(2\pi \) к решениям \(0,281 \) и \(2,861 \) в любом случае, наиболее общее решение для \(\theta \):

        \[\begin{align*}
        \тета ~&=~ 0,983 \;+\; 0,281 \;+\; 2\pi k\quad\text{and}\quad 0,983 \;+\; 2,861 \;+\; 2\pi k\\
        &\Стрелка вправо\четверка \boxed{\theta ~=~ 1,264 \;+\; 2\pi k\quad\text{and}\quad 3.844 \;+\; 2\pi k}
        \quad\text{for \(k=0 \), \(\pm\,1 \), \(\pm\,2 \), \(…\)}
        \ end{align*} \nonumber \]

        Примечание: в примере 6.8, если уравнение было \(\;2\,\sin\;\theta \;+\; 3\,\cos\;\theta ~= ~ 1 \), то мы все равно использовали бы прямоугольный треугольник с длинами сторон \(2\) и \(3\), но вместо формулы вычитания использовали бы формулу сложения синусов.


        Эта страница под названием 6.1: Решение тригонометрических уравнений распространяется в соответствии с лицензией GNU Free Documentation License 1.3 и была создана, изменена и/или курирована Майклом Корралом посредством исходного контента, отредактированного в соответствии со стилем и стандартами платформы LibreTexts; подробная история редактирования доступна по запросу.

        1. Наверх
          • Была ли эта статья полезной?
          1. Тип изделия
            Раздел или Страница
            Автор
            Майкл Коррал
            Лицензия
            ГНУ ФДЛ
            Версия лицензии
            1,3
            Показать страницу TOC
            нет
          2. Теги
            1. источник@http://www.mecmath.net/trig/index.html
            2. тригонометрическое уравнение

          Решение триггерных уравнений не так уж плохо, как кажется!

          Purplemath

          При решении тригонометрических уравнений используются как опорные углы, так и тригонометрические тождества, которые вы запомнили, а также большая часть изученной вами алгебры. Будьте готовы к необходимости подумай , чтобы решить эти уравнения.

          В дальнейшем предполагается, что вы хорошо разбираетесь в значениях тригонометрического отношения в первом квадранте, как работает единичный круг, взаимосвязь между радианами и градусами и как выглядят кривые различных тригонометрических функций, по крайней мере в первый период. Если вы не уверены в себе, вернитесь и сначала просмотрите эти темы.


          Содержание продолжается ниже

          MathHelp.com

          Как и в случае с линейными уравнениями, я сначала выделю член, содержащий переменную:

          sin( x ) + 2 = 3

          sin( x ) = 1

          Теперь я воспользуюсь опорными углами, которые запомнил, чтобы получить окончательный ответ.

          Примечание: В инструкции я указал интервал в градусах, а это значит, что я должен дать ответ в градусах. Да, синус на первом периоде принимает значение 1 при π/2 радианах, но это не тот тип угловой меры, который им нужен, и использование этого в качестве моего ответа, вероятно, приведет к тому, что я, по крайней мере, потеряю мало моментов по этому вопросу.

          Итак, в градусах мой ответ таков:

          x = 90°


          Возникает соблазн быстро вспомнить, что тангенс 60° включает квадратный корень из 3, и дать ответ, но это уравнение не на самом деле нет решения. Я вижу это, когда замедляюсь и делаю шаги. Мой первый шаг:

          тангенс 2 (θ) = −3

          Может ли любой квадрат (тангенса или любой другой триггерной функции) быть отрицательным ? Нет! Так что мой ответ:

          нет решения


          Левая часть этого уравнения делится на множители. Я привык делать простой факторинг, например:

          2 y 2 + 3 y = 0

          y (2 y + 3) = 0

          …а затем решить каждый из факторов. Здесь работает то же самое. Чтобы решить уравнение, которое они мне дали, я начну с разложения на множители:

          Я сделал алгебру; то есть я произвел факторинг, а затем решил каждое из двух уравнений, связанных с факторами. Это создало два триггерных уравнения. Так что теперь я могу сделать триггер; а именно, решение этих двух результирующих тригонометрических уравнений, используя то, что я запомнил о волне косинуса. Из первого уравнения я получаю:

          cos( x ) = 0:

          x = 90°, 270°

          Из второго уравнения я получаю:

          2 cos( x 90 033 ) = sqrt[3]:

          x = 30°, 330°

          Объединяя эти два набора решений, я получаю решение исходного уравнения:

          x = 30°, 90°, 270°, 330° 5

          Во-первых, я перенесу все на одну сторону знака «равно»:

          sin 2 (θ) − sin(θ) − 2 = 0

          Это уравнение является «квадратичным по синусу»; то есть форма уравнения представляет собой формат квадратного уравнения:

          a X 2 + b X + c = 0

          В случае уравнения, которое они хотят, чтобы я решил , X = sin(θ), a  = 1, b  = -1 и c  = -2.

          Так как это квадратичная форма, я могу применить некоторые методы квадратного уравнения. В случае этого уравнения я могу разложить квадратное число на множители:

          sin 2 (θ) − sin(θ) − 2 = 0

          (sin(θ) − 2)(sin(θ) + 1) = 0

          Первый множитель дает мне соответствующее уравнение триггера:

          sin(θ) = 2

          Но синус никогда не больше 1, так что это уравнение не разрешимо; у него нет решения.

          Другой фактор дает мне второе родственное уравнение триггера:

          sin(θ) + 1 = 0

          sin(θ) = −1

          θ = (3/2)π

          Тогда мой ответ: :

          θ = (3/2)π

          (Если вы решаете только градусы в своем классе, значение решения выше равно «270 °».)


          Я могу использовать тождество триггера, чтобы получить квадратное выражение в косинусе:

          cos 2 (α) + cos(α) = sin 2 (α)

          cos 2 (α) + cos(α) = 1 − cos 2 (α)

          2cos 2 903 41 (α) + cos(α) − 1 = 0

          (2cos(α) − 1)(cos(α) + 1) = 0

          cos(α) = 1/2, cos(α) = −1

          Первое уравнение триггера, cos(α) = 1/2, дает мне α = 60° и α = 300°. Второе уравнение дает мне α = 180°. Итак, мое полное решение:

          α = 60°, 180°, 300°


          Я могу использовать тождество двойного угла в правой части, переставить и упростить; тогда я факторизую:

          sin(β) = 2sin(β)cos(β)

          sin(β) − 2sin(β) cos(β) = 0

          sin(β)(1 − 2cos(β) )) = 0

          sin(β) = 0,cos(β) = 1/2

          Синусоида (из первого триггерного уравнения) равна нулю при 0°, 180° и 360°. Но в исходном упражнении 360° не учитываются, так что это последнее значение решения не учитывается в данном конкретном случае.

          Косинус (из второго уравнения триггера) равен 1/2 при 60°, а значит, и при 360° − 60° = 300°. Таким образом, полное решение:

          β = 0°, 60°, 180°, 300°


          Хм… Я действительно ничего не вижу здесь. Конечно, было бы неплохо, если бы одно из этих триггерных выражений было возведено в квадрат…

          Почему бы мне не возвести в квадрат обе стороны и посмотреть, что получится?

          (sin( x ) + cos( x )) 2 = (1) 2

          sin 2 ( x ) + 2sin( x )cos( x ) + cos 2 ( x ) = 1

          [sin 2 90 341 ( x + cos 2 ( x )] + 2sin( x )cos( x ) = 1

          1 + 2sin( x )cos( x ) = 1

          2s in( x )cos( x ) = 0

          sin( x )cos( x ) = 0

          Ага, поди посчитай: я возвел в квадрат и получил то, что мог работать с. Хороший!

          Из последней строки выше либо синус равен нулю, либо косинус равен нулю, поэтому мое решение выглядит так:

          x = 0°, 90°, 180°, 270° !), я возводил в квадрат, чтобы получить это решение, а возведение в квадрат — это «необратимый» процесс.

          (Почему? Если вы возводите что-то в квадрат, вы не можете просто извлечь квадратный корень, чтобы вернуться к тому, с чего вы начали, потому что возведение в квадрат могло где-то изменить знак.)

          Итак, чтобы быть уверенным в моих результатах , мне нужно проверить свои ответы в оригинальное уравнение , чтобы убедиться, что я случайно не создал решения, которые на самом деле не учитываются. Подключив обратно, я вижу:

          sin(0°) + cos(0°) = 0 + 1 = 1

          … так что решение « x = 0°» работает

          sin(90°) + cos(90°) = 1 + 0 = 1

          …поэтому решение « x = 90°» тоже работает

          sin(180°) + cos(180°) = 0 + (-1 ) = −1

          …о, хорошо, значит, « x = 180°» НЕ работает

          sin(270°) + cos(270°) = (−1) + 0 = −1

          . ..поэтому « x = 270°» тоже не работает

          Хорошо, что я проверил свои решения, потому что два из них на самом деле не работают. Они были созданы в процессе возведения в квадрат.

          Мое фактическое решение :

          x = 0°, 90°


          033 )потому( х ) = 0

          … и использовал тождество двойного угла для синуса в обратном порядке вместо деления 2 в предпоследней строке в моих вычислениях. Ответ был бы таким же, но мне нужно было бы учитывать интервал решения:

          2sin( x )cos( x ) = sin(2 x ) = 0

          Тогда 2 x = 0°, 180°, 360°, 540°, д., и разделив 2 из x даст мне x = 0°, 90°, 180°, 270°, что почти такое же решение, как и раньше. После выполнения необходимой проверки (из-за возведения в квадрат) и отбрасывания посторонних решений мой окончательный ответ был бы таким же, как и раньше.

          Трюк с возведением в квадрат в последнем приведенном выше примере встречается нечасто, но если ничего не работает, возможно, стоит попробовать.

        подкоренное число и показатель корня

        • Квадратный корень
        • Арифметический квадратный корень

        Корень  n-ой  степени из числа  a  — это число,  n-ая  степень которого равна  a.  Например, корнем второй степени из  36  будет число  6,  так как:

        62 = 36.

        Для записи корня используется знак  √    (знак корня  или  радикал). Под чертой знака записывается подкоренное число, а над знаком, в левом верхнем углу, показатель корня:

        2√36.

        Подкоренное число — это степень, показатель корня — это показатель степени, корень — основание степени. Если

        ,

        то

        .

        Эта запись читается так: корень  n-ой  степени из числа  a  равен  x.

        Извлечение корня — это действие, обратное возведению в степень, с помощью которого по данной степени и по данному показателю степени находят основание степени.

        Примеры:

        3√125 = 5,   так как   53 = 125;

        2√81 = 9,   так как   92 = 81;

        5√32 = 2,   так как   25 = 32.

        Квадратный корень

        Квадратным корнем из числа  a  называется число, квадрат которого равен  a.

        Например, квадратными корнями из числа  16  являются числа  4  и  -4:

        2√16 = 4   или   2√16 = -4.

        Рассмотрим уравнение

        x2 = a

        при различных значениях   a:

        1. a < 0:

          В данном случае уравнение не будет иметь решений, так как квадрат любого числа всегда является положительным числом или нулём. Следовательно,  x2  не может быть равен отрицательному числу.

        2. a = 0:

          В этом случае уравнение имеет единственное решение:

          x = 0.

        3. a > 0:

          В этом случае уравнение имеет два корня: положительный и отрицательный, модули которых равны. Так как вторая степень отрицательного числа является числом положительным:

          x = ±√a .

        Из рассмотренного примера можно сделать вывод, что для того чтобы из числа можно было извлечь квадратный корень, необходимо, чтобы оно было числом положительным или нулём.

        Арифметический квадратный корень

        Арифметический квадратный корень из положительного числа  a  — это положительное число  x,  квадрат которого равен  a:

        2a = x,   следовательно   x2 = a.

        При обозначении квадратного корня показатель корня опускается, то есть квадратный корень обозначается знаком корня без показателя. Например:

        a  — квадратный корень из  a.

        Обратите внимание, что при чтении выражения слово арифметический опускается.

        Действие, с помощью которого вычисляется квадратный корень, называется извлечением квадратного корня.

        Геометрический и физический смысл определенного интеграла: Геометрический смысл определенного интеграла

        Геометрический и физический смысл определённого интеграла. — Мегаобучалка

        Геометрический смысл определенного интеграла. Если f ( x ) непрерывна и положительна на [ a , b ], то интеграл представляет собой площадь криволинейной трапеции, ограниченной линиями y = 0, x = a , x = b , y = f ( x ) (см. рис. 5.).

        Физический смысл определенного интеграла: пусть материальная точка M движется вдоль числовой оси со скоростью V ( t ), V ( t )≥0 . Тогда путь, пройденный точкой за промежуток времени от t = a до t = b , равен определенному интегралу от скорости: S =

         

         

        Замена переменной в определённом интеграле.

        Пусть для вычисления интеграла  от непрерывной функции сделана подстановка х = φ(t).
        Теорема 1. Если: 1) функция х = φ(t) и ее производная х’ =φ’(t) непрерывны при t ϵ [α;β]; 2) множеством значений функции х = φ(t) при t ϵ [α,β] является отрезок [а; b ]; 3) φ(α) = а и φ(β) = b, то
        =  * φ ’( t ) dt

        Формула называется формулой замены переменной в определенном интеграле. Отметим, что:
        1) при вычислении определенного интеграла методом подстановки возвращаться к старой переменной не требуется
        2) часто вместо подстановки х = φ(t) применяют подстановку t = g(x)
        3) не следует забывать менять пределы интегрирования при замене переменных!

        Интегрирование по частям при вычислении определённого интеграла.

        Пусть U(x) и V(x) — дифференцируемые функции. Тогда d ( U ( x ) V ( x )) = U ( x ) dV ( x ) + V ( x ) dU ( x ) . Поэтому U ( x ) dV ( x ) = d ( U ( x ) V ( x )) – V ( x ) dU ( x ) . Вычисляя интеграл от обеих частей последнего равенства, с учетом того, что ∫ d ( U ( x ) V ( x ))= U ( x ) V ( x )+ C , получаем соотношение

        Называемое формулой интегрирования по частям. Понимают его в том смысле, что множество первообразных, стоящее в левой части, совпадает со множеством первообразных, получаемых по правой части.

         

         

        Вычисление площадей плоских фигур в прямоугольных и полярных координатах

        Допустим, что фигура D предполагает наличие границы ծ D : y = f ( x ), x = a , x = b ( a < b ), f ( x )ϵ C [ a , b ]
        D является криволинейной трапецией и , при условии, что f ( x )>0 на [ a , b ].

        Если D находится ниже оси [ a , b ] (рис. 18.1), то


        Под полярной системой координат понимается совокупность т. O (полюса) и исходящей из данной точки направленной полупрямой l (полярной оси). В качестве полярных координат т. M обозначают числа (полярный радиус) и (полярный угол) (рис. а).

         

        Вычисление длины дуги плоской кривой.

        Кривой линией l -наз. непрерывное отображение отрезка трехмерного пространства. Кривая наз. плоской если она целиком лежит в какой-то плоскости.

        Замечание: Кривую L можно задавать в виде( параметрическое задание кривой) :
        На плоскости бывает задана в таком виде: или в явном виде y = f ( x ) a Предположим, что функция y = f ( x ) непрер. дифференц. Разобьем отрезок [ a , b ] на n частичных отрезков и рассм.точки на кривой с координатами. Рассмотрим ломаную соверш.в этих точках, тогда длина кривой L =

        Геометрический и физический смысл определенного интеграла — КиберПедия

        Навигация:

        Главная Случайная страница Обратная связь ТОП Интересно знать Избранные

        Топ:

        Оценка эффективности инструментов коммуникационной политики: Внешние коммуникации — обмен информацией между организацией и её внешней средой…

        Теоретическая значимость работы: Описание теоретической значимости (ценности) результатов исследования должно присутствовать во введении. ..

        Когда производится ограждение поезда, остановившегося на перегоне: Во всех случаях немедленно должно быть ограждено место препятствия для движения поездов на смежном пути двухпутного…

        Интересное:

        Мероприятия для защиты от морозного пучения грунтов: Инженерная защита от морозного (криогенного) пучения грунтов необходима для легких малоэтажных зданий и других сооружений…

        Берегоукрепление оползневых склонов: На прибрежных склонах основной причиной развития оползневых процессов является подмыв водами рек естественных склонов…

        Лечение прогрессирующих форм рака: Одним из наиболее важных достижений экспериментальной химиотерапии опухолей, начатой в 60-х и реализованной в 70-х годах, является…

        Дисциплины:

        Автоматизация Антропология Археология Архитектура Аудит Биология Бухгалтерия Военная наука Генетика География Геология Демография Журналистика Зоология Иностранные языки Информатика Искусство История Кинематография Компьютеризация Кораблестроение Кулинария Культура Лексикология Лингвистика Литература Логика Маркетинг Математика Машиностроение Медицина Менеджмент Металлургия Метрология Механика Музыкология Науковедение Образование Охрана Труда Педагогика Политология Правоотношение Предпринимательство Приборостроение Программирование Производство Промышленность Психология Радиосвязь Религия Риторика Социология Спорт Стандартизация Статистика Строительство Теология Технологии Торговля Транспорт Фармакология Физика Физиология Философия Финансы Химия Хозяйство Черчение Экология Экономика Электроника Энергетика Юриспруденция

        ⇐ ПредыдущаяСтр 2 из 2

        1. Путь S, пройденный точкой по прямой за время Tt0 со скоростью v=v(t) (v(t) непрерывна на [t0; T]), есть .

        2. Если переменная сила F=f(x) действует в направлении оси Ox (f(x) – непрерывна на [a; b]), то работа этой силы на отрезке [a; b] оси Ох равна .

        3. Если функция f(x) непрерывна и неотрицательна на отрезке [a; b], то геометрически представляет собой площадь криволинейной трапеции, ограниченной сверху графиком функции y=f(x), снизу – отрезком [a; b] оси Ох, с боков – отрезками прямых x=a, x=b.

        Пример 3.5.11.Найти площадь фигуры, ограниченной параболой и гиперболой .

        ○ Найдем точки пересечения параболы и гиперболы, для этого решим систему уравнений:

        ;

        ;

        ;

        , ; , .

        Таким образом, заданные кривые пересекаются в точках А(1; 0) и В(3; 4) (рис. 3.27). Следовательно,

        4,58 (кв. ед.). ●

        Замена переменной в определенном интеграле

        Формула замены переменной в определенном интеграле:

        ,

        где , α и β определяются из условий соответственно.

        Пример 3.5.12. Вычислить .

        Теорема 3.5.6. (Теорема о среднем)Если функция f(x) непрерывна на отрезке [a; b], то в интервале (a; b) найдется такая точка с, что

        .

        Интегрирование по частям в определенном интеграле

        Пусть u=u(x), v=v(x) – непрерывно дифференцируемые на отрезке [a; b] функции. Тогда

        .

        Определенный интеграл с переменным верхним пределом

        Пусть функция f(x) непрерывна на отрезке [a; b]. Рассмотрим интеграл

        ,

        где х – любая точка из [a; b].

        Если F(x) – первообразная функции f(x), т.е. F′(x)=f(x), то согласно формуле Ньютона-Лейбница имеем:

        .

        Отсюда

        .

        Таким образом, производная определенного интеграла с переменным верхним пределом по этому пределу равна значению подынтегральной функции от этого предела.

         

        Несобственные интегралы

        Интегралы с одним или обоими бесконечными пределами получили название несобственных интегралов первого рода. Здесь также, как при вычислении определенных интегралов, можно на практике использовать формулу Ньютона-Лейбница, однако следует помнить, что символ ∞ – не число, а условное обозначение неограниченного возрастания (или убывания) аргумента в процессе его изменения. То есть, со строгих позиций, вычисление несобственного интеграла первого рода – это вычисление некоторого предела, с постоянным использованием теорем о бесконечно малых и бесконечно больших величинах.

        Таким образом:

        ;

        ;

        .

        То есть, символы бесконечности условно заменяются буквенными параметрами, применяется формула Ньютона-Лейбница, после чего обычным образом вычисляются указанные пределы. Если в результате такого расчета получится конечное число А (включая 0), то ответ следует записать в форме: интеграл сходится к значению А. Если же результатом будет +∞ (или –∞) или предел не существует, то ответ: интеграл расходится.

        В практических вычислениях, вполне допустимо не использовать в явной форме операторы lim, но не следует забывать о том, что на самом деле вычисляются пределы, а не конкретные числовые значения.

        Следующим видом несобственных интегралов являются интегралы от функций с разрывом на одном конце (или обоих концах) интервала интегрирования или с разрывом внутри интервала интегрирования. Например: , и т.п. Такие интегралы носят название несобственных интегралов второго рода. Эти интегралы очень опасны, т. к. часто выглядят вполне безобидно (по невнимательности забываем особые точки подынтегральной функции), но применение формулы Ньютона-Лейбница приводит к неверным результатам.

        Вычисление несобственных интегралов второго рода осуществляется приведением к интегралам первого рода (или сумме таких интегралов), то есть, ставится задача вычисления предела относительно точки, в которой подынтегральная функция разрывна.

         


        ⇐ Предыдущая12

        Поделиться с друзьями:

        Индивидуальные и групповые автопоилки: для животных. Схемы и конструкции…

        Общие условия выбора системы дренажа: Система дренажа выбирается в зависимости от характера защищаемого…

        Поперечные профили набережных и береговой полосы: На городских территориях берегоукрепление проектируют с учетом технических и экономических требований, но особое значение придают эстетическим…

        Опора деревянной одностоечной и способы укрепление угловых опор: Опоры ВЛ — конструкции, предназначен­ные для поддерживания проводов на необходимой высоте над землей, водой. ..

        

        Геометрическая интерпретация определенных интегралов

        • Математические сомнения
        • Определенные интегралы

        Объяснение определенного интеграла математическим уравнением определенно сбивает с толку всех на начальном уровне, но геометрическая интерпретация ни для кого не создает проблем. Итак, всем всегда рекомендуется понимать концепцию определенного интегрирования по геометрической системе.

        График функции необходим для объяснения понятия определенного интеграла в геометрической системе. Итак, давайте сначала нарисуем график для функции, а затем выполним еще несколько геометрических шагов для подготовки графика для поддержки геометрической интерпретации.

        1. Пусть $x$ — переменная, представляющая действительные числа по горизонтальной оси $x$.
        2. Давайте рассмотрим функцию, и это функция в терминах $x$. Итак, в математике это записывается как $f(x)$. Соответствующие значения функции для всех значений $x$ отложены по вертикальной оси $y$. Таким образом, математическая связь между значениями $x$ и соответствующими значениями функции $f(x)$ выражается кривой.
        3. Теперь возьмем точку $P$ на оси $x$ и проведем прямую, пересекающую кривую в точке $Q$. Аналогичным образом рассмотрим другую точку на оси $x$, обозначенную как $R$, а затем проведем прямую линию, пересекающую кривую в точке $S$. Помните, что две прямые линии должны быть перпендикулярны горизонтальной оси.
        4. Предположим, что $x$-координата точек $P$ и $R$ обозначена через $a$ и $b$ соответственно.

        Интерпретация

        Образуется замкнутая геометрическая фигура $PQSR$. Площадь замкнутой геометрической формы равна площади, ограниченной кривой функции $f(x)$, прямыми $x = a$ и $x = b$ и осью $x$.

        Область $PQSR$ не является идеальным четырехугольником. Таким образом, найти площадь области по формулам площади невозможно, потому что она не является ни квадратом, ни прямоугольником, ни каким-либо другим четырехугольником. Однако разбиение области на несколько прямоугольников одинаковой длины помогает нам найти ее площадь. Итак, разделите область на несколько прямоугольников одинаковой длины, как показано на рисунке.

        Теперь длина каждого прямоугольника одинакова, и его можно обозначить дифференциальным элементом $\Delta x$ согласно дифференциальному исчислению, но ширина каждого прямоугольника различна. Однако площадь любого прямоугольника можно вычислить по формуле его площади. Интегрирование площадей всех прямоугольников — это концепция нахождения всей площади области $PQSR$.

        Площадь каждого прямоугольника может быть легко вычислена по формуле площади прямоугольника, но мы можем заметить, что мы игнорируем некоторые части области над прямоугольниками. По этой причине сумма площадей всех прямоугольников не равна в точности площади замкнутой геометрической фигуры $PQSR$.

        Если мы уменьшим длину каждого прямоугольника наполовину, то игнорируемая часть области всех прямоугольников уменьшится по сравнению с предыдущим случаем. Теперь сумма площадей всех прямоугольников близка к площади области $PQSR$, но не совсем равна. Однако сумма площадей всех прямоугольников текущего случая больше площадей всех прямоугольников предыдущего случая.

        Два случая показали, что уменьшение длины прямоугольника минимизирует неучтенную площадь над каждым прямоугольником. Следовательно, если длина каждого прямоугольника приблизительно равна нулю, то не учитываемая площадь над каждым прямоугольником также равна нулю. В этом случае сумма площадей всех прямоугольников в точности равна площади замкнутой геометрической фигуры $PQSR$.

        Теперь разделите область области $PQSR$ на несколько прямоугольников одинаковой длины.

        1. Длина каждого прямоугольника очень мала, приблизительно равна нулю, а его длина математически обозначается дифференциалом $dx$. Из-за очень маленькой длины прямоугольник выглядит как прямая линия. Помните, что для удобства мы показываем на рисунке только два прямоугольника.
        2. Ширина каждого прямоугольника является переменной, и ее можно узнать, сравнив ширину двух прямоугольников. Однако ширину любого прямоугольника можно вычислить с помощью функции $f(x)$.

        Следовательно, площадь одного прямоугольника равна произведению длины и ширины прямоугольника.

        $A_R$ $\,=\,$ $dx \times f(x)$

        $\имплицит$ $A_R$ $\,=\,$ $f(x) \times dx$

        $\ ,\,\,\следовательно\,\,\,\,\,\,$ $A_R$ $\,=\,$ $f(x)dx$

        Площадь каждого прямоугольника выражается приведенной выше формулой математически. Итак, складываем площади всех прямоугольников, и их сумма обозначается интегральным символом в исчислении.

        $\implies$ $A$ $\,=\,$ $\displaystyle \int{f(x)}\,dx$

        Функция $f(x)$ является непрерывной функцией. Таким образом, кривая продолжается бесконечно в обоих направлениях. Интеграл от произведения $f(x)$ и $dx$ вычисляет площадь под всей кривой.

        В этом случае площадь под кривой должна быть рассчитана между пределами. Итак, границы должны быть определены, чтобы найти интеграл функции. По этой причине границы $a$ и $b$ отображаются ниже и выше символа интеграла соответственно. {\displaystyle b} {f(x)}\,dx$

        Интеграл функции $f(x)$ от $a$ до $b$ точно вычисляет площадь, ограниченную кривой $f(x)$, линиями $x = a$ и $x = b$, и горизонтальная ось $x$.

        Математические визуализации | Определенный интеграл

        • Геометрия
        • Реальный анализ
        • Комплексный анализ
        • Вероятность
        • История
        • Помощь
        • Контакт
        • Ссылки
        • Карта сайта
        • Обновления
        • Испания

        Определенный интеграл

        Ваш браузер не поддерживает видео тег.

        Интегральное понятие связано с понятием площади. Когда плоская фигура ограничена прямыми линиями, легко вычислить ее площадь. Однако области, ограниченные изогнутыми линиями, сложнее найти (и даже определить). 9n) от n=1 до n=9. Общий результат для произвольного n был получен Ферма.

        Хотя Кавальери не знал термина «функция», мы можем сказать, что одним из его вкладов было рассмотрение проблемы вычисления функции. площадь, ограниченная графиком положительной функции, осью x и двумя вертикальными линиями («криволинейная трапеция» или «площадь под кривой»):

        Мы хотим присвоить этой области число, которое представляет ее площадь, когда функция положительна. Мы будем называть это число определенным интегралом f между a и b.

        Этот интеграл не всегда представляет собой площадь «криволинейной трапеции». Это только случай когда f неотрицательно. Когда f отрицательно, интеграл будет минус площадь. В общем, интеграл — это площадь криволинейной трапеции, лежащей над осью x, уменьшенная на площадь частей, которые лежат ниже оси x.

        Если мы хотим интегрировать линейные функции, проблема проста.

        Проблема усложняется, когда график функции не является линией.

        «Мы будем следовать идее Архимеда. Она состоит в том, чтобы аппроксимировать функцию f горизонтальными (постоянными) функциями, а площадь под f — функцией сумма маленьких прямоугольников» (Ланг)

        В этих случаях мы хотим построить определенный интеграл (число) как результат некоторого предельного процесса. Мы можем начать делить [a,b] на подынтервалы и взять сумма площадей некоторых прямоугольников, аппроксимирующих функцию f в различных точках отрезка. Площадь этих прямоугольников аппроксимирует интеграл. Интеграция – это процесс суммирования.

        Это обозначение, которое мы используем:

        Символ S (удлиненная буква S для суммы) называется знаком интеграла и был введен Лейбницем в 1675 году. Процесс, в результате которого получается результат, называется интеграция. Числа а и b, которые стоят перед знаком интеграла, называются нижним и верхним пределами интегрирования.

        Лейбниц использовал этот символ, потому что считал интеграл суммой бесконечного числа прямоугольников с высотой f (x) и «бесконечно малым». ширина. Его с готовностью приняли многие ранние математики, потому что им нравилось думать об интегрировании как о своего рода «процесс суммирования», который позволял им складывать вместе бесконечно много «бесконечно малых величин».

        Мы попытаемся показать некоторые идеи, лежащие в основе строгого определения интеграла, данного Бернхардом Риманом (1826-1866).

        P является разбиением [a,b].

        Раздел определяет некоторые подынтервалы. Ширина этих подынтервалов может быть разной:

        Учитывая раздел [a,b], мы можем добавить больше чисел к разделу, и тогда мы получим новый раздел с небольшими интервалами. Если мы добавьте достаточное количество промежуточных чисел, тогда интервалы можно сделать сколь угодно малыми.

        Одним из ограничений является использование регулярных подразделений интервала. В этом случае основания прямоугольников равны:

        Для каждого i мы выбираем некоторую точку x i * в [x i , x i+1 ]. Значение f(x i * ) можно рассматривать как высоту прямоугольника.

        «Основная идея, которую мы собираемся осуществить, состоит в том, что по мере того, как мы будем делать интервалы нашего разбиения все меньше и меньше, сумма площадей прямоугольников приблизится к пределу, и этот предел можно использовать для определения площади под кривой» (Ланг).

        Мы можем выбрать, что x i * будет точкой в ​​середине подынтервала (как в матлете и в предыдущих примерах).

        Одним из популярных вариантов является x i * , равный x i , левому концу подынтервала. Тогда высота прямоугольника будет f(x i ):

        Или мы можем выбрать x i * равным x i+1 , правому концу подынтервала. Тогда высота прямоугольника будет f(x я+1 ):

        Выбор этих x i * в [x i , x i+1 ] является произвольным. Затем Риман считал

        Любая из этих сумм называется суммой Римана функции f для P.

        Геометрическая интерпретация: «Это общая площадь n прямоугольников, которая лежит частично под графиком f и частично над ним. Из-за каким бы образом ни были выбраны высоты прямоугольников, мы не можем с уверенностью сказать, меньше ли та или иная сумма Римана или больше интеграла. Но кажется, что перекрытие не должно иметь большого значения; если основания всех прямоугольников узкие достаточно, то сумма Римана должна быть близка к интегралу». (Спивак)

        Если мы увеличим количество прямоугольников, мы (интуитивно) будем иногда ближе к значению, которое является определенным интегралом.

        Тогда можно сказать, что определенный интеграл является пределом сумм Римана при стремлении числа подразделений к бесконечности и ширины каждого подинтервала стремится к нулю. И неважно, какую точку x i * мы выберем в каждом подинтервале.

        «Мораль этой басни состоит в том, что все, что выглядит как хорошее приближение к интегралу, на самом деле им является, при условии, что все длины интервалы в разбиении достаточно малы». (Спивак)

        В математике мы можем изменить функцию и количество прямоугольников. Хотя в каждом интервале высота прямоугольника может быть любым значением функции в точке подынтервала, здесь мы рассматриваем только одну простую возможность: x i * является средней точкой подынтервала. В этом случае суммы Римана называются средними суммами Римана.

        «Интегралы большинства функций невозможно точно определить (хотя их можно вычислить с любой желаемой степенью точности). путем вычисления нижней и верхней сумм). Тем не менее [как мы узнаем позже, например, при изучении основной теоремы исчисления] интеграл от многих функций может быть вычисляется очень легко» (Спивак).

        Аксиоматический подход к интегралу (вслед за Сержем Лангом)

        В своей книге «Первый курс исчисления», прежде чем объяснять суммы Римана, он подчеркивает важность двух свойств, которые будут определять интеграл. для f на [a,b]:

        Пусть a, b — два числа, причем a

        удовлетворяющие следующим свойствам:

        Свойство 1.

        Перемножение трех матриц: Умножение матриц онлайн

        7.1.3. Умножение MathCAD 12 руководство

        RADIOMASTER

        Лучшие смартфоны на Android в 2022 году

        Серия iPhone от Apple редко чем удивляет. Когда вы получаете новый iPhone, общее впечатление, скорее всего, будет очень похожим на ваше предыдущее устройство. Однако всё совсем не так в лагере владельцев устройств на Android. Существуют телефоны Android всех форм и размеров, не говоря уже о разных ценовых категориях. Другими словами, Android-телефон может подойти многим. Однако поиск лучших телефонов на Android может быть сложной задачей.

        1740 0

        Документация Схемотехника CAD / CAM Статьи

        MathCAD 12 MatLab OrCAD P CAD AutoCAD MathCAD 8 — 11

        • Главная
        • /
        • База знаний
        • /
        • CAD / CAM
        • /
        • org/Breadcrumb»>MathCAD 12
        • Линейная алгебра
        • 7.1. Простейшие матричные операции
          • 7.1.1. Транспонирование
          • 7.1.2. Сложение и вычитание
          • 7.1.3. Умножение
        • 7.2. Векторная алгебра
          • 7.2.1. Модуль вектора
          • 7.2.2. Скалярное произведение
          • 7.2.3. Векторное произведение
          • 7.2.4. Векторизация массива
        • 7.3. Вычисление определителей и обращение квадратных матриц
          • 7.3.1. Определитель квадратной матрицы
          • 7.3.2. Ранг матрицы
          • 7.3.3. Обращение квадратной матрицы
          • 7.3.4. Возведение квадратной матрицы в степень
          • 7.3.5. Матричные нормы
          • 7.3.6. Число обусловленности квадратной матрицы
        • 7.4. Вспомогательные матричные функции
          • 7.4.1. Автоматическая генерация матриц
          • 7.4.2. Разбиение и слияние матриц
          • 7. 4.3. Сортировка элементов матриц
          • 7.4.4. Вывод размера матрицы

        При умножении следует помнить, что матрицу размерности MxN допустимо умножать только на матрицу размерности NxP (р может быть любым). В результате получается матрица размерности MхP.

        Чтобы ввести символ умножения, нужно нажать клавишу со звездочкой <*> или воспользоваться панелью инструментов Matrix (Матрица), нажав на ней кнопку Dot Product (Умножение). Умножение матриц обозначается по умолчанию точкой, как показано в листинге 7.5.

        Листинг 7.5. Перемножение матриц

        Обратите внимание (нижняя строка листинга 7.5), что попытка перемножить матрицы А и В несоответствующего (одинакового 2х3) размера оказалась безрезультатной: после введенного знака равенства находится пустой местозаполнитель, а само выражение в редакторе Mathcad выделяется красным цветом. При установке курсора на это выражение появляется сообщение о несовпадении числа строк первой матрицы с числом столбцов второй матрицы.

        Еще один пример, относящийся к умножению вектора на матрицу-строку и, наоборот, строки на вектор, приведен в листинге 7.6.

        ВНИМАНИЕ!

        Тот же самый оператор умножения действует на два вектора по-другому (см. разд. 7.2.2).

        Листинг 7.6. Умножение вектора и строки

        Аналогично сложению матриц со скаляром определяется умножение и деление матрицы на скалярную величину (листинг 7.7). Символ умножения вводится так же, как и в случае умножения двух матриц. На скаляр можно умножать матрицу любой размерности.

        Листинг 7.7. Умножение матрицы на скалярную величину

        Нравится

        Твитнуть

        Теги MathCad САПР

        Сюжеты MathCad

        Глава 1 Основы работы с системой Mathcad 11

        10102 0

        Глава 10 Работа с информационными ресурсами Mathcad 11

        7088 0

        Глава 2 Работа с файлами Mathcad 11

        12833 0

        Комментарии (0)

        Вы должны авторизоваться, чтобы оставлять комментарии.

        Вход

        О проекте Использование материалов Контакты

        Новости Статьи База знаний

        Радиомастер
        © 2005–2022 radiomaster. ru

        При использовании материалов данного сайта прямая и явная ссылка на сайт radiomaster.ru обязательна. 0.2357 s

        Ассемблер ARM64 | Умножение матриц

        Последнее обновление: 17.01.2023

        Рассмотрим расспространенную задачу — умножение матриц. Например, у нас есть следующие матрицы

        матрица A

        a11

        a12

        a13

        a21

        a22

        a23

        a31

        a32

        a33

        матрица B

        b11

        b12

        b13

        b21

        b22

        b23

        b31

        b32

        b33

        То результат перемножения матриц будет равен

        a11*b11 + a12*b21 + a13*b31

        a11*b12 + a12*b22 + a13*b32

        a11*b13 + a12*b23 + a13*b33

        a21*b11 + a22*b21 + a23*b31

        a21*b12 + a22*b22 + a23*b32

        a21*b13 + a22*b23 + a23*b33

        a31*b11 + a32*b21 + a33*b31

        a31*b12 + a32*b22 + a33*b32

        a31*b13 + a32*b23 + a33*b33

        То есть для получения элемента результирующей матрицы, элементы строки первой матрицы последовательно перемножаются на элементы столбцов второй матрицы, и результаты умножений складываются.

        С точки зрения псевдокода это выглядело бы так:

        
        FOR row = 1 to 3
        	FOR col = 1 to 3
        		acum = 0  
        		FOR i = 1 to 3
        			acum = acum + A[row, i]*B[i, col]
        		NEXT I
        		C[row, col] = acum
        	NEXT col
        NEXT row
        

        Напишем программу для перемножения двух матриц размером 3×3. Для упрощения вывода результатов на консоль воспользуемся функции printf языка С.

        Итак, определим файл main.s со следующим кодом:

        
        // Умножение двух матриц 3x3
        //
        // Используемые регистры:
        // W1 - Индекс строки
        // W2 - Индекс столбца
        // X4 - Адрес строки
        // X5 - Адрес столбца
        // X7 - накопленная сумма для элемент результирующей матрицы
        // W9 - элемент матрицы A
        // W10 - элемент матрицы B
        // X19 - элемент матрицы C
        // X20 - счетчик цикла для печати
        // X12 - номер стороки в цикле dotloop
        // X6 - номер столбца в цикле dotloop
        .global main    // точка входа в  программу
        	. equ N, 3      // Размер матрицы
        	.equ WDSIZE, 4 // размер элемента в байтах
        main:
            STR LR, [SP, #-16]!         // сохраняем значение регистра LR
            STP X19, X20, [SP, #-16]!   // сохраняем значения регистров X19 и X20
            MOV W1, #N                  // помещаем в W1 индекс строки
            LDR X4, =A                  // В X4 адрес текущей строки матрицы А
            LDR X19, =C                 // В X19 адрес материцы С
        rowloop:
            LDR X5, =B                  // первый столбец матрицы B
            MOV W2, #N                  // индекс столбца (считаем в обратном порядке до 0)
        colloop:
            MOV X7, #0                  // регистр для накопления результата - по умолчанию равен 0
            MOV W0, #N                  // счетчик цикла
            MOV X12, X4                 // в X12 помещаем адрес строки для перемножения элементов
            MOV X6, X5                  // в X6 помещаем адрес столбца для перемножения элементов
        dotloop:    // Цикл для умножения элементов текущей строки матрицы A на элементы текущего столбца матрицы B
            LDR W9, [X12], #WDSIZE      // загружаем A[row, i] и увеличиваем адрес в X12 на #WDSIZE байт
            LDR W10, [X6], #(N*WDSIZE)  // загружаем в W10 данные из B[i, col]
            SMADDL X7, W9, W10, X7      // умножаем и сохраняем результат в X7
            SUBS W0, W0, #1             // уменьшаем счетчик на 1
            B. NE dotloop                // если W0 не равно 0, то переходим к dotloop для перемножения новых элементов матриц
            STR W7, [X19], #4           // сохраняем результат из W7 в X19 - C[row, col] = W7, увеличиваем адрес в X19 на 4 байта
            ADD X5, X5, #WDSIZE         // Переходим к следующему столбцу в матрице B - увеличиваем адрес в X5 на #WDSIZE байт
            SUBS W2, W2, #1             // увеличиваем счетчик столбцов
            B.NE colloop                // если не все столбцы прошли, то переходим обратно к colloop
            ADD X4, X4, #(N*WDSIZE)     // к адресу в X4 прибавляем #(N*WDSIZE) байт для перехода к адресу новой строки
            SUBS W1, W1, #1             // уменьшаем счетчик строк
            B.NE rowloop                // если еще есть строки, переходим обратно к rowloop
        
            MOV W20, #3                 // проходим по трем строкам
            LDR X19, =C                 // адрес результирующей матрицы C
            // выводим матрицу с помощью функции printf языка C
        printloop:
            LDR X0, =prtstr             // загружаем строку форматирования для функции printf
            LDR W1, [X19], #WDSIZE      // первый элемент текущей строки матрицы
            LDR W2, [X19], #WDSIZE      // второй элемент текущей строки матрицы   
            LDR W3, [X19], #WDSIZE      // третий элемент текущей строки матрицы
            BL printf                   // Вызыв функции printf
            SUBS W20, W20, #1           // уменьшаем счетчик строк
            B. NE printloop              // если есть еще строки, переходим обратно к printloop
        
            MOV X0, #0                  // код возврата из функции
            LDP X19, X20, [SP], #16     // восстанавливаем значение регистров
            LDR LR, [SP], #16           // восстанавливаем регистр LR
            RET                         // выход из функции
        .data
            // первая матрица
            A:  .word 1, 2, 3
                .word 4, 5, 6
                .word 7, 8, 9
            // вторая матрица
            B:  .word 9, 8, 7
                .word 6, 5, 4
                .word 3, 2, 1
        // результирующая матрица
            C: .fill 9, 4, 0
            prtstr: .asciz "%3d %3d %3d\n"
        
        

        Вкратце рассмотрим данный код. Прежде всего в секции .data определены три матрицы. Матрицы A и B состоят из 9 элементов типа .word, то есть чисел размером 4 байта. Матрица C определена с помощью директивы .fill, которая определяет набор из 9 элементов размеров 4 байта, каждый из которых по умолчанию равен 0.

        Для упрощения работы определяем две дополнительные константы:

        
        .equ N, 3      // Размер матрицы
        .equ WDSIZE, 4 // размер элемента в байтах
        

        Вначале определяем значения для прохода по строкам:

        
        MOV W1, #N                  // помещаем в W1 индекс строки
        LDR X4, =A                  // В X4 адрес текущей строки матрицы А
        LDR X19, =C                 // В X19 адрес материцы С
        

        В W1 помещаем счетчик строк, то есть число 3 (надо пройти по трем строкам матрицы А). В регистр X4 загружается адрес матрицы A (адрес первого ее элемента) и в X19 помещается адрес матрицы С, в которую будем сохранять результат.

        Дальше начинаем цикл для прохода по строкам и определяем значения для прохода по столбцам матрицы B:

        
        rowloop:
            LDR X5, =B                  // первый столбец матрицы B
            MOV W2, #N                  // индекс столбца (считаем в обратном порядке до 0)
        

        В регистр X5 помещается адрес матрицы В, а в W2 — счетчик столбцов (то есть надо пройтись по 3 столбцам матрицы В).

        Затем начинаем цикл по столбцам

        
        colloop:
            MOV X7, #0                  // регистр для накопления результата - по умолчанию равен 0
            MOV W0, #N                  // счетчик цикла
            MOV X12, X4                 // в X12 помещаем адрес строки для перемножения элементов
            MOV X6, X5                  // в X6 помещаем адрес столбца для перемножения элементов
        

        В регистр Х7 помещаем число 0 — этот регистр будет накапливать значения для одного элемента матрицы С. Кроме того, в W0 помещается счетчик цикла — надо перемножить 3 элемента из строки матрицы А и столбца марицы B. Регистр Х12 будет указывать на адрес текущего элемента строки матрицы А, а Х6 — на адрес текущего элемента столбца матрицы В.

        Далее перемножаем все элементы текущей строки матрицы А и текущего столбца матрицы В:

        
        dotloop:    // Цикл для умножения элементов текущей строки матрицы A на элементы текущего столбца матрицы B
            LDR W9, [X12], #WDSIZE      // загружаем A[row, i] и увеличиваем адрес в X12 на #WDSIZE байт
            LDR W10, [X6], #(N*WDSIZE)  // загружаем в W10 данные из B[i, col]
            SMADDL X7, W9, W10, X7      // умножаем и сохраняем результат в X7
            SUBS W0, W0, #1             // уменьшаем счетчик на 1
            B. NE dotloop                // если W0 не равно 0, то переходим к dotloop для перемножения новых элементов матриц
        

        Для перемножения в W9 загружаем элемент по адресу X12, при этом увеличиваем данный адрес на #WDSIZE (4) байт, то есть адрес будет указываать на следующий элемент текущей строки матрицы А. Аналогично в W10 загружаем элемент по адресу X6, при этом увеличиваем данный адрес на #(N*WDSIZE) (12) байт, то есть адрес будет указываать на следующий элемент текущего столба матрицы В. Инструкция SMADDL перемножает значения W9 и W10 и прибавляет к содержимому в регистре X7. И пока не пройдем по всем 3 элементам текущих строки и столбца, продолжаем данные действия.

        Таким образом, в цикле dotloop пройдем по 3 ячейкам текущих строки и столбца, получим результ и сохраним его в регистр X12.

        Далее для вычисления элемента матрицы C в следующем столбце переходим к следующему столбцу матрицы B:

        
        STR W7, [X19], #4           // сохраняем результат из W7 в X19 - C[row, col] = W7, увеличиваем адрес в X19 на 4 байта
        ADD X5, X5, #WDSIZE         // Переходим к следующему столбцу в матрице B - увеличиваем адрес в X5 на #WDSIZE байт
        SUBS W2, W2, #1             // увеличиваем счетчик столбцов
        B. NE colloop                // если не все столбцы прошли, то переходим обратно к colloop
        

        При этом полученный результат из X7 сохраняем в текущей элемент матрицы С, адрес которого хранится в Х19. При этом данный адрес увеличиваем на 4 байта, чтобы в следующий раз сохранить значение для следующего элемента матрицы С. Также увеличиваем адрес в X5 на #WDSIZE байт, то есть переходим к новому столбцу матрицы В и уменьшаем счетчик столбцов в регистре W2.

        Если все столбцы матрицы В пройдены, то переходим к новой строке матрицы А:

        
        ADD X4, X4, #(N*WDSIZE)     // к адресу в X4 прибавляем #(N*WDSIZE) байт для перехода к адресу новой строки
        SUBS W1, W1, #1             // уменьшаем счетчик строк
        B.NE rowloop                // если еще есть строки, переходим обратно к rowloop
        

        Для этого изменяем адрес в Х4 на #(N*WDSIZE) байт (по сути на 12 байт — размер строки), уменьшаем счетчик строк в W1 и переходим к следующей строке.

        Если все строки перебраны, то переходим к печати на консоль — проходим по трем строкам матрицы С и единосременно с помощью строки форматирования prtstr выводим значения трех столбцов текущей строки матрицы C.

        Поскольку в данном случае используется функция языка С, скомпилируем приложение с помощью следующей команды:

        
        aarch64-none-linux-gnu-gcc main.s -o main -static
        

        После запуска программа отобразит нам результирующую матрицу:

        
         30  24  18
         84  69  54
        138 114  90
        

        НазадСодержаниеВперед

        3 способа умножения матриц в Python

        Бала Прия С в Разработка | Последнее обновление: 1 июля 2022 г.

        Поделись на:

        Сканер безопасности веб-приложений Invicti — единственное решение, обеспечивающее автоматическую проверку уязвимостей с помощью Proof-Based Scanning™.

        В этом уроке вы узнаете, как умножить две матрицы на Питоне.

        Вы начнете с изучения условия правильного умножения матриц и напишете пользовательскую функцию Python для умножения матриц. Далее вы увидите, как можно добиться того же результата, используя вложенные генераторы списков.

        Наконец, вы приступите к использованию NumPy и его встроенных функций для более эффективного выполнения матричного умножения.

        Как проверить правильность умножения матриц

        Прежде чем писать код Python для умножения матриц, давайте вернемся к основам умножения матриц.

        Матрица Умножение двух матриц A и B допустимо, только если количество столбцов в матрице A равно , равному количеству строк в матрице B .

        Вероятно, вы уже сталкивались с этим условием умножения матриц раньше. Однако задумывались ли вы когда-нибудь, почему это так?

        Ну, это из-за того, как работает умножение матриц. Взгляните на изображение ниже.

        В нашем общем примере матрица A имеет м рядов и n столбцов. А матрица B имеет n строк и p столбцов.

        Какова форма матрицы продуктов?

        Элемент с индексом (i, j) результирующей матрицы C является скалярным произведением строки i матрицы A и столбца j матрицы B.

        Итак, чтобы получить элемент с определенным индексом в результирующей матрице C вам нужно будет вычислить скалярное произведение соответствующей строки и столбца в матрицах A и B соответственно.

        Повторяя вышеописанный процесс, вы получите матрицу произведения C формы m x p — с m строками и p столбцами, как показано ниже.

        А скалярное произведение или скалярное произведение между двумя векторами a и b задается следующим уравнением.

        Подведем итоги:

        • Очевидно, что скалярное произведение определяется только между векторами одинаковой длины.
        • Таким образом, чтобы скалярное произведение между строкой и столбцом было действительным — при умножении двух матриц — вам нужно, чтобы они обе имели одинаковое количество элементов.
        • В приведенном выше общем примере каждая строка в матрице A содержит n элементов. И каждый столбец в матрице B также имеет n элементов.

        Если присмотреться, то n — это количество столбцов в матрице A, а также количество строк в матрице B. И именно поэтому вам нужно количество столбцов в матрице A должно быть равным числу строк в матрице B .

        Надеюсь, вы понимаете условие выполнения матричного умножения и то, как получить каждый элемент в матрице произведения.

        Давайте продолжим писать код Python для умножения двух матриц.

        Напишите пользовательскую функцию Python для умножения матриц

        В качестве первого шага давайте напишем пользовательскую функцию для умножения матриц.

        Эта функция должна выполнять следующие действия:

        • Принимать две матрицы, A и B, в качестве входных данных.
        • Проверить правильность умножения матриц между A и B.
        • Если верно, перемножьте две матрицы A и B и верните матрицу произведения C.
        • В противном случае верните сообщение об ошибке, что матрицы A и B нельзя перемножить.

        Шаг 1 : Создайте две матрицы целых чисел, используя функцию NumPy random.randint() . Вы также можете объявить матрицы как вложенные списки Python.

         импортировать numpy как np
        np.random.seed (27)
        A = np.random.randint (1,10, размер = (3,3))
        B = np.random.randint (1,10, размер = (3,2))
        print(f"Матрица A:\n {A}\n")
        print(f"Матрица B:\n {B}\n")
        # Выход
        Матрица А:
         [[4 99]
         [9 1 6]
         [9 2 3]]
        Матрица Б:
         [[2 2]
         [5 7]
         [4 4]] 

        Шаг 2: Идем дальше и определяем функцию multiple_matrix(A,B) . Эта функция принимает две матрицы A и B в качестве входных данных и возвращает матрицу произведения C , если умножение матриц допустимо.

         по умножению_матрицы (A, B):
          глобальный C
          если A.shape[1] == B.shape[0]:
            C = np.zeros((A.shape[0],B.shape[1]),dtype = int)
            для строки в диапазоне (строки):
                для столбца в диапазоне (столбцы):
                    для elt в диапазоне (len (B)):
                      C[строка, столбец] += A[строка, элт] * B[элт, колонка]
            вернуть С
          еще:
            return "Извините, я не могу умножить A и B. " 

        Разбор определения функции

        Перейдем к разбору определения функции.

        Объявить C как глобальную переменную : По умолчанию все переменные внутри функции Python имеют локальную область видимости. И вы не можете получить к ним доступ извне функции. Чтобы сделать матрицу продукта C доступной извне, нам придется объявить ее как глобальную переменную. Просто добавьте глобальный квалификатор перед именем переменной.

        Проверка правильности умножения матриц: Используйте атрибут формы , чтобы проверить, можно ли умножить A и B. Для любого массива arr , arr.shape[0] и arr.shape[1] укажите количество строк, и столбцов, соответственно. Таким образом, if A.shape[1] == B.shape[0] проверяет правильность умножения матриц. Только если это условие равно True , будет вычислена матрица произведения. В противном случае функция возвращает сообщение об ошибке.

        Использовать вложенные циклы для вычисления значений: Чтобы вычислить элементы результирующей матрицы, мы должны перебрать строки матрицы A, и внешний цикл for делает это. Внутренний цикл for помогает нам пройти через столбец матрицы B. А самый внутренний цикл for помогает получить доступ к каждому элементу в выбранном столбце.

        ▶️ Теперь, когда мы узнали, как работает функция Python для умножения матриц, давайте вызовем функцию с матрицами A и B, которые мы сгенерировали ранее.

         умножить_матрицу (А, В)
        # Выход
        массив([[ 89, 107],
               [47, 49],
               [ 40, 44]]) 

        Поскольку умножение матриц между A и B допустимо, функция multi_matrix() возвращает матрицу произведения C.

        Использование понимания вложенных списков Python для умножения матриц написал функцию Python для умножения матриц. Теперь вы увидите, как можно использовать вложенные списки, чтобы сделать то же самое.

        Вот понимание вложенного списка для умножения матриц.

        Сначала это может показаться сложным. Но мы будем шаг за шагом разбирать понимание вложенного списка.

        Давайте сосредоточимся на анализе одного списка и определим, что он делает.

        Мы будем использовать следующий общий шаблон для понимания списка:

         [ для  в ]
        где,
        : что вы хотите сделать — выражение или операцию
        : каждый элемент, над которым вы хотите выполнить операцию.
        : итерируемый объект (список, кортеж и т. д.), который вы просматриваете в цикле 

        ▶️ Ознакомьтесь с нашим руководством «Понимание списков в Python — с примерами», чтобы получить более глубокое понимание.

        Прежде чем продолжить, обратите внимание, что мы хотели бы построить результирующую матрицу C по одной строке за раз.

        Объяснение понимания вложенного списка

        Шаг 1: Вычисление одного значения в матрице C

        Для заданной строки i матрицы A и столбца j матрицы B приведенное ниже выражение дает запись с индексом (i, j) в матрице C.

         сумма(a*b для a,b в zip(A_row, B_col)
        # zip(A_row, B_col) возвращает итератор кортежей
        # Если A_row = [a1, a2, a3] и B_col = [b1, b2, b3]
        # zip(A_row, B_col) возвращает (a1, b1), (a2, b2) и т. д. 

        Если i = j = 1 , выражение вернет запись c_11 матрицы C. Таким образом, вы можете получить один элемент в одной строке таким образом.

        Шаг 2: Построить одну строку в матрице C

        Наша следующая цель — построить всю строку.

        Для строки 1 в матрице A вам нужно перебрать все столбцы в матрице B, чтобы получить одну полную строку в матрице C.

        Вернуться к шаблону понимания списка.

        • Заменить <сделать это> с выражением из шага 1, потому что это то, что вы хотите сделать.
        • Затем замените на B_col — каждый столбец в матрице B.
        • Наконец, замените на zip(*B) — список, содержащий все столбцы в матрице B.

        А вот и первое понимание списка.

         [сумма (a * b для a, b в zip (A_row, B_col)) для B_col в zip (* B)]
        # zip(*B): * оператор распаковки
        # zip(*B) возвращает список столбцов матрицы B 

        Шаг 3: Постройте все строки и получите матрицу C

        Далее вам нужно будет заполнить матрицу произведения C, вычислив остальные строки.

        А для этого нужно перебрать все строки в матрице A.

        Еще раз вернуться к пониманию списка и сделать следующее.

        • Замените на понимание списка из шага 2. Вспомните, что мы вычислили целую строку на предыдущем шаге.
        • Теперь замените with A_row — каждая строка в матрице A.
        • И ваш — это сама матрица A, когда вы перебираете ее строки.

        И вот наше окончательное понимание вложенного списка. for A_row in A]

        Пришло время проверить результат! ✔

         # преобразование в  com/numpy-reshape-arrays-in-python/">массив NumPy с помощью np.array()
        C = np.array([[sum(a*b для a,b в zip(A_row, B_col)) для B_col в zip(*B)]
            для A_row в A])
        # Выход:
        [[ 89107]
         [ 47 49]
         [ 40 44]] 

        Если вы присмотритесь, это эквивалентно вложенным циклам for, которые мы использовали ранее, только более лаконично.

        Вы также можете сделать это более эффективно, используя некоторые встроенные функции. Давайте узнаем о них в следующем разделе.

        Использование NumPy matmul() для умножения матриц в Python

        np.matmul() принимает две матрицы в качестве входных данных и возвращает произведение, если умножение матриц между входными матрицами равно действительный .

         C = np.matmul(A,B)
        печать(С)
        # Выход:
        [[ 89 107]
         [ 47 49]
         [ 40 44]] 

        Обратите внимание, насколько этот метод проще, чем два метода, которые мы изучали ранее. На самом деле вместо np.matmul() можно использовать эквивалентный оператор @, и мы сразу это увидим.

        Как использовать оператор @ в Python для умножения матриц

        В Python @ — это бинарный оператор, используемый для умножения матриц.

        Он работает с двумя матрицами и, как правило, с N-мерными массивами NumPy и возвращает матрицу произведения.

        Примечание: Для использования оператора @ у вас должен быть Python 3.5 или более поздней версии.

        Вот как вы можете его использовать.

         С = А@В
        печать(С)
        # Выход
        массив([[ 89, 107],
               [47, 49],
               [ 40, 44]]) 

        Обратите внимание, что матрица произведения C такая же, как та, которую мы получили ранее.

        Можно ли использовать np.dot() для умножения матриц?

        Если вы когда-либо сталкивались с кодом, который использует np.dot() для умножения двух матриц, вот как это работает.

         C = np.dot(A,B)
        печать(С)
        # Выход:
        [[ 89 107]
         [ 47 49]
         [ 40 44]] 

        Вы увидите, что np. dot(A, B) также возвращает ожидаемую матрицу продукта.

        Однако, согласно документам NumPy, вы должны использовать np.dot() только для вычисления скалярного произведения двух одномерных векторов, а не для умножения матриц.

        Напомним из предыдущего раздела, что элемент с индексом (i, j) матрицы произведения C является скалярным произведением строки i матрицы A и столбца j матрицы B.

        Поскольку NumPy неявно передает эту операцию скалярного произведения всем строкам и всем столбцам, вы получаете результирующую матрицу произведения. Но чтобы ваш код был читабельным и чтобы избежать двусмысленности, используйте вместо этого np.matmul() или оператор @ .

        Заключение

        🎯 В этом уроке вы узнали следующее.

        • Условие правильности умножения матриц: количество столбцов в матрице A = количество строк в матрице B .
        • Как написать пользовательскую функцию Python, которая проверяет правильность умножения матриц и возвращает матрицу произведения. В теле функции используются вложенные циклы for.
        • Далее вы узнали, как использовать вложенные списки для умножения матриц. Они более лаконичны, чем циклы for, но подвержены проблемам с читабельностью.
        • Наконец, вы узнали, как использовать встроенную функцию NumPy np.matmul() для умножения матриц и как это наиболее эффективно с точки зрения скорости.
        • Вы также узнали об операторе @ для умножения двух матриц в Python.

        На этом мы завершаем обсуждение умножения матриц в Python. В качестве следующего шага узнайте, как проверить, является ли число простым в Python. Или решить интересные задачи на строки Python.

        Приятного обучения!🎉

        Спасибо нашим спонсорам

        Более 20 примеров для умножения матриц NumPy

        В этом уроке мы рассмотрим различные способы выполнения умножения матриц с использованием массивов NumPy. Мы научимся перемножать матрицы разных размеров вместе.

        Также мы узнаем, как ускорить процесс умножения с помощью графического процессора и другие горячие темы, так что давайте начнем!

        Прежде чем мы двинемся дальше, лучше рассмотреть некоторые основные термины матричной алгебры.

         

         

        Основные термины

        Вектор:  Алгебраически вектор представляет собой набор координат точки в пространстве.
        Таким образом, вектор с двумя значениями представляет собой точку в двумерном пространстве. В информатике вектор — это расположение чисел в одном измерении. Он также широко известен как массив, список или кортеж.
        Напр. [1,2,3,4]

        Матрица:  Матрица (множественное число матриц) представляет собой двумерное расположение чисел или набор векторов.
        Пример:

         [[1,2,3],
        [4,5,6],
        [7,8,9]] 

        Скалярное произведение:  Скалярное произведение – это математическая операция между  2 векторами одинаковой длины .
        Равен сумме произведений соответствующих элементов векторов.

        С четким пониманием этих терминов мы готовы к работе.

         

        Умножение матриц на вектор

        Начнем с простой формы умножения матриц — между матрицей и вектором.

        Прежде чем мы продолжим, давайте сначала поймем, как создать матрицу с помощью NumPy.

        Метод array() NumPy используется для представления векторов, матриц и многомерных тензоров. Давайте определим 5-мерный вектор и матрицу 3 × 3, используя NumPy.

         импортировать numpy как np
        а = np.массив ([1, 3, 5, 7, 9])
        б = np.массив([[1, 2, 3],
                     [4, 5, 6],
                     [7, 8, 9]])
        print("Вектор a:\n", a)
        Распечатать()
        print("Matrix b:\n", b) 

        Вывод:


        Давайте теперь посмотрим, как происходит умножение между матрицей и вектором.

        При умножении матрицы на вектор следует помнить о следующих моментах:

        1. Результатом умножения матрицы на вектор является вектор.
        2. Каждый элемент этого вектора получается путем скалярного произведения между каждой строкой матрицы и умножаемым вектором.
        3. Количество столбцов в матрице должно быть равно количеству элементов в векторе.


        Мы будем использовать метод NumPy matmul() для большинства наших операций умножения матриц.
        Давайте определим матрицу 3×3 и умножим ее на вектор длины 3.

         импортировать numpy как np
        а = np.массив([[1, 2, 3],
                     [4, 5, 6],
                     [7, 8, 9]])
        б = np.массив ([10, 20, 30])
        печать("А=", а)
        печать("б =", б)
        печать ("Ab =", np.matmul (а, б)) 

        Вывод:

        Обратите внимание, что результатом является вектор длины, равной строкам матрицы множителей.

        Умножение на другую матрицу

        Теперь мы поняли умножение матрицы на вектор; было бы легко вычислить умножение двух матриц.
        Но перед этим повторим самые важные правила умножения матриц:

        1. Количество столбцов в первой матрице должно быть равно количеству строк во второй матрице.
        2. Если мы умножаем матрицу размеров m x n на другую матрицу размеров n x p, то результатом будет матрица размеров m x p.

        Рассмотрим умножение m x n матрицы A на n x p матрицу B: 
        Произведение двух матриц C = AB будет иметь m строк и p столбцов.
        Каждый элемент в матрице произведения C является результатом скалярного произведения между вектором-строкой в ​​A и вектором-столбцом в B.


        Теперь давайте выполним матричное умножение двух матриц в Python, используя NumPy.
        Мы случайным образом сгенерируем две матрицы размеров 3 x 2 и 2 x 4.
        Мы будем использовать метод np.random.randint() для генерации чисел.

         импортировать numpy как np
        np.random.seed (42)
        A = np.random.randint (0, 15, размер = (3,2))
        B = np.random.randint (0, 15, размер = (2,4))
        print("Матрица А:\n", А)
        print("форма A =", A.shape)
        Распечатать()
        print("Матрица B:\n", B)
        print("shape of B =", B.shape) 

        Вывод:

        Примечание: мы устанавливаем случайное начальное число, используя ‘np. random.seed()’, чтобы сделать генератор случайных чисел детерминированным.
        При каждом запуске этого фрагмента кода будут генерироваться одни и те же случайные числа. Этот шаг необходим, если вы хотите воспроизвести результат позже.

        Вы можете установить любое другое целое число в качестве начального числа, но я предлагаю установить его на 42 для этого урока, чтобы ваши выходные данные соответствовали показанным на выходных снимках экрана.

        Теперь умножим две матрицы, используя Метод np.matmul() . Результирующая матрица должна иметь форму 3 x 4.

         C = np.matmul(A, B)
        print("произведение A и B:\n", C)
        print("shape of product =", C.shape) 

        Вывод:

        Умножение между 3 матрицами

        Умножение трех матриц будет состоять из двух операций умножения 2 матриц, и каждая из двух операций будут следовать тем же правилам, которые обсуждались в предыдущем разделе.

        Допустим, мы перемножаем три матрицы A, B и C, и произведение равно D = ABC.
        Здесь количество столбцов в A должно быть равно количеству строк в B, а количество строк в C должно быть равно количеству столбцов в B.

        Полученная матрица будет иметь строк, равных количеству строк в A и столбцов равно количеству столбцов в C.

        Важным свойством операции умножения матриц является то, что является ассоциативным .
        При мультиматричном умножении порядок отдельных операций умножения не имеет значения и, следовательно, не дает разных результатов.

        Например, в нашем примере умножения трех матриц D = ABC не имеет значения, выполняем ли мы сначала AB или BC.


        Оба порядка дадут один и тот же результат. Давайте сделаем пример на Python.

         импортировать numpy как np
        np.random.seed (42)
        A = np.random.randint (0, 10, размер = (2,2))
        B = np.random.randint (0, 10, размер = (2,3))
        C = np.random.randint (0, 10, размер = (3,3))
        print("Матрица A:\n{}, shape={}\n".format(A, A.shape))
        print("Матрица B:\n{}, shape={}\n". format(B, B.shape))
        print("Матрица C:\n{}, shape={}\n".format(C, C.shape)) 

        Результат:

        На основании правил, которые мы обсуждали выше, умножение этих трех матриц должно дать результирующую матрицу формы (2, 3).
        Обратите внимание, что метод np.matmul() принимает только две матрицы в качестве входных данных для умножения, поэтому мы будем вызывать метод дважды в том порядке, в котором мы хотим умножать, и передавать результат первого вызова в качестве параметра в второй.
        (Мы найдем лучший способ решения этой проблемы в следующем разделе, когда мы введем оператор @)

        Выполним умножение в обоих порядках и проверим свойство ассоциативности.

         D = np.matmul(np.matmul(A,B),C)
        print("Результат умножения в порядке (AB)C:\n\n{},shape={}\n".format(D, D.shape))
        D = np.matmul(A, np.matmul(B,C))
        print("Результат умножения в порядке A(BC):\n\n{},shape={}".format(D, D.shape)) 

        Вывод:

        Как мы видим, Результат умножения трех матриц остается тем же, независимо от того, умножаем ли мы сначала А и В или сначала В и С.
        Таким образом, свойство ассоциативности подтверждается.
        Кроме того, результирующий массив имеет форму (2, 3), что соответствует ожидаемым строкам.

         

        NumPy Умножение трехмерных матриц

        Трехмерная матрица — это не что иное, как набор (или стек) множества двумерных матриц, точно так же, как двумерная матрица представляет собой набор/стек множества одномерных векторов.

        Таким образом, матричное умножение трехмерных матриц включает в себя многократное умножение двумерных матриц, что в конечном итоге сводится к скалярному произведению между их векторами строк и столбцов.

        Рассмотрим пример матрицы A формы (3,3,2), умноженной на другую трехмерную матрицу B формы (3,2,4).

         импортировать numpy как np
        np.random.seed (42)
        A = np.random.randint (0, 10, размер = (3,3,2))
        B = np.random.randint (0, 10, размер = (3,2,4))
        print("A:\n{}, shape={}\nB:\n{}, shape={}".format(A, A.shape,B, B.shape)) 

        Вывод:

        Первая матрица представляет собой стопку из трех двумерных матриц, каждая из которых имеет форму (3,2), а вторая матрица представляет собой стопку из трех двумерных матриц, каждая из которых имеет форму (2,4).

        Умножение матриц между этими двумя включает три умножения между соответствующими двумерными матрицами A и B, имеющими формы (3,2) и (2,4) соответственно.

        В частности, первое умножение будет между A[0] и B[0], второе умножение будет между A[1] и B[1] и, наконец, третье умножение будет между A[2] и БИ 2].

        Результат каждого отдельного умножения двумерных матриц будет иметь форму (3,4). Следовательно, конечным продуктом двух трехмерных матриц будет матрица формы (3,3,4).

        Давайте реализуем это с помощью кода.

         C = np.matmul(A,B)
        print("Product C:\n{}, shape={}".format(C, C.shape)) 

        Вывод:

        Альтернативы np.matmul()

        Кроме ‘np.matmul ()’ существует два других способа выполнения матричного умножения — метод np.dot() и оператор ‘@’ , каждый из которых предлагает некоторые различия/гибкость в операциях матричного умножения.

        Метод ‘np.dot()’

        Вы можете использовать этот метод для нахождения скалярного произведения векторов, но если мы передадим две двумерные матрицы, то он будет вести себя аналогично методу ‘np. matmul()’ и будет возвращать результат матричного умножения две матрицы.

        Давайте рассмотрим пример:

         импортировать numpy как np
        # матрица 3x2
        A = np.массив([[8, 2, 2],
                     [1, 0, 3]])
        # матрица 2x3
        B = np.массив([[1, 3],
                     [5, 0],
                     [9, 6]])
        # точечный продукт должен возвращать продукт 2x2
        С = np.точка (А, В)
        print("произведение A и B:\n{} shape={}".format(C, C.shape)) 

        Вывод:

        Здесь мы определили матрицу 3×2, а матрицу 2×3 и их скалярное произведение дает результат 2×2, который представляет собой матричное умножение двух матриц,
        то же самое, что и ‘ np.matmul()’ вернется.

        Разница между np.dot() и np.matmul() заключается в их работе с трехмерными матрицами.
        В то время как np.matmul() работает с двумя трехмерными матрицами путем вычисления матричного умножения соответствующих пар двумерных матриц (как обсуждалось в последнем разделе), np.dot(), с другой стороны, вычисляет скалярные произведения различных пар векторы-строки и векторы-столбцы из первой и второй матрицы соответственно.

        np.dot() для двух трехмерных матриц A и B возвращает произведение суммы по последней оси A и предпоследней оси B.
        Это неинтуитивно и нелегко понять .

        Итак, если A имеет форму (a, b, c), а B имеет форму (d, c, e), то результат np.dot(A, B) будет иметь форму (a, d, b,e), отдельный элемент которого в позиции (i,j,k,m) определяется как:

         dot(A, B)[i,j,k,m] = sum(A[i,j,: ] * B[k,:,m]) 

        Проверим на примере:

         импортировать numpy как np
        np.random.seed (42)
        A = np.random.randint (0, 10, размер = (2,3,2))
        B = np.random.randint (0, 10, размер = (3,2,4))
        print("A:\n{}, shape={}\nB:\n{}, shape={}".format(A, A.shape,B, B.shape)) 

        Вывод:

        Если теперь мы передадим эти матрицы методу ‘np.dot()’, он вернет матрицу формы (2,3,3,4), отдельные элементы которой вычисляются по приведенной выше формуле.

         C = np.dot(A,B)
        print("np.dot(A,B) =\n{}, shape={}".format(C, C.shape)) 

        Вывод:

        Другое важное различие между ‘np. matmul()’ и ‘np.dot()’ состоит в том, что ‘np.matmul()’ не допускает умножения на скаляр (мы обсудим это в следующий раздел), в то время как ‘np.dot()’ разрешает это.

        Оператор «@»

        Оператор @, представленный в Python 3.5, выполняет ту же операцию, что и «np.matmul()».

        Давайте рассмотрим более ранний пример np.matmul() с использованием оператора @ и увидим тот же результат, что и ранее:

         импортировать numpy как np
        np.random.seed (42)
        A = np.random.randint (0, 15, размер = (3,2))
        B = np.random.randint (0, 15, размер = (2,4))
        print("Матрица A:\n{}, shape={}".format(A, A.shape))
        print("Матрица B:\n{}, shape={}".format(B, B.shape))
        С = А @ В
        print("произведение A и B:\n{}, shape={}".format(C, C.shape)) 

        Вывод:

        Оператор ‘@’ становится удобным, когда мы выполняем матричное умножение из более чем двух матриц.

        Раньше нам приходилось вызывать np.matmul() несколько раз и передавать их результаты в качестве параметра следующему вызову.
        Теперь мы можем выполнить ту же операцию более простым (и более интуитивным) способом:

         import numpy as np
        np.random.seed (42)
        A = np.random.randint (0, 10, размер = (2,2))
        B = np.random.randint (0, 10, размер = (2,3))
        C = np.random.randint (0, 10, размер = (3,3))
        print("Матрица A:\n{}, shape={}\n".format(A, A.shape))
        print("Матрица B:\n{}, shape={}\n".format(B, B.shape))
        print("Матрица C:\n{}, shape={}\n".format(C, C.shape))
        D = A @ B @ C # раньше np.matmul(np.matmul(A,B),C)
        print("Продукт ABC:\n\n{}, shape={}\n".format(D, D.shape)) 

        Вывод:

         

        Умножение на скаляр (одно значение)

        До сих пор мы выполняли умножение матрицы на вектор или другую матрицу. Но что происходит, когда мы выполняем умножение матриц на скалярное или одно числовое значение?

        Результат такой операции получается путем умножения каждого элемента матрицы на скалярное значение. Таким образом, выходная матрица имеет ту же размерность, что и входная матрица.

        Обратите внимание, что ‘np.matmul()’ не позволяет умножать матрицу на скаляр. Вы можете добиться этого, используя np.dot() или с помощью оператора ‘*’.

        Давайте посмотрим на это на примере кода.

         импортировать numpy как np
        A = np.массив([[1,2,3],
                     [4,5, 6],
                     [7, 8, 9]])
        В = А * 10
        print("Матрица A:\n{}, shape={}\n".format(A, A.shape))
        print("Умножение A на 10:\n{}, shape={}".format(B, B.shape)) 

        Вывод:

         

        Поэлементное матричное умножение

        Иногда нам нужно сделать умножение соответствующих элементов двух матриц, имеющих одинаковую форму.


        Эта операция также называется продуктом Адамара .  Он принимает две матрицы одинакового размера и создает третью матрицу того же размера.

        Этого можно добиться, вызвав функцию Multi() NumPy или используя оператор ‘*’ .

         импортировать numpy как np
        np. random.seed (42)
        A = np.random.randint (0, 10, размер = (3,3))
        B = np.random.randint (0, 10, размер = (3,3))
        print("Матрица A:\n{}\n".format(A))
        печать ("Матрица B:\n{}\n".format(B))
        C = np.multiply(A,B) # или A * B
        print("Поэлементное умножение A и B:\n{}".format(C)) 

        Вывод:

        Единственное правило, которое нужно помнить при поэлементном умножении, заключается в том, что две матрицы должны иметь одинаковую форму .
        Однако, если одно измерение матрицы отсутствует, NumPy будет транслировать его, чтобы оно соответствовало форме другой матрицы.

        На самом деле умножение матриц на скаляр также включает передачу скалярного значения в матрицу формы, равной матричному операнду при умножении.

        Это означает, что когда мы умножаем матрицу формы (3,3) на скалярное значение 10, NumPy создаст другую матрицу формы (3,3) с постоянными значениями десять во всех позициях в матрице и выполнит поэлементно умножение двух матриц.

        Давайте разберемся с этим на примере:

         импортировать numpy как np
        np.random.seed (42)
        A = np.random.randint (0, 10, размер = (3,4))
        B = np.массив ([[1,2,3,4]])
        print("Матрица A:\n{}, shape={}\n".format(A, A.shape))
        print("Матрица B:\n{}, shape={}\n".format(B, B.shape))
        С = А * В
        print("Поэлементное умножение A и B:\n{}".format(C)) 

        Вывод:

        Обратите внимание, как вторая матрица, имевшая форму (1,4), была преобразована в ( 3,4) матрица через трансляцию, причем поэлементное умножение между двумя матрицами имело место.

         

        Матрица, возведенная в степень (возведение матрицы в степень)

        Подобно тому, как мы можем возвести скалярное значение в степень, мы можем проделать ту же операцию с матрицами.
        Точно так же, как возведение скалярного значения (основания) в степень n равносильно многократному умножению n оснований, та же закономерность наблюдается при возведении матрицы в степень, которая включает в себя повторяющиеся матричные умножения.

        Например, если мы возведем матрицу A в степень n, она будет равна матричному умножению n матриц, каждая из которых будет матрицей A.


        Обратите внимание: чтобы эта операция была возможной, базовая матрица должна быть квадратной .
        Это делается для того, чтобы количество столбцов в предыдущей матрице было равно количеству строк в следующей матрице.

        Эта операция обеспечивается в Python методом linalg.matrix_power() NumPy, который принимает базовую матрицу и целочисленную степень в качестве параметров.

        Давайте посмотрим на пример в Python:

         импортировать numpy как np
        np.random.seed (10)
        A = np.random.randint (0, 10, размер = (3,3))
        A_to_power_3 = np.linalg.matrix_power(A, 3)
        print("Матрица A:\n{}, shape={}\n".format(A, A.shape))
        print("A в степени 3:\n{}, shape={}".format(A_to_power_3,A_to_power_3.shape)) 

        Вывод:

        Мы можем проверить этот результат, выполнив обычное умножение матриц с тремя операндами (все они A), используя оператор @:

         B = A @ A @ A
        print("B = A @ A @ A :\n{}, shape={}". format(B, B.shape)) 

        Вывод:

        Как видите, результаты обеих операций соответствие.

        В связи с этой операцией возникает важный вопрос: Что происходит, когда мощность равна 0?
        Чтобы ответить на этот вопрос, давайте рассмотрим, что происходит, когда мы возводим скалярную базу в степень 0,9.0423 Получаем значение 1, верно? Теперь, что эквивалентно 1 в матричной алгебре? Вы правильно угадали!

        Это матрица идентичности.

        Таким образом, возведение матрицы n x n в степень 0 приводит к единичной матрице I формы n x n.

        Давайте быстро проверим это на Python, используя нашу предыдущую матрицу A.

         C = np.linalg.matrix_power(A, 0)
        print("A в степени 0:\n{}, shape={}".format(C, C.shape)) 

        Вывод:

        Поэлементное возведение в степень

        Точно так же, как мы могли бы выполнять поэлементное умножение матриц, мы также можем выполнять поэлементное возведение в степень, т. е. возводить каждый отдельный элемент матрицы в некоторую степень.

        Этого можно добиться в Python с помощью стандартного оператора экспоненты ‘ ** ’ — пример оператора, перегружающего .

        Опять же, мы можем предоставить одну постоянную степень для всех элементов в матрице или матрицу степеней для каждого элемента в базовой матрице.

        Давайте посмотрим на примеры того и другого в Python: 9полномочия:\n{}, shape={}\n».format(C, C.shape))

        Вывод:

        Умножение определенного индекса

        Предположим, у нас есть матрица 5 x 6 A и другую матрицу 3 x 3 B. Очевидно, мы не можем перемножить эти две вместе из-за несоответствия размеров

        Но что, если мы хотим умножить подматрицу 3×3 в матрице A на матрицу B, сохраняя при этом другие элементы в A неизменными?
        Для лучшего понимания обратитесь к следующему изображению:


        Вы можете выполнить эту операцию в Python, используя нарезку матрицы для извлечения подматрицы из A, выполняя умножение на B, а затем записывая результат по соответствующему индексу в A.

        Давайте посмотрим это в действии.

         импортировать numpy как np
        np.random.seed (42)
        A = np.random.randint (0, 10, размер = (5,6))
        B = np.random.randint (0, 10, размер = (3,3))
        print("Матрица A:\n{}, shape={}\n".format(A, A.shape))
        print("Матрица B:\n{}, shape={}\n".format(B, B.shape))
        С = А[1:4,2:5] @ В
        А[1:4,2:5] = С
        print("Матрица A после умножения подматриц:\n{}, shape={}\n".format(A, A.shape)) 

        Вывод:

        Как видите, только элементы с индексами строк с 1 по 3 и индексами столбцов со 2 по 4 были умножены на B, и то же самое было записано обратно в A, в то время как остальные элементы A имеют остался неизменным.

        Также нет необходимости перезаписывать исходную матрицу. Мы также можем записать результат в новую матрицу, сначала скопировав исходную матрицу в новую матрицу, а затем записав произведение в позиции подматрицы.

         

        Умножение матриц с использованием графического процессора

        Мы знаем, что NumPy ускоряет матричные операции, распараллелив множество вычислений и используя возможности параллельных вычислений нашего ЦП.

        Однако современным приложениям нужно нечто большее. ЦП предлагают ограниченные вычислительные возможности, и этого недостаточно для большого количества необходимых нам вычислений, как правило, в таких приложениях, как глубокое обучение.

        Именно здесь на сцену выходят графические процессоры. Они предлагают большие вычислительные возможности и превосходную инфраструктуру параллельных вычислений, которая помогает нам сэкономить значительное количество времени, выполняя сотни тысяч операций за доли секунды.

        В этом разделе мы рассмотрим, как можно выполнять умножение матриц на графическом процессоре вместо центрального процессора и сэкономить при этом много времени.

        NumPy не предлагает функции для умножения матриц на графическом процессоре. Поэтому мы должны установить некоторые дополнительные библиотеки, которые помогут нам достичь нашей цели.

        Сначала мы установим библиотеки « scikit-cuda » и « PyCUDA », используя установку pip. Эти библиотеки помогают нам выполнять вычисления на графических процессорах на базе CUDA. Чтобы установить эти библиотеки с вашего терминала, если на вашем компьютере установлен графический процессор.

         pip установить pycuda
        pip install scikit-cuda 

        Если на вашем компьютере нет графического процессора, вы можете попробовать ноутбуки Google Colab и включить доступ к графическому процессору; это бесплатно для использования. Теперь мы напишем код для генерации двух матриц 1000×1000 и выполнения матричного умножения между ними двумя методами:

      • Использование метода scikit-cuda ‘ linalg.mdot() ’ на графическом процессоре
      • Во втором методе мы будем генерировать матрицы на процессоре; затем мы будем хранить их на графическом процессоре (используя PyCUDA’s ‘ gpuarray.to_gpu() ‘метод) перед выполнением умножения между ними. Мы будем использовать модуль « time » для вычисления времени вычислений в обоих случаях.

        Использование ЦП

         импортировать numpy как np
        время импорта
        # генерация матриц 1000 x 1000
        np.random.seed (42)
        x = np.random.randint(0,256, размер=(1000,1000)).astype("float64")
        y = np.random.randint(0,256, размер=(1000,1000)).astype("float64")
        # вычисление времени умножения на CPU
        тик = время.время()
        г = np.matmul (х, у)
        ток = время.время()
        time_taken = toc - тик #время в секундах
        print("Время, затрачиваемое ЦП (в мс) = {}".format(time_taken*1000))
        
         

        Вывод:

        На некоторых старых аппаратных системах вы можете получить ошибку памяти, но если вам повезет, это будет работать в течение длительного времени (зависит от вашей системы).

        Теперь давайте выполним то же самое умножение на графическом процессоре и посмотрим, как отличается время вычислений между ними.

        Использование графического процессора

         #вычисление времени умножения на графическом процессоре
        linalg.init()
        # хранение массивов на GPU
        x_gpu = gpuarray. to_gpu(x)
        y_gpu = gpuarray.to_gpu(y)
        тик = время.время()
        #выполнение умножения
        z_gpu = linalg.mdot(x_gpu, y_gpu)
        ток = время.время()
        time_taken = toc - тик #время в секундах
        print("Время, затраченное на GPU (в мс) = {}".format(time_taken*1000)) 

        Вывод:

        Как мы видим, выполнение той же операции на GPU дает нам ускорение в 70 раз по сравнению с CPU.
        Это все еще было небольшое вычисление. Для крупномасштабных вычислений графические процессоры дают нам ускорение на несколько порядков.

         

        Заключение

        В этом уроке мы рассмотрели, как происходит умножение двух матриц, управляющие им правила и как их реализовать в Python.
        Мы также рассмотрели различные варианты стандартного умножения матриц (и их реализацию в NumPy), такие как умножение более двух матриц, умножение только на определенный индекс или степень матрицы.

        Мы также рассмотрели поэлементные вычисления в матрицах, такие как поэлементное матричное умножение или поэлементное возведение в степень.

        © 2015 - 2019 Муниципальное казённое общеобразовательное учреждение «Таловская средняя школа»

        Карта сайта